◆ わからない問題はここに書いてね 265 ◆

このエントリーをはてなブックマークに追加
1132人目の素数さん
  ・累乗 x^2=x*x(掛け算で×は使わない) ・対数 log_[3](9)=2(底は3)
  ・積分 ∫[x=1,3] (e^(x+3))dx        ・数列の和  Σ[k=1,n]A(k)
  ・分数 (a+b)/(c+d) (分子a+b、分母c+d) ・ベクトル AB↑ a↑
   _        。
 , '´   ヽ      // ̄ ̄ ̄ ̄ ̄ ̄ ̄ ̄ ̄ ̄ ̄ ̄ ̄ ̄ ̄ ̄ ̄ ̄
 ! i iハル)))〉  /  | 上記のように書いてローマ数字や丸付き数字などを
 i!iiリ゚ ヮ゚ノij /   < 避けて頂けると助かりますわ。
 li/([l个j]P´     | また複数のスレッドで質問する行為はご遠慮下さい。
ノノく|_|〉リ        ー――――――――――――――――――
  ,し'ノ  ※累乗や分数などは誤解されぬよう括弧の多用をお願いします

他の記号(>>2-3にもあります)と過去ログ
http://members.at.infoseek.co.jp/mathmathmath/
前のスレッド
http://science6.2ch.net/test/read.cgi/math/1265293548/
よくある質問
http://www.geocities.co.jp/CollegeLife-Club/7442/math/index.html
(その他注意・関連リンクは>>2>>3>>4辺りを参照)
2132人目の素数さん:2010/04/04(日) 22:42:25 BE:170408636-S★(518988)
●スカラー:a,b,...,z, A,...,Z, α,β,...,ω, Α,Β,...,Ω,...(「ぎりしゃ」「あるふぁ〜おめが」で変換)
●ベクトル:V=[v1,v2,...], |V>,V↑,vector(V) (混同しないならスカラーの記号でいい。通常は縦ベクトル)
●テンソル:T^[i,j,k...]_[p,q,r,...], T[i,j,k,...;p,q,r,...]  (上下付き1成分表示)
●行列  M[i,j], I[i,j]=δ_[i,j]  M=[[M[1,1],M[2,1],...],[M[1,2],M[2,2],...],...], I=[[1,0,0,...],[0,1,0,...],...]
(右は全成分表示。行または列ごとに表示する。例:M=[[1,-1],[3,2]])
●転置行列・随伴行列:M ',tM, M†("†"は「きごう」で変換可) ●行列式・トレース:|A|=det(A), tr(A)

●複号:a±b("±"は「きごう」で変換可)
●内積・外積・3重積:a・b, a×b, a・(b×c)=(a×b)・c=det([a,b,c]), a×(b×c)

●関数・数列:f(x), f[x] a(n), a[n], a_n
●平方根:√(a+b)=(a+b)^(1/2)=sqrt(a+b) ("√"は「るーと」で変換可)
●指数関数・対数関数:exp(x+y)=e^(x+y) ln(x/2)=log[e](x/2)(exp(x)はeのx乗、lnは自然対数)
●三角比:sin(a), cos(x+y), tan(x/2)
●絶対値:|x|  ●共役複素数:z~ ●ガウス記号:[x] (関数の変数表示と混同しないよう注意)
●階乗:n!=n*(n-1)*(n-2)*...*2*1, n!!=n*(n-2)*(n-4)*...
●順列・組合せ:P[n,k]=nPk, C[n,k]=nCk, Π[n,k]=nΠk, H[n,k]=nHk ("Π"は「ぱい」で変換可)
3132人目の素数さん:2010/04/04(日) 22:42:33 BE:511223696-S★(518988)
●微分・偏微分:dy/dx=y', ∂y/∂x=y_x ("∂"は「きごう」で変換可)
●ベクトル微分:∇f=grad(f), ∇・A=div(A),∇xA=rot(A), (∇^2)f=Δf
("∇"は「きごう」,"Δ"は「でるた」で変換可.)
●積分:∫[0,1]f(x)dx=F(x)|_[x=0,1], ∫[y=0,x]f(x,y)dy, ∬[D]f(x,y)dxdy, ∬[C]f(r)dl
("∫"は「いんてぐらる」,"∬"は「きごう」で変換可)
●数列和・数列積:Σ[k=1,n]a(k), Π[k=1,n]a(k) ("Σ"は「しぐま」,"Π"は「ぱい」で変換可)
●極限:lim[x→∞]f(x) ("∞"は「むげんだい」で変換可)

●図形:"△"は「さんかく」 "∠"は「かく」 "⊥"は「すいちょく」 "≡"は「ごうどう」 "∽"は「きごう」
●論理・集合:"⇔⇒∀∃∧∨¬∈∋⊆⊇⊂⊃∪∩"は「きごう」で変換
●等号・不等号:"≠≒<>≦≧≪≫"は「きごう」で変換
4132人目の素数さん:2010/04/04(日) 22:42:50 BE:378684858-S★(518988)
【関連スレッド】
雑談はここに書け!【37】
http://science6.2ch.net/test/read.cgi/math/1261794262/
くだらねぇ問題はここへ書け ver.3.14(63桁略)7816
http://science6.2ch.net/test/read.cgi/math/1249096955/
分からない問題はここに書いてね330
http://science6.2ch.net/test/read.cgi/math/1269099055/

【業務連絡】
■レスの数が970ぐらいになったら新しいスレッドを立て、そちらには
  関連リンク・注意書きを、古い方には新スレへの誘導を貼るようお願いします。
■単発質問スレと古いスレに書き込まれた質問は、このスレか関連スレに誘導して下さい。
【削除依頼スレッド】
http://qb5.2ch.net/test/read.cgi/saku/1033142451/l50 (レス削除)
http://qb5.2ch.net/test/read.cgi/saku/1106022021/l50 (スレッド削除)
http://qb5.2ch.net/test/read.cgi/saku2ch/1039177898/l50 (重要削除)

━━━━━━━━━━━━━━━━━━━━━━━━━━━━━━

       ◆ わからない問題はここに書いてね 265 ◆
 移転が完了致しましたわ♪ それでは皆様、遠慮なくお使い下さい。

━━━━━━━━━━━━━━━━━━━━━━━━━━━━━━
5132人目の素数さん:2010/04/06(火) 09:07:26
2chの書き込みがどれくらい減ったかわかりますか?
6132人目の素数さん:2010/04/06(火) 23:20:26
前スレ終了
7132人目の素数さん:2010/04/07(水) 12:08:40
以下の問題がわかりません。

次の集合を外延的記法で表せ.
{x|x∈R,i(x+i)^4∈R)}

○自分なりの回答

i(x+i)^4
=i(x^4+4x^3i-6x^2-4xi+1)
=x^4i-4x^3-6x^2i+4x+i

i(x^4-6x^2+1)=0となるx∈Rの条件を求めればいい.
i(x^4-6x^2+1)=0
(x^4-6x^2+1)=0
x^2=3±√(9-1)
=3±2√2

ここでxを求めることができず、思考が止まりました。
87:2010/04/07(水) 12:11:37
×回答
○解答
9132人目の素数さん:2010/04/07(水) 12:29:13
nxn ラテン方陣(各行と列の数字がすべて異なるもの)の中で
左右斜めの向き(将棋の角の動きの範囲)の数字もすべて異なるものは存在するんですか?

10132人目の素数さん:2010/04/07(水) 12:40:18
斜めの向きには升目がn以上あるやん
11132人目の素数さん:2010/04/07(水) 13:19:19
ないやろ
12132人目の素数さん:2010/04/07(水) 13:22:41
>>11>>10へのレス

>>9
5×5に25まで埋める?1から5までを5組埋める?
13132人目の素数さん:2010/04/07(水) 13:27:09
左向きの斜めと右向きの斜めには同じ数字があってもいいってことか
14132人目の素数さん:2010/04/07(水) 18:14:06
>>7
二重根号でググれ。
15132人目の素数さん:2010/04/07(水) 18:17:09
>>9
1,2,3,..n の各数字をn個ずつすべのnクイーン問題の配置にすることが可能かっていう
意味です。
16132人目の素数さん:2010/04/07(水) 20:52:38
tanh^(−1)xを自然対数であらわせ、そして証明しろと課題がだされました
わからないので教えてください
17132人目の素数さん:2010/04/07(水) 22:26:51
tanhの定義は知ってるのか?
あと、「^(-1)」は逆数なのか?逆関数なのか?
18132人目の素数さん:2010/04/07(水) 22:32:58
逆関数である可能性が高いとみた
それにしても胡散臭いな、こんな問題が実在するとは思えない
19132人目の素数さん:2010/04/07(水) 23:07:22
金1枚=850
銀1枚=450
銅1枚=150

5800にするには、それぞれ何枚ずつですか?
20132人目の素数さん:2010/04/07(水) 23:13:06
>>19 金4銀2銅10
21132人目の素数さん:2010/04/07(水) 23:18:13
>>20
11枚以内の硬貨でが抜けてました。
でも、ありがとう!
そして、すみません。
227:2010/04/08(木) 04:57:44
>>14さん、解けました。
ありがとうございます。
23132人目の素数さん:2010/04/08(木) 07:04:21
>>17-18
tanhの逆関数です
答えは1/2log(1-x)/(1+x)
です
24132人目の素数さん:2010/04/08(木) 09:24:09
>>23
問題の内容はともかく表現が意味不明?

「tanh^(-1)xを自然対数で表せ」または
「tanh^(-1)x=○○を証明しろ」だったらまだわかるんだけど

それはともかく、>>17の言うようにtanhって何なのか知ってるの
25132人目の素数さん:2010/04/08(木) 09:35:49
キチガイは巣に帰れ
2617じゃないが:2010/04/08(木) 09:51:34
はいぱぼりっくたんじぇんとですか?
双曲線関数?ですか?
27132人目の素数さん:2010/04/08(木) 16:05:51
http://imepita.jp/20100408/578110

これを教えてくれませんか?
28132人目の素数さん:2010/04/08(木) 16:19:28
>>27 このくらいテキストで書けよ。画像にする意味ないだろ。
あまりの怠惰さに、答える意欲がなくなる。
29132人目の素数さん:2010/04/08(木) 16:27:10
すいませんでした

z=(√5-1)/4+{√(10+2√5)}i/4
とおく
z^5、|z-1|、|z+1|
を求めよ

これでいいですか?
30132人目の素数さん:2010/04/08(木) 16:30:49
>>29
Z=e^(2/5π)であることに気づけば楽勝
31132人目の素数さん:2010/04/08(木) 16:31:12
>>27 >>29
この z は z^5 - 1 = 0 という方程式の解のひとつなのよ。それがわかれば
問題で聞かれたすべての値はすらすら出てくる。
3230:2010/04/08(木) 16:31:40
typo
Z=e^(2/5πi)であることに気づけば楽勝
33132人目の素数さん:2010/04/08(木) 16:40:12
すいません
どうやって
Z=e^(2/5πi)
って気付くんですか?
有名な問題なのですか?
34132人目の素数さん:2010/04/08(木) 16:50:32
π,π/2,π/3,π/4,π/5,π/6の三角比くらい把握しておくべき。
全暗記しないまでも、見たらそれと気付けるよう目を通しておくと良い。
35132人目の素数さん:2010/04/08(木) 17:00:16
>>33
とにかくz^5を計算してみるべし。経験があれば見当もつきやすい。
36132人目の素数さん:2010/04/08(木) 17:40:33
みなさんありがとうございました
37132人目の素数さん:2010/04/08(木) 19:42:29
中学数学の問題です。
ABとDCが平行な台形ABCDで
対角線の交点をMとすると
(△AMDの面積)=(△BMCの面積)
となることを中学生でもわかるように教えることはできるでしょうか?
よろしくお願いいたします。
38132人目の素数さん:2010/04/08(木) 20:24:01
>>37
等積変形である2つの三角形から、同じ三角形を除けばよい。
39132人目の素数さん:2010/04/08(木) 20:24:38
>>37
(△ACDの面積)=(△BCDの面積) で、両辺から(△MCDの面積)を引く
でわからない?
40132人目の素数さん:2010/04/08(木) 22:15:18
質問です。
(δφ/δt)=v(δφ/δx)の解が
δ= f(x+vt)となるのは何ですか?
41132人目の素数さん:2010/04/08(木) 22:17:10
間違えました。
δ=f(vx+t)となるのは何でですか?
42132人目の素数さん:2010/04/08(木) 22:20:36
本当すいません。φ=f(vx+t)でした。
43132人目の素数さん:2010/04/08(木) 22:23:43
代入してみ
44132人目の素数さん:2010/04/08(木) 22:33:32
代入してみた事あります。確かになりますが。。何かピンと来ないといいますか。

何か直感的に分かり易い証明とかありますか?
4537:2010/04/08(木) 22:41:07
>>38
>>39
ありがとうございます。
よくわかりました。
46132人目の素数さん:2010/04/09(金) 12:12:12
ちょっとスレ違いかもしれませんが、応用数学の院生の就職先ってどんなところがありますか?
47132人目の素数さん:2010/04/09(金) 14:21:25
SEとか塾講師とか
48132人目の素数さん:2010/04/09(金) 14:32:06
>>21
金 銀 銅   枚数
4  5  1   10
4  4  4   12
4  3  7   14
4  2  10   16
4  1  13   18
4  0  16   20

1  11  0   12
1  10  3   14
1  9  6   16
1  8  9   18
  ………
  中略
  ………
1  1  30   32
1  0  33   34
49132人目の素数さん:2010/04/09(金) 15:30:02
>>33
複素平面の知識があれば
特に方針を知らなくても
Z^5を求めようとして
まずZ^2を出し、
そのあとZ^3かZ^4を求めたあたりで気づくんじゃないかな

とにかく手をうごかしてみることだ
50132人目の素数さん:2010/04/09(金) 17:12:38
sinπ/15の出し方教えてくれませんか
51132人目の素数さん:2010/04/09(金) 17:18:34
>>50
そのままぐぐってみる
52132人目の素数さん:2010/04/09(金) 18:33:20
>>50 sin(π/5)は知ってる?
53132人目の素数さん:2010/04/10(土) 00:36:50
>>52
知ってます
54132人目の素数さん:2010/04/10(土) 00:39:18
>>50
55132人目の素数さん:2010/04/10(土) 00:59:14
↑アホ発見
56132人目の素数さん:2010/04/10(土) 01:13:42
アホではなく嫌がらせです
57132人目の素数さん:2010/04/10(土) 01:14:53
「そのまま」ググったらどうなると思う
58132人目の素数さん:2010/04/10(土) 01:30:29
どうなるんだ?
59132人目の素数さん:2010/04/10(土) 01:31:23
試せばいいだろww
60132人目の素数さん:2010/04/10(土) 01:32:31
精神年齢が一挙に低下
61132人目の素数さん:2010/04/10(土) 01:34:03
試して質問者の意図した答えが出るのか?
62132人目の素数さん:2010/04/10(土) 01:57:28
lim an=+∞ならば
lim1/n(a1+a2+.......+an)=+∞
であることをan=a+εを考えることによりε-N論法をもちいて証明せよ
ちなみにlimは全部n→∞です
63132人目の素数さん:2010/04/10(土) 02:08:32
>>61
だからそれも含めて嫌がらせなんだろ
64132人目の素数さん:2010/04/10(土) 02:38:05
>>53
であるならば、sin(π/5)からsin(π/10)、cos(π/10)を導出する。
π/15=π/6-π/10なので、あとは加法定理で。
65132人目の素数さん:2010/04/10(土) 09:13:51
「そのまま」てのは
「sinπ/15」じゃなくて
「sinπ/15の出し方教えてくれませんか」でぐぐるんだよ
66132人目の素数さん:2010/04/10(土) 09:40:06
すみません
税理士試験の勉強をしてる文系の馬鹿な私に助言をください
とある方程式が解けません

2.5x*4/6=330.000

結論から言うとx=198.000なのですが
式を解こうとすると割り切れずに解けません
どうすれば解けるのでしょうか?
67132人目の素数さん:2010/04/10(土) 09:42:01
>>50
> sinπ/15の出し方教えてくれませんか
sin(π/15)と書け、と何度言われたら・・・
68132人目の素数さん:2010/04/10(土) 09:44:11
微分方程式をyについて解く問題なんですが
dy/dx=e^(x+y)+2xe^(x^2+y)
の答えは
y=log(e^x+e^(x^2)+cで合ってますか?検算すればいいと思うのですが、複雑になって解けませんでした
もし間違ってたら解答を教えてください
69132人目の素数さん:2010/04/10(土) 09:54:43
>>68
マルチはするな、と何度言われたら・・・
70132人目の素数さん:2010/04/10(土) 10:00:41
>>68
違う。
71132人目の素数さん:2010/04/10(土) 10:43:16
>>66
どう解こうとして、何が割り切れなかったのか詳しく
72132人目の素数さん:2010/04/10(土) 10:50:09
まさかとは思うが文系の人間って
自分がどこのスレで質問すべきか、なんて考えるアタマがないとか言わないよね
73132人目の素数さん:2010/04/10(土) 10:51:16
>>66
> 2.5x*4/6=330.000
まず 2.5*x*(4/6)=330.000 の両辺に6/4を掛けて(つまり、6を掛けてから4で割って)
2.5*x*(4/6)*(6/4)=330.000*(6/4)
これから 2.5*x=495.000
この両辺を2.5で割って
x=198.000
74132人目の素数さん:2010/04/10(土) 12:08:00
>>70
具体的に教えてほしいです。解が異なるなら解答もお願いします。
75132人目の素数さん:2010/04/10(土) 12:50:15
>>68
検算が複雑でできないという答をどうやって導出したんだよ?
76132人目の素数さん:2010/04/10(土) 12:55:32
パッと見ただけでめんどくさくてやる気が起きないのでてめえら代わりにやれ

の婉曲表現
77132人目の素数さん:2010/04/10(土) 16:21:39
解答者側の質が著しく低下しているな。
中卒の煽りしかいない。
78132人目の素数さん:2010/04/10(土) 16:51:40
>>77
ヒント
時期 受験失敗 ストレス
79132人目の素数さん:2010/04/10(土) 16:52:01
いつの時期もだろ
80132人目の素数さん:2010/04/10(土) 17:12:04
問題←→解答
質問←→回答

ここは質問スレだから回答者
81132人目の素数さん:2010/04/10(土) 17:14:21
丸投げ君の対義語として解答者ということで
あながち間違いではなさそう
82132人目の素数さん:2010/04/10(土) 17:23:43
流れたので

微分方程式をyについて解く問題なんですが
dy/dx=e^(x+y)+2xe^(x^2+y)
の答えは
y=log(e^x+e^(x^2)+cで合ってますか?検算すればいいと思うのですが、複雑になって解けませんでした
もし間違ってたら解答を教えてください
83132人目の素数さん:2010/04/10(土) 17:32:13
違うって書いてあるだろう。
検算もできないレベルの人に解を教えても猫に小判。
84132人目の素数さん:2010/04/10(土) 17:36:33
>>83
一番下の行参照お願いします
85132人目の素数さん:2010/04/10(土) 17:39:30
それを踏まえて言っている。
ウソ教えたって、確認する能力ないんだろ。
繰り返すが、そんな奴に解を教えても猫に小判だ。
86132人目の素数さん:2010/04/10(土) 17:41:13
積分定数がlogの外に出るワケないだろJK
87132人目の素数さん:2010/04/10(土) 17:46:15
>>85
分からないなら一々レスしないで頂きたい、レスが流れるので
それでも違うというからには解を示して見てください、過程はいいので。あなたの浅知恵程度なら看破できると思います。
あなたにできればの話ですが
88132人目の素数さん:2010/04/10(土) 17:47:22
>>86
y=log(e^x+e^(x^2)+c)でした、訂正します
89132人目の素数さん:2010/04/10(土) 17:49:13
ひでえww
90132人目の素数さん:2010/04/10(土) 17:54:20
>>85
おいどうした、まだか?括弧も脳内補完できなかったのか?早くしろ
91132人目の素数さん:2010/04/10(土) 17:56:57
ここまでどれが俺の自演?
92132人目の素数さん:2010/04/10(土) 17:58:35
>>88
残念ちょっち違う。
符号を見直すと正解にたどり着けるかも。
93132人目の素数さん:2010/04/10(土) 18:20:14
>>87>>90
ツマラン
94132人目の素数さん:2010/04/10(土) 18:32:02
>>85
どうしたーーーーーーー?まだかーーーー?
95132人目の素数さん:2010/04/10(土) 18:44:28
合ってるよ
96132人目の素数さん:2010/04/10(土) 18:53:12
>>94
まだあ?
97132人目の素数さん:2010/04/10(土) 19:07:43
>>88
> >>86
> y=log(e^x+e^(x^2)+c)でした、訂正します
これについてdy/dxを求めてみなさい。
98132人目の素数さん:2010/04/10(土) 19:26:05
>>97
だから解を示してみ、はよしろ
99132人目の素数さん:2010/04/10(土) 19:28:17
なんだ荒らしか
相手して損した
次の質問どうぞ↓
100132人目の素数さん:2010/04/10(土) 19:30:54
流れたので

微分方程式をyについて解く問題なんですが
dy/dx=e^(x+y)+2xe^(x^2+y)
の答えは
y=log(e^x+e^(x^2)+cで合ってますか?検算すればいいと思うのですが、複雑になって解けませんでした
もし間違ってたら解答を教えてください
101132人目の素数さん:2010/04/10(土) 19:32:38
102132人目の素数さん:2010/04/10(土) 19:32:59
100 名前:あぼ〜ん[レスあぼ〜ん] 投稿日:あぼ〜ん
103132人目の素数さん:2010/04/10(土) 19:37:19
同じ質問を連投か。「流れた」ってww
数日くらい待てないのかね。催促みたいで押し付けがましいわ。
104132人目の素数さん:2010/04/10(土) 19:51:39
このスレはキチガイ専用だから動でもエエわ
105132人目の素数さん:2010/04/10(土) 21:01:45
>>100
お前は y=log(x)の微分もできないのか?
106132人目の素数さん:2010/04/10(土) 21:11:04
>>100
y=log(e^x+e^(x^2)+c) は dy/dx=e^(x-y)+e^(x^2-y) の解だ。
107132人目の素数さん:2010/04/10(土) 21:27:38
108132人目の素数さん:2010/04/10(土) 21:29:27
109132人目の素数さん:2010/04/10(土) 21:38:07
644:作者の都合により名無しです :2010/04/10(土) 20:54:08 ID:NmRY+TVp0 [sage]
じゃあ、次の問題 >>640,>>642,>>643 の誰がやってもいい。ただし今日中にやってくれ。
f:X→Y,g:Y→Zで、f,g:単射のとき、gとfの合成関数g。fが単射であることを示せ。

少年漫画板で暴れられて鬱陶しいです。助けて下さい
110132人目の素数さん:2010/04/10(土) 21:43:11
>>106
なりました、一応確認ですがdy/dx=e^(x-y)+"2x"e^(x^2-y)ですよね?

dy/dx=e^(x+y)+2xe^(x^2+y)の解はy=log{-1/(e^2+e^(x^2)+c)}で合ってますでしょうか
111132人目の素数さん:2010/04/10(土) 21:49:47
>>110
合ってるけど、分母に式があるのは美しくない。

y=-log(-e^2-e^(x^2)+c)

と書いた方がいい。
112132人目の素数さん:2010/04/10(土) 22:05:24
>>111
ありがとうございました
113今年から大一:2010/04/10(土) 22:08:04
予備校時代の問題で今でもよくわからない問題がありまして
ずっと悩んでます。
ほぼ丸投げで申し訳ないのですが
先生方のお力をお借りできないでしょうか

-問題-
関数 f(x) = ||sin x-1/2|-1/2| について次の問いに答えよ。
ただし -π≦x≦π とする。

関数 y=f(x) のグラフを書きなさい。

追加:答えを見てもよくわかってないのですが
    sin x<1/2のときに f(x)=|sin x|になる理由も教えていただけたら
114132人目の素数さん:2010/04/10(土) 22:08:55
>>109
単射の定義をみたすことを確認するだけ。
115132人目の素数さん:2010/04/10(土) 22:10:09
>>113
|sin(x)|=|-sin(x)|
116132人目の素数さん:2010/04/10(土) 22:11:35
>>113
 sin(x) < 1/2 のとき |sin(x) - 1/2| = 1/2 - sin(x),
117132人目の素数さん:2010/04/10(土) 22:11:46
>>115
ありがとうございます
そうですね、すみませんつまらない内容で
本当にありがとうございます
118132人目の素数さん:2010/04/10(土) 22:17:41
>>114
大変残念なことに数学は数UBまでしかやってない文系ですので、さっぱり分からないんです。
119132人目の素数さん:2010/04/10(土) 22:20:56
>>118
単射の定義を知らないのか?
120132人目の素数さん:2010/04/10(土) 22:24:13
さっきwikiで初めて見ました
121132人目の素数さん:2010/04/10(土) 22:24:32
UBまでしかしてないって言ってるのに知ってるわけないだろ
122132人目の素数さん:2010/04/10(土) 22:31:38
スレ荒らし退治依頼なんかまともに相手してやるなよ
123132人目の素数さん:2010/04/10(土) 22:32:05
それは高校までの話だろ。
124熊猫 ◆ghclfYsc82 :2010/04/10(土) 22:39:39
>>122
スレ荒らしの退治やったらワシに任せなはれ。ワシがヨシナにしたるさかいナ。


125132人目の素数さん:2010/04/10(土) 22:42:38
ではさっそくお願いします
猫と名のつくコテを排除してくださいな
126132人目の素数さん:2010/04/10(土) 22:56:08
猫は馬鹿
127132人目の素数さん:2010/04/10(土) 22:59:07
数2の問題です

自然数 n に対して、an、bnを
(3+√2)^n = an+bn√2
によって定める。
nが奇数のとき、an、bnは共に奇数であって
nが偶数のとき、anは奇数で、bnは偶数であることを
数学的機能法によって示せ

解けるもんならといて見ろwwwwwww

スミマセン、お手引きお願いします。
本当に困ってます
128132人目の素数さん:2010/04/10(土) 23:11:54
>>127
テンプレの数列の表記読まなかったか?
あと質問するのに横柄すぎ。死ね。市ねでも氏ねでもなく死ね。
129132人目の素数さん:2010/04/10(土) 23:17:04
>>127
a_(n+1)+b_(n+1)√2
=(3+√2)^2(a_(n-1)+b_(n-1)√2)
=(11+6√2)(a_(n-1)+b_(n-1)√2)
=11a_(n-1)+12b_(n-1)+(6a_(n-1)+11b_(n-1))√2
より
a_(n+1)=11a_(n-1)+12b_(n-1)、b_(n+1)=6a_(n-1)+11b_(n-1)
そして
a_1=3、b_1=1、a_2=11、b_2=6だ。

あとは、帰納法の定石通りにすすめればよい。
偶数+奇数は奇数 偶数×奇数は偶数、奇数×奇数は奇数 などというあたりまえの関係を使えばよい。

130132人目の素数さん:2010/04/10(土) 23:18:19
>>100 >>110
 exp(-y) dy = {e^x + (2x)exp(x^2)}dx,
なので、変数分離形。積分すると
 -exp(-y) = e^x + exp(x^2) + c1,
でつね。
131132人目の素数さん:2010/04/10(土) 23:20:35
>>129
大変お手数をかけました
わかりやすいです
ありがとうございます
132132人目の素数さん:2010/04/10(土) 23:20:41
釣られが減らないのは何故だ?
133132人目の素数さん:2010/04/10(土) 23:44:43
曲面x^2+y^2=2*a*z (a>0) と平面z=xで囲まれた立体の体積の求め方を教えていただけませんか?
特に領域のとりかたなのですが、
0<=x<=2*a, x^2/2*a<=z<=x では間違いでしょうか?
それとも極座標によって解く方がよいのでしょうか?
ご指導よろしくお願いします
134132人目の素数さん:2010/04/10(土) 23:54:10
>>133
xy面上の積分にするのがいいんでは?
135132人目の素数さん:2010/04/11(日) 00:06:39
>>134
あ、円形になるのでしょうか?
やってみます!
136132人目の素数さん:2010/04/11(日) 00:14:29
>>134
xyで極座標を用いたらすんなり解けました!ありがとうございました!
137132人目の素数さん:2010/04/11(日) 00:23:44
>>132
連中の狙い通りに動いてやる義理はもちろんのこと
そうしない義理もないから
138日曜日は回答者 ◆Z6lIyUlGt2 :2010/04/11(日) 04:04:18
>>62
> an=a+εを考えることにより
の意図が不明なので、無視すると
lim a_n = +∞ より、任意のKに大してある自然数N'が存在して
n>N' ⇒ a_n > 2K
S = Σ_{n=1}^{N'} a_n とおく
S≧KN'ならばN=N'とする
そうでないなら、N > N' + (KN'-S)/K となる自然数を一つとってNとする
どちらにせよn>N ⇒ (Σ_{k=1}^n a_k)/n >K
139132人目の素数さん:2010/04/11(日) 15:36:50
0<=x+y<=1, 0<=x-y<=1
この二重積分の範囲の取り方がわからないです。
作図したいのですが、どのようにすればよいのでしょうか?
140132人目の素数さん:2010/04/11(日) 15:49:03
s=1/(t+1)dx
s=s^nx^n/n!=s0+dsx/1!+ddsx^2/2!+...
s=(t+1)^-1x-dt(t+1)^-2x^2/2!+...
141132人目の素数さん:2010/04/11(日) 15:53:18
合成函数のn階微分の公式ってなんですか?
142132人目の素数さん:2010/04/11(日) 15:56:41
Faà di Bruno's Formula
143132人目の素数さん:2010/04/11(日) 15:59:26
144132人目の素数さん:2010/04/11(日) 16:01:21
すべての導函数は被積分函数のテイラー級数であらわせる。

Sfdx=Σf^n-1x^n/n!
145132人目の素数さん:2010/04/11(日) 16:01:24
>>143
0のほうを忘れてる
146132人目の素数さん:2010/04/11(日) 16:05:27
0<=x+y<=1, 0<=x-y<=1
y=-x+s,0<s<1
y=x+s,0>s>-1
147132人目の素数さん:2010/04/11(日) 16:17:55
>>145
こうでしょうか…?
http://beebee2see.appspot.com/i/agpiZWViZWUyc2VlchQLEgxJbWFnZUFuZFRleHQYif1mDA.jpg

>>146
詳しく教えていただけませんか?
148132人目の素数さん:2010/04/11(日) 16:47:39
6m^2-37m+6m=0
を解け

どうしろと・・・
149132人目の素数さん:2010/04/11(日) 16:52:28
整頓してmでくくる
150132人目の素数さん:2010/04/11(日) 16:54:20
>>148です

間違えました

6m^2-37m+6= でした
151147:2010/04/11(日) 16:58:57
>>147は合ってないですね…全然わからないです…

>>150
解の公式を使ってはいけないのでしょうか?
152132人目の素数さん:2010/04/11(日) 17:02:35
0<=x+y<=1, 0<=x-y<=1
x+y>=0 , x+y<=1 , x-y>=0 , x-y<=1
y>=-x , y<=-x+1 , y<=x , y>=x-1

習ってるなら置換した方が楽だと思う
153132人目の素数さん:2010/04/11(日) 17:04:05
(6m-1)(m-6)
37=36+1に気がつかないのはマズい
154132人目の素数さん:2010/04/11(日) 17:09:01
6(m-1/6)(m-6)=0
m=1/6,6ですね

ありやした
155132人目の素数さん:2010/04/11(日) 17:11:51
>>152
置換とはどういうことでしょうか…
無知でスミマセン…
156132人目の素数さん:2010/04/11(日) 17:14:51
157132人目の素数さん:2010/04/11(日) 17:18:32
こういうタイプの積分は置換積分を使った方が楽ってこと

>>156
あってる
158132人目の素数さん:2010/04/11(日) 17:32:30
>>157
無事解けました!ありがとうございます。

D: 0<=x+y<=1, 0<=x-y<=1 の範囲でのxについての二重積分なのですが、どのように置換したらよいのでしょうか?
よろしかったら教えていただけませんか
159132人目の素数さん:2010/04/11(日) 17:35:17
>>146
あ、これでしょうか?
160132人目の素数さん:2010/04/11(日) 17:39:36
確率分かりません・・・
<問>
そのつど戻すという取り方で,1組のトランプ札から1枚ずつ札を取り出すものとする。
スペードが少なくとも1回得られる確率を0.9以上にするには,何枚の札を取りださねば
ならないか。

<解>
・テキスト:8枚
・自分  :9枚
 1-(0.75)^n ≧ 0.1
 n ≧ 8.004・・・
 よって9枚 
161132人目の素数さん:2010/04/11(日) 17:41:20
>>160 訂正
(誤)
 1-(0.75)^n ≧ 0.1
(正)
 1-(0.75)^n ≧ 0.9
162132人目の素数さん:2010/04/11(日) 19:37:10
k<3/2かつx≧-3かつx+k+2≧0を満たすkの値の範囲を求めよ。ただし、xとkはともに実数とする。
は計算不能ですか?

解けないんですが…
163132人目の素数さん:2010/04/11(日) 19:59:27
>>162
グラフを書いてみる
164132人目の素数さん:2010/04/11(日) 20:02:16
>>162 計算可能で、解もあるが?
165132人目の素数さん:2010/04/11(日) 21:45:30
>>162
いろいろやってみたけど解は出ないと思うよ。
問題は何だったの?

>>163
グラフ書いてできるんなら解けば?
もったいぶってるのかなんか知らないけど、解けないんなら解けないって言えばいいじゃん
166132人目の素数さん:2010/04/11(日) 22:27:08
>>165
アホ極まれり、か
167132人目の素数さん:2010/04/11(日) 23:00:27
k<3/2
x≧-3
x+k+2≧0
3/2>k>=2ーx<=5
168132人目の素数さん:2010/04/11(日) 23:07:09
ちょっと大学の友人間で出た話なんだが一つ
ソリティアで一枚めくりの時、初期配置で詰みになる確率を求めよ

何から手をつけたらいいのか分からん
169132人目の素数さん:2010/04/11(日) 23:18:12
>>167
k≧-x-2であり、x≧-3 から -x-2≦1
よって、-x-2≦k<3/2 を満たすkは存在する。つまり、kがk<3/2を満たすなら、x≧-k-2を満たすxをとると
点(x,k)は所与の不等式を満たす領域の点として得られる。
したがって、kの取り得る値は k<3/2  
170132人目の素数さん:2010/04/11(日) 23:38:11
y=x^2+ax+bは点(-1、-1)を通りy=2x-8に接している。この時のa、bの値を求めよ。
という問題の解き方がイマイチわかりません。最初の式に点の値を代入して1文字削除するのは正しいですか?
詳しく解説してください!
171132人目の素数さん:2010/04/12(月) 02:05:36
>>165
まれにみるばかだな

>>169
領域としてあらわしたあと
xを定義域として、xの値によってkの値域が変わることまで示すべきでは?
-x-2 ≦ k < 3/2 (x≧-3)
172132人目の素数さん:2010/04/12(月) 03:12:39
>>160

 1-(0.75)^8 =0.89988…なので
解答の方は
精度の悪い計算機でも使って0.9にしてしまったのかもしれないな
173132人目の素数さん:2010/04/12(月) 03:22:07
>>168
初期配置のせいで詰みになる、ということ?
やり方がまずくて詰みなのは無しで、
ありとあらゆるやり方をしても配置上絶対解法が存在しないというような?

1枚めくりということは、めくるカードは自由に使っていいというのとほぼ同じか。…(1)

確率を出すからには、全配置を同様に確からしいとして、それが何通りあるかをまず出すべきかな。
スートは色の情報のみあればいいので、赤と黒に単純化して良し
(1)より、めくるカードの順序は関係なし

あとは、明らかにそれ以上めくれない状況から取りかかれば少しずつ答えに近づけるんじゃないかな。
見えているカードおよびめくれるカードより大きい数字や、場から除去できるとっかかりのAが
全て今見えていない奥に隠れている場合など。

単純に数式化して一つ二つの式で解決するものではなく
数万通りくらいに場合分けしてそれぞれの配置の個性に応じてちゃんとクリアできるかどうかを判別していく必要があるんじゃないかと思う。
174132人目の素数さん:2010/04/12(月) 03:23:44
>スートは色の情報のみあればいいので
除去するときにはスートの区別が関係するな
175132人目の素数さん:2010/04/12(月) 16:02:39
まあ、
> 初期配置で詰みになる
がどういう意味なのかわからないと話にならないよなー
・1枚ずつカードをめくったが、全然動かせないまま終了した
・動かせることは動かせたが、常に選択の余地がなく、完成せずに終了した
・完成せずに終了したあと検証してみたところ、どう動かしたとしても失敗だったと分かった
一番上ならともかく、一番したの(>>173の言うような)場合は、はっきり言って手計算では無理じゃないか?
176132人目の素数さん:2010/04/12(月) 16:26:07
>>175
一番下だろうけど(当然、真ん中や上も包括される)

真ん中だとしても手計算は厳しそう
一番上の「全然動かせない」なら、逆にレアすぎて計算できそうだけど

全然動かせないというのが一枚めくりの方まで指してて
一枚めくりの分を開かれたカードの上に重ねることも、右上にAから順に収納していくこともできないとなると
そういう状態は存在するんだろうか
177132人目の素数さん:2010/04/12(月) 16:37:49
>>176
> 全然動かせないというのが一枚めくりの方まで指してて
> 一枚めくりの分を開かれたカードの上に重ねることも、右上にAから順に収納していくこともできないとなると
> そういう状態は存在するんだろうか
そりゃ存在することは存在するよ
極端な例だと、山札24枚は2~7で開かれたカードはKとJのみ、という状況
178132人目の素数さん:2010/04/12(月) 20:42:53
>>170
・y=x^2+ax+bが(-1,-1)を通ることから、(a,b)はこの式に(x,y)=(-1,-1)を代入した式を満たす。
・y=x^2+ax+bとy=2x-8が接することから、x^2+ax+b=2x-8は重解をもつ。すなわち(a,b)は(判別式)=0を満たす。
あとはこの連立方程式を解くだけ。
179132人目の素数さん:2010/04/12(月) 23:17:56
c/a+b+c/a-b


どうして2ac/a^2-b^2 になるのですか?
180132人目の素数さん:2010/04/12(月) 23:28:11
>>179


おい それじゃよくわからんぞ


c/a+b + c/a-b

なら
c(a-b)/(a+b)(a-b) + c(a+b)/(a-b)(a+b)
分母が通分できたから分子を計算して
2ac
分母を展開して
a^2-b^2


常識だぞ
181132人目の素数さん:2010/04/12(月) 23:35:22
>>179 ならん

テレパスすると分数表記が誤っている
182132人目の素数さん:2010/04/12(月) 23:36:58
>>180
おまえも分数表記が甘い

c(a-b)/((a+b)(a-b)) + c(a+b)/((a-b)(a+b))
183132人目の素数さん:2010/04/13(火) 00:37:00
>>182
過剰

あるいはブランクなどでの区別も紛らわしいとする態度なら
>>180
>c/a+b + c/a-b
も訂正すべき
184132人目の素数さん:2010/04/13(火) 00:44:11
>>183
> >>182
> 過剰

ほう、どこが?
185132人目の素数さん:2010/04/13(火) 07:23:26
>>183
過剰ではない。必須。
c(a-b)/(a+b)(a-b) + c(a+b)/(a-b)(a+b)だと
c(a-b)^2/(a+b) + c(a+b)^2/(a-b)の意味になる。
186132人目の素数さん:2010/04/13(火) 08:05:24
texに対応したらいいのにな
運営してる奴が文系の馬鹿だから仕方ないけど
規制ばかりかけるから書く気なくすし
誰か新しい掲示板つくらんかな
187132人目の素数さん:2010/04/13(火) 11:51:56
m≠-1 (mod p) なら

Σ[k=1,p-1] k^m ≡0 (mod p)

は常に成立しますか?
188132人目の素数さん:2010/04/13(火) 13:02:41
>>187
p=2, m=2 のときどう?
189132人目の素数さん:2010/04/13(火) 13:08:27
書き忘れました。pは奇素数です。p=7のときに京大入試にでたものの
一般化が成立するのかな?と思いまして。
190132人目の素数さん:2010/04/13(火) 13:41:38
p=3, m=4
191132人目の素数さん:2010/04/13(火) 13:49:09
TEXは書くのが大変
IMEは読むのが大変
192132人目の素数さん:2010/04/13(火) 18:39:13
通常の位相(ユークリッド空間R^2の開集合系)をもったR^2において

A={(x,y)∈R^2|xy=1} は閉集合であることを示せ
193168:2010/04/13(火) 19:09:14
生活の変化でしばらくレス見れんかったスマソ

>>175の下の例のつもりで書き込んでた。言葉が足りなかったな
自分文系で上の例すら計算出し方分からないからおまいらの知恵を借してくれ
194132人目の素数さん:2010/04/13(火) 20:17:34
2chで誤字につっこむのもなんだが、
文系で「借してくれ」はねーべ。
195132人目の素数さん:2010/04/13(火) 21:04:21
(AcUB)∩(AUBc) = (A∩B)U(Ac∩Bc)
こいつの証明よろしくおねがいします
196132人目の素数さん:2010/04/13(火) 21:26:59
四角形ABCDにおいて
∠ABD=a ∠DBC=b ∠ACB=c ∠DCA=d とおく
次の各場合について∠ADBの大きさを求めよ

(1)a=12°b=36°c=48°d=24°
(2)a=20°b=20°c=40°d=40°
(3)a=20°b=60°c=50°d=30°

お願いします
197132人目の素数さん:2010/04/13(火) 21:36:21
>>196
マルチ
198132人目の素数さん:2010/04/13(火) 21:44:20
>>195
分配法則でばらす。
199132人目の素数さん:2010/04/13(火) 22:36:42
>>189
mがp-1で割り切れない数なら成り立つ。
200132人目の素数さん:2010/04/13(火) 22:57:21
>>192
まず通常の位相での開集合とは何かをはっきりさせる。
201132人目の素数さん:2010/04/13(火) 23:16:02
>>198
(AcUB)ってBに置き換えることってできますか?
202132人目の素数さん:2010/04/13(火) 23:26:10
>>201
No
203132人目の素数さん:2010/04/14(水) 01:27:31
>>194
かえって誤変換することのほうが難易度が高いと思う。
どうやったんだろう
204132人目の素数さん:2010/04/14(水) 06:44:04
アホな質問ですが、\mathfrakの大文字のG,N,Oの
手書きは筆記体で書けばいいのですか?
小文字のa,g,hなんかは決まった書き方のようですが。
205雑談:2010/04/14(水) 07:30:56
>>203 この手のは
普通に変換して「候補が出ない!」ってとこまでは「ガイシュツ」と似ているが
そのあと
「たいしゃく」を変換して自分が思いこんでいるほうの漢字を選んだりしている
のではあるまいか
206204:2010/04/14(水) 14:34:06
あとアレフ(無限の濃度)もお願いします。
207132人目の素数さん:2010/04/15(木) 13:54:36
アレフだのマルチだの2ちゃんは物騒だな
208132人目の素数さん:2010/04/15(木) 20:20:56
>>192
問題修正
通常の位相をもったRの直積R^2において

A={(x,y)∈R^2|xy=1} の射影p_1(A)は閉集合ではないが(自明)、
Aは閉集合であることを示せ(積位相の定義から厳密に)
209132人目の素数さん:2010/04/15(木) 20:31:56
直積→積空間
210132人目の素数さん:2010/04/16(金) 23:47:45
>>208
普通にxy平面でのグラフを手掛かりに考えたら証明は自ずから浮かんでくると思う。
211132人目の素数さん:2010/04/17(土) 11:16:46
お願いします。
直線:(x-x0)/l =(y-y0)/m =(z-z0)/n 
と平面:ax+by+cz=dとが平行である条件を求めよ。
212132人目の素数さん:2010/04/17(土) 11:29:15
>>211
直線の方向ベクトル(l,m,n)と平面の法線ベクトル(a,b,c)が直行する。

加えて、直線が平面に含まれないこと。
213212:2010/04/17(土) 11:30:13
typo orz

×直行する
○直交する
214132人目の素数さん:2010/04/17(土) 12:15:37
>>211
問題州の答えでは、x=x0+lt、y=y0+mt、z=z0+ntをax+by+cz=dに代入し、
(al+bm+cn)t=d-(ax0+by0+cz0)なので、条件は、al+bm+cn=0だと出ているんですが、
これはどういう意味なんでしょうか?
215132人目の素数さん:2010/04/17(土) 12:48:03
>>214
al+bm+cn≠0 ならば (al+bm+cn)t=d-(ax0+by0+cz0) を満たす t がある。
つまりその直線と平面は交わるということ。
216132人目の素数さん:2010/04/17(土) 13:12:01
>>214 
わかりました!!ありがとうございます!!!
217132人目の素数さん:2010/04/17(土) 13:46:44
無数にあるってのは?
218132人目の素数さん:2010/04/17(土) 15:30:58
直線:(x-x0)/l =(y-y0)/m =(z-z0)/n 
と平面:ax+by+cz=dとが平行である条件を求めよ。

交わらない
(x-x0)/l =(y-y0)/m =(z-z0)/n=t
x=lt+x0,y=mt+y0,z=nt+z0
alt+ax0+bmt+by0+cnt+cz0=d
t=(d-ax0-by0-cz0)/(al+bm+cn)
が存在しないー>al+bm+cn=0
219132人目の素数さん:2010/04/17(土) 15:48:47
al+bm+cn=iでもいい
220132人目の素数さん:2010/04/17(土) 18:27:52
(剰余演算)式x=y(mod5)を、x、yを5で割ったあまりが等しいと読む。
次を満たす x、yE{0,1,2,3,4}を求めよ。

(1)2x+3=1(mod5)

(2) 3x−2y=4
   2x+3y=2 (mod5) ←連立です
221132人目の素数さん:2010/04/17(土) 18:29:44
理解ができなくて困っています。
教えてくださいお願いします。
222132人目の素数さん:2010/04/17(土) 18:48:59
普通に方程式を解くのとほとんど同じ。
方程式に対してやっていい操作に加えて「片方の辺に5を足す」
という操作をすることができるので、それによってx,y∈{0,1,2,3,4}にする。
223132人目の素数さん:2010/04/17(土) 18:51:58
分かんないなら1個1個入れてもいいじゃん。
224132人目の素数さん:2010/04/17(土) 19:01:41
4つの数字を、+−×÷で10を作るやつ
2 2 8 9 が解けないから解いといて

例6789
(9−7)×8−6
225132人目の素数さん:2010/04/17(土) 19:08:14
2x+3=1(mod5)
01234
30241
(2) 3x−2y=4
   2x+3y=2 (mod5)
01234
41302
3-401
20314
-0123
226132人目の素数さん:2010/04/17(土) 19:14:05
ありがとうございます。
実は、私も1つ1つ代入してやったんですが、これで良いんですか?
227132人目の素数さん:2010/04/17(土) 19:17:06
(-8-(+7)=-15となる計算を「なんで-15になるの?-1じゃないの?」って聞かれたらうまく説明できなかったorz
228132人目の素数さん:2010/04/17(土) 19:17:17
>>224
(9-8/2)*2
というか検索ぐらいしろよ。
229132人目の素数さん:2010/04/17(土) 19:26:55
3x-2y=4
2x+3y=2
y=1
3x=1
2x=4
x=2
01234
41302
3-101
20314
-0123
230132人目の素数さん:2010/04/17(土) 19:30:03
連立はそのままやったほうがいいかも。別解もあるから。
231132人目の素数さん:2010/04/17(土) 19:40:14
>>230
別解も教えてください
232132人目の素数さん:2010/04/17(土) 19:44:15
(3x-2y-4)(2x+3y-2)=0
x^2-y^2-1-3x=0
01234
01441
02413
42243
04114
1--1-
4--4-
x=0,4,0,4
y=1,4,4,1
233132人目の素数さん:2010/04/17(土) 20:10:26
(3x-2y-4)(2x+3y-2)=0
6x^2+9xy-6x-4xy-6y^2+4y-8x-12y+8
x^2+x-y^2+2y+3=0
01234
01441
20402
04114
01022
30--3
42--4
x=0,4,0,4,1,1
y=3,4,4,3,0,2
234132人目の素数さん:2010/04/17(土) 20:32:07
1,2だけです
235132人目の素数さん:2010/04/17(土) 21:01:46
3x-2y=4
2x+3y=2 mod 5

3x-2y=1 x=1+2m,y=1+3m
3x-2y=4 x=4(1+2m),y=4(1+3m),x=2,y=1
2x+3y=1 x=-1+3n,y=1-2n
2x+3y=2 x=2(-1+3n),y=2(1-2n),x=2,y=1

x=4+3m=3+n->3m-n=4
y=4+2m=2+n->2m-n=3
01234m
41302
14203n
31420
24130n
m=1,n=4
236132人目の素数さん:2010/04/18(日) 01:12:13
わかりません。お願いします。

三角形ABCにおいて、BCに平行な直線がAB,ACと交わる点をE,Fとする。
BF、CEの交わる点をGとするとき、AGはBCの中点を通ることを示せ。
237132人目の素数さん:2010/04/18(日) 03:02:57
R上の関数f(x):=(x^2-2x+1)/(x^2-1)について、任意のε>0が与えられたとき、
|x-1|<δ⇒|f(x)|<εが成り立つようにδ>0を定めなさい。

δ=min(3ε,1).と思ったんですが、これを用いての証明ができません。
238132人目の素数さん:2010/04/18(日) 03:08:42
>>237
不等号を記号で変換するのを忘れてしまいました。
すいません。
239132人目の素数さん:2010/04/18(日) 03:20:58
>>237,238
約分してから分子はδで上から評価。
分母は三角不等式を使って下から評価。
δ=min(1,ε) くらいに取れる。
240132人目の素数さん:2010/04/18(日) 05:55:35
(x^2-2x+1)/(x^2-1)<e
(x-1)/(x+1)<e
(1-e)x<e+1
x<(e+1)/(1-e)=δ
241132人目の素数さん:2010/04/18(日) 07:08:56
|x-1|=δ<|(e+1)/(1-e)-1|
242132人目の素数さん:2010/04/18(日) 11:01:32
AGはBCの中点を通ることを示せ。

紙で折ってみれば?
243132人目の素数さん:2010/04/18(日) 11:04:09
最近幼稚園の入園試験問題を聞くのが流行っているようだ。
244132人目の素数さん:2010/04/18(日) 11:06:57
折り紙禁止くさい問題なのに折り紙して解いたとかいってるやつくさい
245132人目の素数さん:2010/04/18(日) 11:15:53
分かりませんお願いします
sin^2θ+cos^2θ=1を証明せよ
246132人目の素数さん:2010/04/18(日) 11:17:47
定義からすぐに出る
教科書に載ってるはずだから見てみるといいよ
247132人目の素数さん:2010/04/18(日) 16:46:45
>>245
しばくぞ
248132人目の素数さん:2010/04/18(日) 16:47:06
ζ(s)関数の自明でない零点sは、全て実部が1/2の直線上に存在する
249132人目の素数さん:2010/04/18(日) 16:49:45
>>248
しばくぞ
250猫は雑魚 ◆ghclfYsc82 :2010/04/18(日) 16:57:13
>>248
定義からすぐに出ません
教科書に載ってないはずだから見ても無駄です

注釈:貴方が証明を見付けたらお金が儲かります。


251132人目の素数さん:2010/04/18(日) 17:01:38
y=x^x(x<0)をxで微分するとどうなるんでしょうか?
ググったところx<0ではxが整数のときのみyの値が出ると聞きました。
ということは不連続なので微分不可能ということで良いのでしょうか?
教えてください。
252132人目の素数さん:2010/04/18(日) 17:47:48
p=x^4-2x^3-3x^2-5x-7 x=2-√3 pの値をもとめよ。
この問題を解説つきで教えてください
253132人目の素数さん:2010/04/18(日) 17:56:08
>>252
このxは2次方程式 f(x)=x^2-4x+1=0 を満たす。
pをf(x)で割った余りr(x)をもとめ、r(2-√3)を計算する。
254132人目の素数さん:2010/04/18(日) 18:13:52
>>253
f(x)=0なのに割っていいの?0で割るのってアウトじゃ・・・
255132人目の素数さん:2010/04/18(日) 18:45:30
言い換えよう
Xの4次多項式 X^4-2X^3-3X^2-5X-7 を Xの2次多項式f(X)=X^2-4X+1 で割った余りr(X)を求めて 
r(2-√3)を計算する。
256132人目の素数さん:2010/04/18(日) 18:59:38
x=2-√3pという問題か?と思ったらx=2-√3だったのか
257132人目の素数さん:2010/04/18(日) 19:11:33
x^3-3x-11
----------------------
x-2
x^4-2x^3-3x^2-5x-7
x^4-2x^3
----------------------
-3x^2-5x-7
-3x^3+6x
------------
-11x-7
-11x+22
------------
-29
p=(x-2)(x^3-3x-11)-29
=(x-2)((x-2)(x^2+3x+1)-9)-39
=(x-2)((x-2)((x-2)((x-2)+18)-9)-39
=√3((√3((√3(√3+18)-9)-39

x^2+2x+1
---------
x^3-3x-11
x^3-2x^2
-----------
2x^2-3x-11
2x^2-4x
----------
x-11
x-2
----------
-9
258132人目の素数さん:2010/04/18(日) 19:33:35
この問題どなたか解答書いていただけませんか?お願い致します。
((x^2)+1)y'=xyをみたす一般解を求めよ。
また解のうちy(0)=1となるようなものを求めよ。
259132人目の素数さん:2010/04/18(日) 20:58:16
>>258
両辺y(x^2+1)で割ってxで積分
260132人目の素数さん:2010/04/19(月) 05:30:10
>>236 チェバよりAGとBCの交点をMとするとAE*BM*CF=EB*MC*FA EFはBCに平行よりAE*CF=EB*FAよりBM=MC
261132人目の素数さん:2010/04/19(月) 14:59:50
どなたかお願いします。集合論の問題です
A{Xは集合|Xの要素は1つ}
Aは集合でないことを証明せよ。
262132人目の素数さん:2010/04/19(月) 17:28:10
確率200分の1で当たるスロットマシーンが、2000回まわしても当たらない確率は?
コレってどうやって計算するのですか?
式を教えてください。
263132人目の素数さん:2010/04/19(月) 17:46:03
(1-1/200)^2000
264132人目の素数さん:2010/04/19(月) 17:55:30
>>261
Aが集合だと仮定する。
すると∪A(つまり、Aの要素すべての和集合)も集合。
xが集合のとき、{x}は要素一つの集合だから、{x}∈A。したがってx∈∪A。
つまり∪Aは全ての集合を含む集合になる。
ラッセルのパラドックスから、これは不合理。
265132人目の素数さん:2010/04/19(月) 17:59:34
>>254
0 で割ってはいけませんよねえ
でも割り算の意味がちがうんよ
ちがうのに同じ言葉使うなんて
まぎらわしいよね
266132人目の素数さん:2010/04/19(月) 20:39:28
>>264 ありがとうございます 助かりました

267132人目の素数さん:2010/04/19(月) 21:11:43
是非とも皆さんの力を貸してください。

数学的帰納法を使った証明問題で、特に後半の因数分解(?)の計算方法を詳しく知りたいです。

全ての自然数nに対して、
1^3+3^3+5^3+ +(2n-1)^3=n^2(2^2-1)


kを任意の自然数とするn≧3の自然数に対して、
(1+k)^n>1+nk


よろしくお願いします!
268132人目の素数さん:2010/04/19(月) 21:16:07
どなたかお願い致します。中学生の問題です。

ある学校の男子の人数は全生徒数の七分の四より36人少なく、女子の人数は
全校生徒数の九分の四より25人多い。全校生徒数は何人か?


もうひとつ。
A君、B君、C君の3人がある枚数の折り紙で鶴を折る。全部折るのにA君とB君の
2人で折ると45分、B君とC君の2人で折ると90分、B君だけで折ると
120分かかる。この時、A君、B君、C君の3人で折り紙を折った場合、何分かかるか?


以上です。
269132人目の素数さん:2010/04/19(月) 21:33:13
>>268

全校生徒をXとおきます。
男子は 4/7*X
女子は 4/9*X  となり、

4/7*X+4/9*X=X 
という方程式を作れます。

これを解くと X=693人となります。



ABCの1分あたりに折れる枚数をそれぞれabcとおきます。

AとBは1分あたり、a+b=X/45 @ と表せます。
同様に、
b+c=X/120 A
b=x/120 B となります。

@+A-B=(1分あたりのABCの折れる枚数)=X/40

よって40分となります。



間違っているかもしれませんが、いかがでしょうか?

270132人目の素数さん:2010/04/19(月) 21:39:43
>>267
> 1^3+3^3+5^3+ +(2n-1)^3=n^2(2^2-1)
n=1 のとき 左辺=1, 右辺=3 ?
271132人目の素数さん:2010/04/19(月) 21:50:30
>>267
> 全ての自然数nに対して、
> 1^3+3^3+5^3+ +(2n-1)^3=n^2(2^2-1)

1^3+3^3+5^3+ +(2n-1)^3=n^2(2n^2-1) だな。

1^3+3^3+5^3+ +(2n-1)^3+(2n+1)^3=n^2(2n^2-1)+(2n+1)^3
この右辺が (n+1)^2(2(n+1)^2-1) になることを確認する。

> kを任意の自然数とするn≧3の自然数に対して、
> (1+k)^n>1+nk
(1+k)^(n+1)=(1+k)^n(1+k)>(1+nk)(1+k)=1+nk+k+nk^2
この右辺をじっくり眺める。

272132人目の素数さん:2010/04/19(月) 21:52:36
>>270

ご指摘ありがとうございます!
1^3+3^3+5^3+ +(2n-1)^3=n^2(2n^2-1)

でした!

申し訳ありませんが、よろしくお願いします。
273132人目の素数さん:2010/04/19(月) 22:03:39
>>269
ウソつくな。両辺をXで割ってみ
4/7*X+4/9*X=X
274132人目の素数さん:2010/04/19(月) 22:05:26
小中学生スレでやれ
275132人目の素数さん:2010/04/19(月) 22:09:15
>>271
ありがとうございます!

大変申し訳ありませんが、
n^2(2n^2-1)+(2n+1)^3=(n+1)^2(2(n+1)^2-1)
となる、計算の過程をもう少し詳しく教えていただけませんか?><

また、
(1+k)^(n+1)
この計算の方法も併せてご指導願いたいです。

よろしくお願いします!
276132人目の素数さん:2010/04/19(月) 22:11:18
>>273
すいません。
書き込み方が悪かったです。

(4/7)*X+(4/9)*X=X

こういう意味でした・・・
277132人目の素数さん:2010/04/19(月) 22:25:26
>>276
それを解いたらX=0
278132人目の素数さん:2010/04/19(月) 22:28:42
>>276
黙ってその両辺をXで割ってみろや
279132人目の素数さん:2010/04/19(月) 22:28:53
うわ。完全にすっとばしてました。

(4/7)*X-36+(4/9)*X+25=X


こうやな!

ほんまにすいません。
280132人目の素数さん:2010/04/19(月) 22:33:29
>>275
> n^2(2n^2-1)+(2n+1)^3=(n+1)^2(2(n+1)^2-1) ・・・(1)
> となる、計算の過程をもう少し詳しく教えていただけませんか?><

帰納法を使うこの手の問題では、結論の式の形が予想できる(与えられている)のだから、
どんどん計算するだけ。
(1)の式の右辺、左辺、展開して等しいことを確認すればよい。
1分もあればできるはず。


> また、
> (1+k)^(n+1)
> この計算の方法も併せてご指導願いたいです。

(1+k)^(n+1)=(1+k)^n(1+k)>(1+nk)(1+k)=1+nk+k+nk^2

を見てないの?
281132人目の素数さん:2010/04/19(月) 22:42:35
>>269
何度もありがとうございました。
282132人目の素数さん:2010/04/19(月) 22:51:46
>>280
>帰納法を使うこの手の問題では、結論の式の形が予想できる(与えられている)のだから、
>どんどん計算するだけ。
>(1)の式の右辺、左辺、展開して等しいことを確認すればよい。
>1分もあればできるはず。

解決しました!その発想はなかったです><


>(1+k)^(n+1)=(1+k)^n(1+k)>(1+nk)(1+k)=1+nk+k+nk^2
>を見てないの?

文字の○乗の計算方法がわからないです・・・
283132人目の素数さん:2010/04/19(月) 22:56:50
>>282
本質的に分配法則以外は使わん。
284132人目の素数さん:2010/04/19(月) 22:57:28
>>282

> >(1+k)^(n+1)=(1+k)^n(1+k)>(1+nk)(1+k)=1+nk+k+nk^2
> >を見てないの?
>
> 文字の○乗の計算方法がわからないです・・・
そんなのを使わない(2項展開を使っても解けるけど、今は帰納法)

(1+k)^n > 1+nk が帰納法の仮定だ。
これの両辺に(1+k)をかけて
(1+k)^n(1+k)>(1+nk)(1+k)=1+nk+k+nk^2=1+(n+1)k+nk^2>1+(n+1)k
285132人目の素数さん:2010/04/19(月) 23:00:53
5^m+8と5^n+3の最大公約数を求めよという問題です。
ユークリッドの互助法を使うようなのですが、どうすればいいのでしょうか?
286132人目の素数さん:2010/04/19(月) 23:04:03
>>285
ユークリッドの互除法を使えばよい
287sage:2010/04/19(月) 23:08:44
>>267
k^2(2k^2-1)+(2k+1)^3のすべてのkをすべてk+1にすれば最初は複雑だけど
簡単に(k+1)^2(2k^2+1)になるよ


あとふたつめはベルヌーイの不等式でも示せるよ

288132人目の素数さん:2010/04/19(月) 23:17:00
289132人目の素数さん:2010/04/19(月) 23:17:42
>>283
>>284
何度もありがとうございます。
理解力が乏しくて申し訳ありません。

>(1+k)^n > 1+nk が帰納法の仮定だ。
>これの両辺に(1+k)をかけて
これの左辺は、数列でいうと次の目盛りに移動した(2→3のように)ということでよろしいでしょうか?

その考え方であってたら、
(1+k)^n(1+k)>(1+nk)(1+k)=1+nk+k+nk^2=1+(n+1)k+nk^2
までは理解できました。

後ろに>1+(n+1)k がつくのは帰納法だからでしょうか?><
290132人目の素数さん:2010/04/19(月) 23:26:35
>>283
>>284

解けました!
ポイントを押さえた解説本当にありがとうございます!

もしかしたらまた助けを求めに来るかもしれませんが、その際はどうかよろしくお願いします!
291132人目の素数さん:2010/04/19(月) 23:27:40
>>286
(5^m+8) = (5^n+3) * 5^(m/n) + (-3*5^(m/n)+8)
(5^n+3) = (-3*5^(m/n)+8) * 5^n/8 + (3/8*5^m+3)
(-3*5^(m/n)+8) = (3/8*5^m+3) * (-8*5^(1/n)) + (-24*5^(1/n)+8)
(3/8*5^m+3) = (-24*5^(1/n)+8) * (-1/64*5^nm) + (-1/8 * 5^nm + 3)

どうすれば・・・・。
292132人目の素数さん:2010/04/19(月) 23:28:40
CANADAの6文字を並べ替えて単語を作る。
作った全ての単語を辞書式に並べていくとき、
CANADAは何番目にありますか?

詰んでしまいました。
293132人目の素数さん:2010/04/19(月) 23:39:09
n→∞のときの極限を求めよ。[]はガウス記号
sin2π{√(n^2+[n/3])}
よろしくお願いします。
294132人目の素数さん:2010/04/19(月) 23:44:42
>>292
74?
295132人目の素数さん:2010/04/19(月) 23:47:22
>>292
71だった
296132人目の素数さん:2010/04/20(火) 00:38:00
71に至るまでの過程もよろしくお願いします。
297132人目の素数さん:2010/04/20(火) 00:52:22
>>296
お前、何考えてンの。
お前の考えが正しいかどうかを確認するのが71と>>295さんが示してくれた数字だろ。
まず、お前の考えをかいてみな。
まず、CANより前にある文字の並びはなんだ?
298132人目の素数さん:2010/04/20(火) 01:01:04
親切な人だ
299132人目の素数さん:2010/04/20(火) 01:02:18
300132人目の素数さん:2010/04/20(火) 01:19:06
【コイントス投資法】
@「表」が出たらL、「裏」がでたらS
Aリミット30、ストップ30
B枚数は証拠金10万円につき2枚まで
C勝ったら次は倍の枚数でエントリー
D負けても2連勝してもBに戻る

解説頼みます
301132人目の素数さん:2010/04/20(火) 03:15:01
次の関数の導関数(dY/dx)を求めよ。
 
 X=Y^3-3Y

解説していただけたら助かります。
302132人目の素数さん:2010/04/20(火) 03:30:14
>>301
1=(3Y^2-3)dY/dx
303132人目の素数さん:2010/04/20(火) 06:39:03
CANADA
AX=5!/2!=60
CADX=3!/2!=3
CANAAD=1
60+3+1=64
64+1=65
304132人目の素数さん:2010/04/20(火) 08:40:43
まとめスレでみかけた問題なのですが、すっきりしないので教えて下さい。

ここにA・B・Cの3人の男がいる
3人の男達は拳銃で決闘を行う事になった
その決闘のルールは次の通り
・まずAから撃つ事ができる
・その後、B、Cと順番に拳銃を撃つ
・Cが終了後は、再びAに戻る
・誰か1人が残るまでA→B→C→A…の順番で永遠に決闘は続く
・途中、誰かが死んだ場合は、その死んだ者を飛ばして順番通りに続ける

3名の命中率は次の通り
A…30%
B…80%
C…100%
命中した場合は必ず死亡する

生存率が一番高くなる為にAが最初に取るべき行動とは何?
305132人目の素数さん:2010/04/20(火) 08:48:07
Aが最初に狙うに合理的な相手は当然100%を誇るCになる
Bを狙い、偶然命中すれば次はCの順番になり100%、Aは倒される
しかしCを狙い倒せたとしても次のBに80%の確率でAは倒される
Aが最初の射撃で外したとする
次にBは必然的にCを狙う
なぜならAを倒せば次のCに100%倒されるから
そのBが狙ったCへの銃弾が当たれば次はAの番
30%の確率でBを倒せる
BのCへの銃弾が外れた場合は次はCの順番
Bが狙った動機と同じ理屈で必然的にCはBを狙う
命中率は100%なのでBは倒れ順番はAに
Aは30%の確率でCを倒せる

ゆえにAが最初に狙う相手・結果によるAの生存率は、
・Bを狙い命中…0%
・Bを狙い外れ…30%
・Cを狙い命中…20%
・Cを狙い外れ…30%

つまりAの最初の射撃は外れた方が一番生存率を上げる結果となる
だから正解は、Aは最初は誰も狙わず空をめがけて撃つ(誰も狙わず、わざと外す)


というものなんですが、正解の行動をとるべきなのは理解できました。
ただ、各々の確率がこれでいいのかピンと来ません。
自分が2割・相手が8割の命中率で順に撃ちあった際の生存率はどう計算すればいいですか?
306132人目の素数さん:2010/04/20(火) 09:12:57
>>305
行動は正しいが確率はおかしい。
自分の命中率がa、相手がbで、先攻が自分のときに
生存率をpとすると、 p = a + (1-a)*(1-b)*p となる。
なので、p = a/{a+b-ab}。

a=0.2, b=0.8 なら p=5/21。
307132人目の素数さん:2010/04/20(火) 09:46:15
0.2/{0.2+0.8-0.16}=0.2/0.84=1/42…?
308132人目の素数さん:2010/04/20(火) 10:50:07
あばうと2.5
309132人目の素数さん:2010/04/20(火) 11:13:02
2.6ダタ
310132人目の素数さん:2010/04/20(火) 11:20:02
1. ∫[∞,0]( xe^(-ax) )dx
2. ∫[∞,-∞]( e^(-x^2) )dx
3. ∫[∞,-∞]( xe^(-x^2) )dx

ただし、a>0とする

よろしくお願いします
311132人目の素数さん:2010/04/20(火) 11:24:29
1. -a^(-2)
2. -√π
3. 0
312310:2010/04/20(火) 12:08:57
>>311
ありがとうございます。
申し訳ありませんが、解答の過程を示して頂いてもよろしいでしょうか?
参考にさせて頂きたいので‥
お手数おかけしてすいません。
313132人目の素数さん:2010/04/20(火) 12:25:52
じゃんけんで出る確率がグー1/2,チョキ1/4, パー 1/4の人とたくさんじゃんけんを
するときに勝率を最も大きくするにはグー,チョキ, パー を出す確率をいくつにするといいか?
314132人目の素数さん:2010/04/20(火) 13:04:20
>>313
相手が必ずその確率で選択するとわかっているなら、
パー100%が最良に決まってる。
315132人目の素数さん:2010/04/20(火) 13:44:16
>>314
グー 1/3+2ε,
チョキ 1/3 -ε
パー 1/3-ε
(0<ε< 1/3 ) でもパー100%になるか?違う気がするけど。。
316132人目の素数さん:2010/04/20(火) 14:59:44
>>315
100%パー
パ:a チ:b グ:c の割合でじゃんけんをしたとき、勝つ確率は a/2 + b/4 + c/4
これが最大になるのは a = 1 b = c = 0
317132人目の素数さん:2010/04/20(火) 15:01:30
>>315
ちがわねーよ。
何かおかしいと感じるなら、
それは相手がどういう確率で出すかを知っているという設定が不自然なだけ。
それを知らないから、ゲーム理論では双方とも全部等確率という戦略を選ぶことになる。
318132人目の素数さん:2010/04/20(火) 16:03:18
X:可分距離空間
X:ANR⇒X:locally contractible
を示せ。
いろいろ試したのですが上手くいきませんでした。よろしくお願いします。
319 ◆27Tn7FHaVY :2010/04/20(火) 18:45:00
p+q+r=1; max{(1/2)(r-q)+(1/4)(p-r)+(1/4)(q-r)}
ぐらいはやるんじゃないの

面同意
320132人目の素数さん:2010/04/20(火) 21:40:03
パ:a チ:b グ:c の割合でじゃんけんをしたとき、
勝つ確率(ただし引き分けは0.5勝とする)は
a/2 + b/4 + c/4 + (a/4+b/4+c/2)*(1/2)=5a/8 +3b/8 + c/2
が最大になるのはどうなるんだ?
321132人目の素数さん:2010/04/20(火) 21:43:45
限定じゃんけん
322132人目の素数さん:2010/04/20(火) 23:00:23
>>303
> CANADA
> AX=5!/2!=60

 CAA

> CADX=3!/2!=3
> CANAAD=1
> 60+3+1=64
> 64+1=65
>
323132人目の素数さん:2010/04/20(火) 23:14:02
sin(acos(x))=√(1-x^2)
の証明の仕方を教えて下さい
324132人目の素数さん:2010/04/20(火) 23:16:51
>>323
acos(x)=θとおく。
325132人目の素数さん:2010/04/20(火) 23:20:04
arccos(x)=y
cos(y)=x
[sin(y)]^2=1-[cos(y)]^2
326132人目の素数さん:2010/04/20(火) 23:31:23
>>310

1. ∫ x・e^(-ax) dx = x・(-1/a)e^(-ax) + (1/a)∫e^(-ax) dx
          = x・(-1/a)e^(-ax) - (1/a^2)e^(-ax) + c,

3. ∫ x・e^(-x^2) dx = -(1/2)・e^(-x^2) + c'
327132人目の素数さん:2010/04/20(火) 23:46:23
>>326
3. わざわざ計算するのか
328132人目の素数さん:2010/04/20(火) 23:50:44
>>324,325
御丁寧な回答ありがとうございました。
329132人目の素数さん:2010/04/21(水) 00:04:20
>>310

上端と下端が逆じゃないか?
330132人目の素数さん:2010/04/21(水) 00:27:33
>>320
それだって(a,b,c)=(1,0,0)が最大だろ。
a+b+c=1
X=(5/8)a+(3/8)b+(1/2)c=(3/8)+(1/4)a+(1/8)c=(3/8)+(1/8)(a+c)+(1/8)a
a+c≦1、a≦1なので、
X≦(3/8)+(1/8)+(1/8)=5/8で、
等号はa=1,c=0のとき成立
331132人目の素数さん:2010/04/21(水) 00:33:53
最大化すべき関数が
グー、チョキ、パーの割合の一次式で書けるってだけで
グーだらけかチョキだらけかパーだらけの
少なくともどれかひとつは関数を最大にする
332132人目の素数さん:2010/04/21(水) 00:59:44
領域 x+y+z=1 , 0≦x,y,z,≦1
を通る法線ベクトルが(a,b,c)の平面で原点からの距離を考えると
端の点のどれかを通るときに最大になる。
333132人目の素数さん:2010/04/21(水) 06:20:42
CAA =3!=6
65+6=71
334132人目の素数さん:2010/04/21(水) 06:43:46
d=(x,y,1-x-y)*(a,b,c)/(a^2+b^2+c^2)^.5
(x,y,1-x-y)=(a,b,c)s
x=as=a/(a+b+c)
y=bs=b/(a+b+c)
1-as-bs=cs
s=1/(a+b+c)
d=(a^2+b^2+c^2)^.5/(a+b+c)
d=
335132人目の素数さん:2010/04/21(水) 10:33:56
なんか、√のことを^.5と書く変な奴があちこちに出没してるな
336132人目の素数さん:2010/04/21(水) 10:54:46
せめて^(1/2)にしろと
337132人目の素数さん:2010/04/21(水) 10:59:31
>>335
別におかしな表記ではないし、以前から見られるが
338132人目の素数さん:2010/04/21(水) 11:04:33
>>335
いや、ここで主張しているのは、
「変な奴があちこちに出没していて、そいつの特徴は『√のことを^.5と書く』というものである」
ということで、√のことを^.5と書くこと自体が変という意図はなかった。すまん。
339132人目の素数さん:2010/04/21(水) 12:44:23
いや十分変だろ
340132人目の素数さん:2010/04/21(水) 13:57:58
>>338
承知した。
^.5はマルチバイトの使えない英語圏ではよく見かける表記だし。それはそれ。
341132人目の素数さん:2010/04/21(水) 15:11:13
平方根と指数関数を本当に区別出来てるなら^.5と書いたほうがいいってことは当たり前なんだけど。
ここで愚痴ってる奴は教科書偏重主義の頭でっかちだから見たことない数式だとすぐ拒否反応が出るんだろうな。
いつ見てもこのスレの住人はキモヲタニートのようだ。
342132人目の素数さん:2010/04/21(水) 15:15:03
> 平方根と指数関数を本当に区別出来てるなら
場合に応じて使い分けるんでは?
343132人目の素数さん:2010/04/21(水) 15:53:01
ずいぶん前からなので、いまさら「あちこちに出没」とか言われてもなー

式変形しか書かなくて会話する意思がなさそうなので、話しかけた事も無い
話しかけたら背景を説明してくれるのかな?
344132人目の素数さん:2010/04/21(水) 15:58:07
興味ないから話しかけることもないね。

道を歩いてて、犬のフン見つけても話しかけないだろ?
345132人目の素数さん:2010/04/21(水) 16:04:21
君には説明してもらえないだろうし、教科書偏重主義のままじゃ何を聞いても理解できないだろうね。
日本なら平方根と指数関数を特別区別しなくても、自分の論理的文章でどう違うか説明できなくても2流大学程度の数学科修士は普通には卒業出来るから。いいんじゃないか。
346132人目の素数さん:2010/04/21(水) 16:11:24
勝手に教科書偏重主義とか決めつけて、何がしたいんだ
自分以外の書き込みが全部同一人物に見える病気か何かですか?
347132人目の素数さん:2010/04/21(水) 16:42:48
数ヲタニート死ね
348132人目の素数さん:2010/04/21(水) 16:46:31
>>344
いくら教養を高めてもそういうたとえしか出来ないところが君の知能レベルの限界ってことだろうなw
349132人目の素数さん:2010/04/21(水) 16:53:22
このスレ二人しかいないと思ってんのかね。このオッサン。
350132人目の素数さん:2010/04/21(水) 16:57:44
>>344
意味不明
351132人目の素数さん:2010/04/21(水) 17:02:27
数ヲタニート死ね
352132人目の素数さん:2010/04/21(水) 17:12:07
>>336
^(1/2)は認めるけど^.5は認めないってこと?
なんか恥ずかしいなぁと感じない?
353132人目の素数さん:2010/04/21(水) 17:15:01
^.5 って表記を知らなかったんだろ
いまさらそんなこと言って何がしたいの?
354132人目の素数さん:2010/04/21(水) 17:25:08
純粋に数学の議論としても^.5の表記は非常に重要だけど、物理や工学の数値計算と同じように考えてるからそういうところが見えないんだろうな。
数学の教科書ではそういう議論はしないし、もし注意があってもコラム程度の扱いで証明すべき定理や公式でもないから、^.5や^(1/2)などを数学概念として教科書で教えるようなことはない。
教科書偏重主義というのは、自分の理解に自信がないから教科書のフレーズを何年経ってもオウム返しするしか能がない奴のことだと思うよ。
355132人目の素数さん:2010/04/21(水) 17:28:51
わかりました。
わかりましたので、もう出てこないでください。
ありがとうございました。
356132人目の素数さん:2010/04/21(水) 17:31:36
>>355
> わかりましたので、もう出てこないでください。

行の前半と後半に関連が見出せないんだが。
理由の説明になってない。
357132人目の素数さん:2010/04/21(水) 17:33:44
数ヲタニート死ね
358132人目の素数さん:2010/04/21(水) 17:35:38
>>344
今日は一体誰と戦っているんですか?
359132人目の素数さん:2010/04/21(水) 22:47:45
r=x+y+z Cを定ベクトル fを微分可能なrの関数とするとき
rot(C×rf)を求めよ。

rot(C×r)なら分かるんだけど、fがいまいち理解出来ない。
http://www.moge.org/okabe/temp/elemag/node133.html
↑のrot(A×B)のやつそのまま使っておkなの?

360132人目の素数さん:2010/04/21(水) 23:07:20
有理数全体Qにおいて関係〜を次のように定める

a〜b:=0でない整数m,nがありa^m=b^nとなる。
このとき、〜はQ上の同地関係になる事を示せ。

誰かお願いします…><
361132人目の素数さん:2010/04/21(水) 23:10:57
>>360
同値関係の定義をまずここに書いてみろ
362132人目の素数さん:2010/04/21(水) 23:27:24
反射律a〜a
対象律a〜b⇒b〜a
推移律a〜b,b〜c⇒a〜c

これでいいのか?
363132人目の素数さん:2010/04/21(水) 23:32:58
>>362
OK。あとは三つ全て成り立っているか示せばいいだけ。
364132人目の素数さん:2010/04/21(水) 23:33:39
>>362
その3つのうち、2つくらいは自力でできるんじゃないか?
365132人目の素数さん:2010/04/21(水) 23:59:43
ちょっとやってみる
366132人目の素数さん:2010/04/22(木) 00:32:49
やってみたがわからなくて笑えてきた

何をどうやって書けばいいのかも全然わからない
どうしよう
367132人目の素数さん:2010/04/22(木) 00:37:00
どんなq∈Qをもってきてもq^1=q^1だからq〜qが成り立つ … 反射律

どんなp, q∈Qをもってきても p^m=q^nならq^n=p^mだから
p〜q⇒q〜pが成り立つ …対象律

どんなp, q, r∈Qをもってきても…ここを書いてみ
368132人目の素数さん:2010/04/22(木) 00:58:58
今ちょっとずつ書いてる

色んな考えをまとめながら少しずつ書いてるから遅くてごめん
369132人目の素数さん:2010/04/22(木) 01:09:59
p,q,r∈Qを任意に取る
〜の定義よりp^m=q^n,q^n=r^lが成り立つので
等号の性質を利用してp^m=r^l
よってq〜rとなる

こんな感じでいいのかな
370132人目の素数さん:2010/04/22(木) 01:12:39

p〜qかつq〜rとする

っていうのを2行目にはさんでください
371132人目の素数さん:2010/04/22(木) 07:46:43
なんで都合よくnがどちらにも現れるんだ
372132人目の素数さん:2010/04/22(木) 12:05:42
t
373132人目の素数さん:2010/04/22(木) 14:18:09
問題じゃないんですけど勝つ確率5割で勝つと1枚チップをもらえるゲームで
チップをどんどん増やす方法ってありますか?
374132人目の素数さん:2010/04/22(木) 14:21:13
>>373
負けてもチップが減らないならゲームをどんどんやるだけ。
375132人目の素数さん:2010/04/22(木) 14:22:51
>>373
通常の答え
あるわけがない。あっても誰も言わない

別の答え
負けなければいい
負けてもチップを払わなくていいゲームならやればいい・・・など
376132人目の素数さん:2010/04/23(金) 01:45:37
どうしてもわからない問題があったので質問します。
f(x)=sin1/x(x≠0),0(x=0)
という関数のxにおける連続性は?
という問題です。
指導教員によれば、an=1/(π/2+2nπ),bn=1/(-π/2+2nπ)とおいて、
lim[n→∞]f(an)=1,lim[n→-∞]f(bn)=-1だからlim[x→0]f(x)は存在せず、f(x)はx=0で連続ではない。
だそうなんですが、理解できない点があります。
まず、lim[n→∞]f(an)=lim[n→∞]sin(π/2+2nπ)ですが、n→∞なので、lim[n→∞]f(an)の値は収束せず振動するのではないか、という点。
lim[n→-∞]f(bn)も同様の理由で、値は収束するのではなく振動するのではないかと思います。
また、そもそもなぜlim[n→∞]f(an)=1,lim[n→-∞]f(bn)=-1という二つの極限からlim[x→0]f(x)での連続性の有無を示すことができるのかも理解できません。
拙い文章で申し訳ないのですが、ぜひ解説してほしいです。
よろしくお願いします。
377132人目の素数さん:2010/04/23(金) 02:14:01
>>376
lim[n→∞]f(a_n)=lim[n→∞]sin(π/2+2nπ)はnが自然数って条件省いてるな
nが実数ならa_nもb_nも数列(定義域が自然数の関数としても捉えられる)ではなく
関数(定義域が実数の関数)になる

後者は教科書とかで連続の定義がどうされているか見直せとしか
378132人目の素数さん:2010/04/23(金) 20:41:39
欧米では0.35は.35って普通に書くぞ。日本だけガラパゴスなんですよ。
379132人目の素数さん:2010/04/23(金) 20:57:32
「数」は無限でしょうか?
380132人目の素数さん:2010/04/23(金) 21:19:25
>>378
とっくに決着した話をなんで蒸し返すの?
381376:2010/04/23(金) 23:00:01
>>377
遅くなりました。
レスポンスありがとうございます。
nは整数という条件があるんですね。
それなら納得できます。
後者のほうも納得できました。
ありがとうございました。
382132人目の素数さん:2010/04/24(土) 06:05:13
連続ってどう定義されるの?
383132人目の素数さん:2010/04/24(土) 06:16:12
>>382
なぜ… 教科書を 見ない…
384132人目の素数さん:2010/04/24(土) 06:29:58
「数」は無限でしょうか?
385132人目の素数さん:2010/04/24(土) 07:33:03
数学で無限とは終わりがないって意味
386132人目の素数さん:2010/04/24(土) 07:43:49
数学であつかう各種命題にはそもそも「終わり」ってのはあったのかい?
387132人目の素数さん:2010/04/24(土) 10:36:15
終わりがないのが終わり
388132人目の素数さん:2010/04/24(土) 16:32:56
周期Tのx(t)を標本化間隔T/Nで標本化したx(N/T)の離散フーリエ変換において
T/Nを大きくした場合、離散フーリエ変換後どのような変化がおこるか説明しなさい

どなたか分かりやすいサイト、もしくは解説を教えていただけないでしょうか
おねがいします
389132人目の素数さん:2010/04/24(土) 20:30:07
無限÷無限ってどーなるの?
390132人目の素数さん:2010/04/24(土) 20:31:16
すみません僕たちを助けてください
http://anchorage.2ch.net/test/read.cgi/livemarket2/1271814599/501-600
391132人目の素数さん:2010/04/24(土) 20:32:49
>>389
無限は数じゃないから演算できない。
無限÷無限形式の極限とは違う話。
392132人目の素数さん:2010/04/24(土) 20:42:42
>>390
質問は簡潔に。
その100レス読んで問題点を洗い出して解決し、わかりやすく説明して欲しい
という依頼なら「知るかボケ。んなヒマあるか。自分で数学勉強しろ」。

つーか、高校数学程度の素養もなしにFXに手を出す時点でクズ。
393132人目の素数さん:2010/04/24(土) 20:44:26
というかFXと数学は何か関係あるのか?
394132人目の素数さん:2010/04/24(土) 20:50:57
あるともいえるないともいえる
395132人目の素数さん:2010/04/24(土) 20:55:54
>>394
自分で何言ってるかわかってんの?
>>392でクズというだけの自信があるなら、数学ができるとFXで成功するって理屈を説明してくれない?
396132人目の素数さん:2010/04/24(土) 20:57:54
数字を扱うギャンブルに数学ができないのに手を出すのは微妙

レベルの高いことは知らなくても高校数学を簡単でいいから理解ができるくらいは必要なんじゃね?
397132人目の素数さん:2010/04/24(土) 20:58:34
どうせFXで30万ぐらいスったんだろww
数学には自信があると思い込んでいた自分が実は一番のクズ野郎だったことに気がついたほうがいいw
398132人目の素数さん:2010/04/24(土) 21:00:40
その短文の中ですでに論理が飛躍してるなw
399132人目の素数さん:2010/04/24(土) 21:03:01
>>395
俺は>>392ではないがとりあえず確率、統計を少しはかじったほうがいいかも
金融工学でもおk

考えに幅が広がる
400132人目の素数さん:2010/04/24(土) 21:05:01
>>395
> 自分で何言ってるかわかってんの?

↓コレそっくり適用できるな。前半と後半にまったく関連がない。本人は大アリだと思い込んでるようだが。

> >>392でクズというだけの自信があるなら、数学ができるとFXで成功するって理屈を説明してくれない?
401132人目の素数さん:2010/04/24(土) 21:10:24
あくまで統計は統計だからね
期待値を上げる手段にしかならないけど、
その期待値を求める為の関数が出来るだけ大量に必要になる。
数学より経済学からはじめた方が成功率が上がりそうなわけ
402132人目の素数さん:2010/04/24(土) 21:11:31
>>398,399
授業料としても30万は高いと思うけど、30万スったのは授業料だったという自戒のための考えの幅は広がるかも名w
403132人目の素数さん:2010/04/24(土) 21:20:00
お前にとって30万が大金なのは良くわかったよ。
404132人目の素数さん:2010/04/24(土) 21:22:52
30万ジンバブエ・ドルが何だって?
405132人目の素数さん:2010/04/24(土) 21:23:10
お得意の数学を使ってあなたは月30万円以上稼げますか?
406132人目の素数さん:2010/04/24(土) 21:27:08
質問.
空間Aに二つの点xとyを置く。
点が同一の座標に移動するまで、
α.どちらか片方の点のみをランダムに動かす
β.両方の点をランダムに動かす
どちらか片方の処理を選択し、同一座標に移動するまで延々と繰り返す。
この様な事を行った場合、方法αとβどちらを選択した方が
早く同一座標まで移動する可能性が高いか。

この問題ってどのような解答になるのでしょうか?
407132人目の素数さん:2010/04/24(土) 21:42:36
空間Aの性質によるだろう
408132人目の素数さん:2010/04/24(土) 21:44:34
思い込みって怖いなって改めて思った
409132人目の素数さん:2010/04/24(土) 21:46:01
>>395
> >>394
> 自分で何言ってるかわかってんの?
> >>392でクズというだけの自信があるなら、数学ができるとFXで成功するって理屈を説明してくれない?
A:数学の素養がない
B:FXに手を出す
392 は ある人がAかつBならその人はクズ
ある人がクズでないならその人は「AかつB」ではない。
すなわち、ある人がクズでないならその人は数学の素養があるかFXには手を出さない。
すなわち、そのひとが数学の素養があるならFXには手を出さない、同じことだた
FXに手を出しているなら数学の素養がある、ということだ

いずれにしろ、>>392さんに、数学ができるとFXで成功する、などという理屈を説明する義務はないな。
410132人目の素数さん:2010/04/24(土) 21:50:14
>>409
で、FXだと合計いくら負けたん?
411406:2010/04/24(土) 21:51:22
とりあえず、空間Aは有限の2次元の空間でかまわないです。
別にユークリッド空間とかアフィン空間とかそういう特殊な物じゃなくてOKということで
412132人目の素数さん:2010/04/24(土) 21:55:17
>>406
どのみち確率0でないか?
413132人目の素数さん:2010/04/24(土) 22:01:46
>>412
確率0?それもまた問題だw
414132人目の素数さん:2010/04/24(土) 22:02:36
お得意の数学を使ってあなたは月30万円以上稼げますか?
415406:2010/04/24(土) 22:03:00
空間が有限だった場合、ループ回数を無限まで増やせば、
確率は0にはならないと思いますけど
416132人目の素数さん:2010/04/24(土) 22:06:59
>>415
ゼロ(定数として絶対ゼロ0.000)にならないというその考えもまた問題だw
417132人目の素数さん:2010/04/24(土) 22:07:43
>>412,413,415
時間も空間も離散的とするんだろう。
418132人目の素数さん:2010/04/24(土) 22:14:18
いろいろな本読んで勉強しているお得意の数学で議論して欲しいところだったんですけど、こういう具体的な問題になると素人と大差ないんですね。
ある程度数学の用語やそれらしい単語で語れば何が問題なのかわかるんでしょうけど、もしお得意の数学を本当に理解してるならそういう用語を使わなくてもついてこれたと思いますよ。
419132人目の素数さん:2010/04/24(土) 22:23:38
コンピュータ君お得意の演説が始まりました。
静聴せよ、静聴せよ
420132人目の素数さん:2010/04/24(土) 22:25:00
>>418
病人の文章だな。
誰が何を誰に伝えたいのかまるっきり分からない。
421132人目の素数さん:2010/04/24(土) 22:34:02
よくある「煽れば答えてくれると思っている」の手口。はい次。
422406:2010/04/24(土) 22:34:33
0になるならどういうロジックで0になるのでしょう。
αとβどっちが高く高くなるか。どれくらいの差が出る物なのでしょう
423132人目の素数さん:2010/04/24(土) 22:35:53
β上に立っている人に聞いてみな
424132人目の素数さん:2010/04/24(土) 22:37:29
>>420
あなたの方が問題あると思いますよ。
時間も空間も離散的と捉えると、ランダムは説明できるんですか?
425132人目の素数さん:2010/04/24(土) 22:38:00
>>406
壁を考慮せずに済むように、空間を無限に取るなら話は簡単。
両方を動かすのと、片方を固定してもう片方を2連続で動かすのは同じことなので、
両方動かす方は、片方だけ動かすときの半分の時間で到達できる。
到達できる確率は同じ。

ただし、最初の時点で2粒子の座標の差の成分和が奇数だと
両方動かす方では衝突できない。
426406:2010/04/24(土) 22:41:09
>>425
αとβの違いは、「動く」という処理のステップが1つか2つかの違いだから、
α:β=1:2
という事ですね。
ありがとうございます。
427132人目の素数さん:2010/04/24(土) 22:44:46
>>424
FXで小銭スッたからって逆ギレすんなよ
428132人目の素数さん:2010/04/24(土) 22:51:59
>>424
数学以前に日本語勉強してきなさい。
はい、次。
429132人目の素数さん:2010/04/24(土) 22:52:20
ある定点で出会う、ということと
単に出会うということとが曖昧に語られている。
430132人目の素数さん:2010/04/24(土) 22:53:08
何コイツ?
何様のつもり??w
431132人目の素数さん:2010/04/24(土) 22:57:41
自慢の数学を駆使してFXをやってみて30万もスったのに「大金じゃねー」とかいって涙目な数ヲタw
432132人目の素数さん:2010/04/24(土) 22:58:51

30万で決まりなんだww
433132人目の素数さん:2010/04/24(土) 22:59:28
お得意の数学を使ってあなたは月30万円以上稼げますか?
434132人目の素数さん:2010/04/24(土) 23:00:55
繰り返すな
435132人目の素数さん:2010/04/24(土) 23:00:57
>>432
何であなたの「w」は小文字なんですか?
それも「ww」と2つも?w
436132人目の素数さん:2010/04/24(土) 23:02:06
>>431
いるいるww
437132人目の素数さん:2010/04/24(土) 23:05:10
>>429
○○の公理系を採用した○○空間で・・・の言語で書かないともうあなたは他人とはコミュネケーション出来なくなっちゃったんですか・・・?
438132人目の素数さん:2010/04/24(土) 23:10:29
「同一の座標に移動」「同一座標に移動」「同一座標まで移動」
この3つの意味が曖昧だ、という日本語の表現を問題にしているだけ。
439132人目の素数さん:2010/04/24(土) 23:18:14
自分で問題を解いたこともなくいかに教科書天下りのおんぶにだっこだったかを自覚してきたみたいですね。
440132人目の素数さん:2010/04/24(土) 23:23:03
この流れであえて聞く
FXって何?いや、やっぱり答えなくてもいいや
441132人目の素数さん:2010/04/24(土) 23:24:55
お得意の数学を使ってあなたは月30万円以上稼げますか?
442132人目の素数さん:2010/04/24(土) 23:29:17
>>440
フリーセックスの略
443132人目の素数さん:2010/04/24(土) 23:30:43
>>439
で406の同一の座標とはどういう意味で語っているの
444132人目の素数さん:2010/04/24(土) 23:31:48
>>440
富士ゼロックスだよ
日本人なら知っておいた方が良い
445132人目の素数さん:2010/04/24(土) 23:40:04
>>440
次期主力戦闘機だよ。常識だろ
446132人目の素数さん:2010/04/24(土) 23:52:24
∞から∞を割ることは可能なのでしょうか?
可能だとしたら、答えは何になるのでしょうか?
447132人目の素数さん:2010/04/24(土) 23:56:11
不能
448132人目の素数さん:2010/04/24(土) 23:59:23
>>446
蒸し返すな、クズ
449132人目の素数さん:2010/04/25(日) 00:00:09
>>446
もう答えもらってるだろ
450132人目の素数さん:2010/04/25(日) 00:00:59
>>438
「同一の座標に移動」「同一座標に移動」
どう違うんですか?
451132人目の素数さん:2010/04/25(日) 00:03:00
>>403,409,432,448

お得意の数学を使ってあなたは月30万円以上稼げますか?
452132人目の素数さん:2010/04/25(日) 00:04:42
>>449
もらってません。だから教えてください。
453132人目の素数さん:2010/04/25(日) 00:05:59
>>452
スレ読み返せ
454132人目の素数さん:2010/04/25(日) 00:07:52
>>453
どこから?
455132人目の素数さん:2010/04/25(日) 00:08:18
>>452
教えた
456132人目の素数さん:2010/04/25(日) 00:11:16
>>455
いや、教えてもらってません。
457132人目の素数さん:2010/04/25(日) 00:18:08
教えたって言ってるから教えたんだろう。
たぶんあのレスだろうと思うが。
それを教えてもらってないと言うのなら、今回教えても認識できないだろう。
つまり、改めて教えても無駄。
458132人目の素数さん:2010/04/25(日) 00:19:50
>>456
教えた。
459132人目の素数さん:2010/04/25(日) 00:24:39
スロットでも30万は大金じゃないね
460132人目の素数さん:2010/04/25(日) 00:28:13
というか、概念が違うってさっき言ってたしなw
30万とかくだらない事に固執してるようなやつは論理的分析が必要になるようなFXやっても失敗するだけだと言える
461132人目の素数さん:2010/04/25(日) 00:34:01
えええ!
FXごときに論理的分析なんかが必要なんですか?!
462132人目の素数さん:2010/04/25(日) 00:36:23
「FXごとき」って言ってる奴は十中八九負けて封印中
463132人目の素数さん:2010/04/25(日) 00:41:19
>>460
統計・確率って数学じゃなかったんですか?
464132人目の素数さん:2010/04/25(日) 00:46:04
>>450
少なくも一方が移動するとき、二つの点が出会ふ確率は如何
と問へば誤解は生ぜざるべしと言ふべけんや。
465132人目の素数さん:2010/04/25(日) 00:58:55
>>463
なかったんです。
466132人目の素数さん:2010/04/25(日) 01:00:32
>>438
あなたが日本語表現ってものにこだわりをお持ちなら、「同一座標」って書いてあるので曖昧なのは3つじゃなくて2つじゃないですか?
467132人目の素数さん:2010/04/25(日) 01:01:50
>>460
パチンコでも30万はぜんぜん大金じゃないよね?
468132人目の素数さん:2010/04/25(日) 01:02:14
>>466
そういうことも含めてです。
469132人目の素数さん:2010/04/25(日) 01:04:23
で、>>450
470132人目の素数さん:2010/04/25(日) 01:06:43
確率・統計は論理学扱いだったんですか…びっくり!?
471132人目の素数さん:2010/04/25(日) 01:07:09
同一座標まで、という表現は気持ち悪い。
なにか、行く先が措定されている表現だから。
472132人目の素数さん:2010/04/25(日) 01:08:16
確率論は解析学だから数学だけど、確率・統計は統計学。論理学とはどちらも縁がない。
473132人目の素数さん:2010/04/25(日) 01:18:38
で、>>460の論理的分析って何のことですか?
474132人目の素数さん:2010/04/25(日) 01:24:56
確率論って(関数)解析学扱いだったんですか?
475132人目の素数さん:2010/04/25(日) 01:26:28
測度論ですよ。
476132人目の素数さん:2010/04/25(日) 01:32:38
>>474
いいえ、るベー愚積分です
477132人目の素数さん:2010/04/25(日) 01:38:22
測度論ですか?
確率というからには |s| > 1 or |s|==1となるsは当然考えない公理で展開されるんですよね。
478132人目の素数さん:2010/04/25(日) 01:55:16
何だよFXって・・・F(x)だろ
479132人目の素数さん:2010/04/25(日) 01:55:25
>>458
教えてもらってません。
480132人目の素数さん:2010/04/25(日) 01:58:29
>>478
それ、俺が二年前に考えて発表を自粛したギャグなんだが
481132人目の素数さん:2010/04/25(日) 02:11:12
>>479
教えた。
482132人目の素数さん:2010/04/25(日) 02:18:27
測度論スレは荒れてるのか。
もう少し知的(知能が高い)だったらちょっとだけ相手してやってもよかったけど、ただの勉強不足な高校生の延長って感じがしなくもない。
483132人目の素数さん:2010/04/25(日) 02:23:54
Xー37Bの飛行実験が成功したそうです。シャトルはただのデコイでした。
484132人目の素数さん:2010/04/25(日) 02:36:48
485132人目の素数さん:2010/04/25(日) 02:39:19
486132人目の素数さん:2010/04/25(日) 02:55:18
リデストリビューションのベースが合意の下の搾取だから。。。うそだろ
H大はあほですね。
完全雇用ができないのでガス抜きのためにエアクッションが置かれているだけ。
アレは洗脳だな
487132人目の素数さん:2010/04/25(日) 03:06:50
もし市場がナベアツ数にしたがっていたら。。。
488132人目の素数さん:2010/04/25(日) 04:04:05
t
489132人目の素数さん:2010/04/25(日) 04:26:28
無限から無限を引いたら答えは何になるのでしょうか?
490132人目の素数さん:2010/04/25(日) 04:47:06
無限は状態なので一般的な加法は定められていない
491132人目の素数さん:2010/04/25(日) 04:49:29
>>490
もうちょっと詳しく説明してくれませんか?お願いします。
492132人目の素数さん:2010/04/25(日) 05:09:18
>>491
無限は何かを足したり引いたりするものじゃない
それだけ
493132人目の素数さん:2010/04/25(日) 05:12:47
>>490
無限が出て来ると問題になるのはどういう問題を計算している時だと思いますか?
494132人目の素数さん:2010/04/25(日) 05:21:08
>>492
じゃあ、無限を足したり引いたりするものとして考えた場合、どうなりますか?
495490 492:2010/04/25(日) 05:28:53
あーっと…
そりゃ全ての具体的な数より大きな∞ってのを数のように考えて
ある有限値a(正零負どれもあり)に対して
∞ + a = ∞ とか -∞ + a = -∞ とか考えることはあるけれど
あくまで∀a∀b∃x、x+a > b のような過程を経ているだけであって
同じような思考法で∀a∀bに対してa+bがどのような性質を持つか
考えればそれは不定にしかならない。
ただし例えばaとbに特別な相関があればその限りではないが
それはaとbにどんな相関があるか個々にみていくしかないだろう。

>493
え?なんで俺がそんな質問されるんだろ。

と書いていたら>>494
そういう非一般的、特殊な話を論じたいのなら
対象となる集合と加法と無限の定義を自分で出してみること。
496132人目の素数さん:2010/04/25(日) 05:47:34
>>495
その議論で突き進むなら、今まで∞としておいた「それ」を、
必要なら新たに○○公理を作って○○がおkなマイ○○を定義すればおかkってことですか。
しかもマイ○○はa,bに相関があることは当然重視しますよってことで。
もしそう考えてるなら∞の議論って感じじゃなくて○○論の方でしょうか。
497132人目の素数さん:2010/04/25(日) 05:53:08
無限とは落ちこぼれの頭が作り出した幻です。真実は今井塾セミナーのホームページに書かれてあります。
498132人目の素数さん:2010/04/25(日) 07:22:05
で、>>460の論理的分析って何のことですか?
499132人目の素数さん:2010/04/25(日) 09:02:34
エクステンデイッドリアルナンバーは∞の加減乗除をやるよ。
500132人目の素数さん:2010/04/25(日) 10:15:34
減と除は部分演算だろ
501132人目の素数さん:2010/04/25(日) 10:16:01
10個の区別がつかない玉を3個の容器にいれるときの確率はいくらか。ただし2つの容器が0個でもいい。

この問題の解き方を教えてください

○=玉、■=容器のしきりとして12!/10!2!の同じものがある順列では答えはでないのは何故でしょうか?
502132人目の素数さん:2010/04/25(日) 10:23:59
容器の区別がないから
503501:2010/04/25(日) 10:27:24
容器の区別がないからと思って12!の分母に更に2!をつけたら駄目なのでしょうか…
504132人目の素数さん:2010/04/25(日) 10:39:28
>>503
そういう疑問を持つときは、地道に全部書き出してみる姿勢が大事。
どこに同じものが現れるのかとか、どういう理屈で最後に割っているのかとか、
自分で手を動かしながら考えると、曖昧な当てっこゲームを繰り返して
時間を無駄にするハメにならずに済む。
505132人目の素数さん:2010/04/25(日) 10:59:18
>>501
> 10個の区別がつかない玉を3個の容器にいれるときの確率はいくらか。
問題が意味不明なのだが。
506132人目の素数さん:2010/04/25(日) 12:56:13
>>504
わかりました ありがとうございます

>>505
すいません
507132人目の素数さん:2010/04/25(日) 14:07:08
10=
0010
019
118
127
028
226
037
136
235
334
046
145
244
442
055
508132人目の素数さん:2010/04/25(日) 14:11:56
(a+b+c)^10=10Cra^r(10-r)Cpb^pc^(10-r-p)
P(r,p,10-r-p)=10Cr(10-r)Cp/3^10
509490 492:2010/04/25(日) 14:12:22
>>507
244と442が被ってる
510132人目の素数さん:2010/04/25(日) 14:15:57
P=P(r,p,10-r-p)/3!=10Cr(10-r)Cp/(6*3^10)
511132人目の素数さん:2010/04/25(日) 14:31:58
10!/(10-r)!r!*(10-r)!/(10-r-p)!p!=10!/r!(10-r-p)!p!
P(0,0,10)+P(10,0,0)+P(0,10,0)=3*10!/0!10!0!3*3^10=1/3^10
P(1,1,9)=10!/1!8!1!/3^10=90/3^10=10/3^8
512132人目の素数さん:2010/04/25(日) 14:40:16
P=P(r,p,10-r-p)=10Cr(10-r)Cp/(3^10)
513132人目の素数さん:2010/04/25(日) 14:55:46
P=P(r,p,10-r-p)+P(p,10-r-p,r)+P(10-r-p,r,p)+P(p,r,10-r-p)+P(r,10-r-p,p)+P(10-r-p,p,r)
=3!*(10Cr(10-r)Cp)/(3^10)
=3!10!/r!(10-r-p)!p!

514132人目の素数さん:2010/04/25(日) 14:59:08
オイラーの公式はどう見ても定義ですよね?
これで感動する理由がよく分からない...
515132人目の素数さん:2010/04/25(日) 15:01:42
>>514
ほう?
516132人目の素数さん:2010/04/25(日) 15:10:41
(a+b+c)^10
P(0,0,10)+P(0,10,0)+P(10,0,0)
P(r,p,10-r-p)=
3!P(r,p,10-r-p),r<p<10-r-p
3C2P(r,r,10-2r),r<10-2r
P(r,r,r),3r=10
517132人目の素数さん:2010/04/25(日) 15:14:50
(ai)^m
P(si)=mCs1(m-s1)Cs2.../n^m
=m!/s1!s2!...n^m
P=F(si)P(si)
518132人目の素数さん:2010/04/25(日) 15:28:51
人様が何に感動するかなんてわかったもんじゃないだろ、普通
519132人目の素数さん:2010/04/25(日) 15:32:41
e^x=1+x+x^2/2+x^3/6+...
を実数で証明した後、虚数を代入する無茶ぶり
520132人目の素数さん:2010/04/25(日) 15:39:17
オイラーの公式を知った時はπ代入したときにe^iπ=-1になることに感動しました
521132人目の素数さん:2010/04/25(日) 16:20:13
ふつうはそれに行列Aを入れるんだぞ。けっこう回して使っている。
522132人目の素数さん:2010/04/25(日) 16:26:27
俺は作用素を入れる
523132人目の素数さん:2010/04/25(日) 16:54:13
どうしてもわからない問題があるので教えて下さい。
数学に関しては素人です。

あるパーティーで司会者が私にA、B、C、三つの箱を差し出した。
この箱の中には一個だけ賞金が入っており、残り二つは空っぽだという。
司会者が私にこの三つの箱から一つだけ選べという(司会者はどの箱が当たりかを知っている)。

私はここでAを選んだ。
司会者はそこでCを開け、中身が空っぽなのを私に見せる。
司会者は私に言う。
「今ここで変更するチャンスをあげます。Bにしますか?それともAのままで良いですか?」
さて、ここで変更した場合と、変更しない場合と、どちらが賞金を当てる確率が高いか。

変更した場合の方が当たる確率が高くなるというのが正解らしいのですが、どうも理解できません。
524523:2010/04/25(日) 16:57:06
以下のような理解は本当に正しいのでしょうか?

解説によればAで当たる確率は1/3、
BとCを一個の箱として考えれば、それが当たりの確率は2/3である。
Cが外れである今、Aが当たりの確率は1/3、Bが当たりの確率は2/3である。

これをよりわかりやすく説明するために100個の箱(1)〜(100)で例える人もいます。
つまり、100個のうち当たりがあるのはたった一個。自分が一個目の箱(1)を選んだ後、
司会者がその他98個(3)〜(100)の箱を開け、外れであることを私に見せる。
(1)が当たりの確率は1%だが、(3)〜(100)の99個を一個の箱として考えたとき、
それが当たりである確率は99%である。
(3)〜(100)が外れであることがはっきりした以上、
(1)が当たりである確率は1%であり、(2)が当たりである確率は99%である。

このような解説はどうも子供だましであるような気がしてなりません。
選べる箱は結局最終的には当たりか外れかの二個なのだから、1/2じゃないですか?
司会者が外れの箱を開いて私に見せるというのは関係ない気がします。
変更するかしないか私に持ちかけた時点で私が箱を選ぶものと考えると、1/2しかないじゃないですか・・・
525523:2010/04/25(日) 16:58:38
>>524に誤りがあったので訂正します

× (3)〜(100)の99個を一個の箱として考えたとき、
○ (2)〜(100)の99個を一個の箱として考えたとき、
526132人目の素数さん:2010/04/25(日) 17:01:21
何をどう解説されたってお前は
>選べる箱は結局最終的には当たりか外れかの二個なのだから、1/2じゃないですか?
としか考えないだろうよ。
527523:2010/04/25(日) 17:04:52
>>526
まぁそう言わず、解説してみて下さいよ・・・
528132人目の素数さん:2010/04/25(日) 17:08:43
>>524
>司会者が外れの箱を開いて私に見せるというのは関係ない気がします。

そのとおり
だからAがハズレの確率は2/3のまま
529132人目の素数さん:2010/04/25(日) 17:25:40
530132人目の素数さん:2010/04/25(日) 17:26:20
531132人目の素数さん:2010/04/25(日) 17:34:37
>>519
やっぱりそれは厳密にはおかしいんですか?
自分は確かそういう風に教えてもらいましたが、感覚的な理解と言われました。
532132人目の素数さん:2010/04/25(日) 17:35:16
ある賭場で壷振りお蝶が私にA、B、C、三つの壷を差し出した。
この箱の中には一個だけ丁が入っており、残り二つは半だという。
お蝶が私にこの三つの壷から一つだけ選べという(お蝶はどの壷が丁かを知っている)。

私はここでAを選んだ。
お蝶はそこでCを開け、中身が半なのを私に見せる。
お蝶は私に言う。
「今ここで変更するチャンスをあげます。Bにしますか?それともAのままで良いですか?」
さて、ここで変更した場合と、変更しない場合と、どちらが丁を当てる確率が高いか。
533132人目の素数さん:2010/04/25(日) 17:37:35
ある賭場で壷振りお蝶が私にA、B、C、三つの壷を差し出した。
この壷の中には一個だけ丁が入っており、残り二つは半だという。
お蝶が私にこの三つの壷から一つだけ選べという(お蝶はどの壷が丁かを知っている)。

私はここでAを選んだ。
お蝶はそこでCを開け、中身が半なのを私に見せる。
お蝶は私に言う。
「今ここで変更するチャンスをあげます。Bにしますか?それともAのままで良いですか?」
さて、ここで変更した場合と、変更しない場合と、どちらが丁を当てる確率が高いか。
534132人目の素数さん:2010/04/25(日) 17:43:13
壷に丁が入っているのは最初から1/2の確率
3個の壷から丁を当てるのは1/3の確率
2個になったから丁を当てるのは1/2の確率
変えても変えなくてもおなじこと。

535 ◆27Tn7FHaVY :2010/04/25(日) 17:45:19
そいつは数板ふぁQ100問だな
536132人目の素数さん:2010/04/25(日) 17:48:53
でも正解は3個の壷のあたりの確率はそれぞれ1/3で変わらない。

つぼに丁を入れた最初の試行はそれ以後の事象の結果の影響は受けない。

あとから壷を1個足してシャッフルしても。
537132人目の素数さん:2010/04/25(日) 17:53:19
>「今ここで変更するチャンスをあげます。Bにしますか?それともAのままで良いですか?」

と司会者が持ちかけた時点で、私が賞金を選ぶと考えた場合、
AかBのどちらかにある賞金を当てる1/2の確率にしかならないのでは?
箱を3個提供しようが100個提供しようが1000個提供しようが、
司会者は勝手に外れの1個、98個、998個を開け、結局はどちらかに当たりのある2個を提供するわけでしょ?
Aが1/3でBが2/3、あるいはAが1/100でBが99/100、Aが1/1000でBが999/1000なんてミラクルなことになるのか・・・

もしそれを認めてしまうと、・・・
司会者が私に2個の箱A、Bを提供する。どちらか1個に賞金がある。
私はAを選ぶ。司会者はどこからか中身が空っぽである箱を98個を持ってきて私に見せる。
この時点でBが当たりの確率が一気に99%になる、
というようなミラクルを認めてしまうような理屈に聞こえる
538132人目の素数さん:2010/04/25(日) 18:00:33
ラプラス変換に強い人いますか
539132人目の素数さん:2010/04/25(日) 18:36:08
ラプラスの悪魔ってなに?
540132人目の素数さん:2010/04/25(日) 18:39:04
>>537
前者は情報が増えてるけど、後者はなんの情報も増えてない
541132人目の素数さん:2010/04/25(日) 18:40:22
>>539
wikipediaに基本的なこと載ってるよ…
仮想の「未来を完璧に計算する悪魔」のこと
542132人目の素数さん:2010/04/25(日) 18:43:55
マクスエルの悪魔も超時空の悪魔ならエントロピーは増大しないのですね
フラクタル時空の中で位置を見ればすべての未来がすべて見えるから
543132人目の素数さん:2010/04/25(日) 18:44:04
数学って難しいネー( ´゚д゚)(゚д゚` )ネー
544132人目の素数さん:2010/04/25(日) 19:22:25
http://www.hinduism.co.za/vedic.htm#Books on Vedic Maths
ここにインド式計算について乗ってるんだけど、どうしてこんなことが成り立つの?
545132人目の素数さん:2010/04/25(日) 19:23:41
ああ、なんか自己解決できそうだから頑張ってみる。
546132人目の素数さん:2010/04/25(日) 23:16:01
547132人目の素数さん:2010/04/25(日) 23:21:55
lim[x→∞]f(x)=a ならば lim[x→∞]|f(x)|=|a| (ただしa≠0)
をイプシロンデルタ論法を使って証明しろと言われ、
イプシロンデルタ論法自体は理解したつもりでしたがどう使うのかが全然分かりません
548132人目の素数さん:2010/04/25(日) 23:33:02
>>546
自身の判断を信じなさい
549132人目の素数さん:2010/04/25(日) 23:42:38
>>548
曖昧な3つを自分で列挙しておいてそんな自分がどういう判断を下せるというんですか?
550132人目の素数さん:2010/04/25(日) 23:49:56
「無」は無限ですか?
551132人目の素数さん:2010/04/26(月) 01:02:10
|f-a|>||f|-|a||>-|f-a|
552132人目の素数さん:2010/04/26(月) 01:15:33
>>549
あなたの日本語を信じなさい、ということです。
それが信じられないなら、それがあの文の曖昧さということになりますね。
幸運を祈ります。
553132人目の素数さん:2010/04/26(月) 01:18:56
554132人目の素数さん:2010/04/26(月) 01:21:13
>>547
まず
>lim[x→∞]f(x)=a
をεδを使って書く。
次にlim[x→∞]|f(x)|=|a|(を意味するεδの論理式)を示す
555132人目の素数さん:2010/04/26(月) 03:24:11
確率の問題です。教えてください。

300人ずつのグループが90組あり、合計27000人のリストがあります。
そのリストの中から1000人をランダムに選んで1列にならべた時、
同じグループの人が隣同士になる確率はどれくらいですか?
556132人目の素数さん:2010/04/26(月) 04:04:01
t
557132人目の素数さん:2010/04/26(月) 10:25:54
>>555
解析的には簡単に出せないと思う
558132人目の素数さん:2010/04/26(月) 11:01:33
この板で速レスしてくるような数ヲタニートさんって日ごろ何してるんですか?
559132人目の素数さん:2010/04/26(月) 11:50:50
>>557さん
レスありがとうございます。
どういう考え方をすればよいですか?
解析的に出せないというのは、その組み合わせ(順列?)が何通りあるか
地道に数えないといけないということでしょうか。
また教えていただけるとありがたいです。
560132人目の素数さん:2010/04/26(月) 11:54:58
横レスだが実直に>>555をテストしてみたらほとんど100%だったが
理論値が必要だとしたら俺にはお手上げ
561132人目の素数さん:2010/04/26(月) 11:59:29
場合分けが莫大になってしまうんじゃないんかな?
うまい方法って存在するんだろうか?
562132人目の素数さん:2010/04/26(月) 12:05:37
>>560の言うようにシミュレーションするのが良いと思う
563132人目の素数さん:2010/04/26(月) 12:26:11
この板で速レスしてくるような数ヲタニートさんって日ごろ何してるんですか?
564132人目の素数さん:2010/04/26(月) 12:51:01
>>560->>562さん
レスありがとうございます。
100%になるとは思っていませんでした!すごく低い確率なのかと・・・。(数学は苦手です)
100%とは、同じグループの人が少なくとも1組は隣同士になるってことですよね?
シミュレーションのやり方がよくわかりません・・・。すみません、要領を得なくて。
どういう手順なのか教えていただけると助かります。

>>555の問題で、組数が900組に増えて、(300人*900組=27万人)
その中から1万人を選ぶ場合についてはどうでしょうか?
これはもっと低い確率になりますか?
565132人目の素数さん:2010/04/26(月) 12:56:47
>>564
本当に解きたい問題はどんな?
566132人目の素数さん:2010/04/26(月) 13:02:34
>>564
元の問題の意味が不明確だが、どの人も自分(その人自身)と同じグループの人と隣り合わせではないって意味?
それなら、100%っぽいって思ったけどなあ。
新しい条件の場合でも1万人も選んだら同グループが隣り合わせになっていない確率はかなり低いんじゃないか?
つまり、どこかで同グループの人が隣り合っている確率はかなり高いんじゃないかな。

同じクラスに誕生日が同じなのが一組もいない確率が意外に低いってのに似てる。
567132人目の素数さん:2010/04/26(月) 13:13:04
バグとかがなければいいんだが
prompt%perl -MList::Util
for$x(1..900){
  for$y(1..300){
    push @a,$x;
  }
}
while(1){
  print STDERR qq#$q/$p #;
  $p++;
  @a=List::Util::shuffle(@a);
  for$x(0..9998){
    if($a[$x]==$a[$x+1]){
      $q++;
      print STDERR qq/ group $a[$x] = \$a[$x] = \$a[$x+1]\n/;
      last;
    }
  }
}
ほとんど100%になるようだ
568132人目の素数さん:2010/04/26(月) 13:30:16
この板で速レスしてくるような数ヲタニートさんって日ごろ何してるんですか?
569132人目の素数さん:2010/04/26(月) 15:01:57
>>568
俺は学生です
ほかの人の事はわからない
570132人目の素数さん:2010/04/26(月) 15:13:40
ニートに尋ねてるのに、返答する学生(?)乙
571132人目の素数さん:2010/04/26(月) 15:15:18
ボクはニートです
日頃はオナニーをして暮らしています
572132人目の素数さん:2010/04/26(月) 18:20:46
ボクはオナニー以外に他人のあら探しもしています
573132人目の素数さん:2010/04/26(月) 19:49:42
>>555です。
たくさんのレスありがとうございました。お礼が遅くなってしまってすみません。

>>565さん
>>555の問題+その人数が変化したときに確率はどう変化するのかが知りたかったです。

>>566さん
説明がわかりにくくてすみません。
そうです、逆に言えば、自分自身と同じグループの人と隣合わせになる人が、1000人のうち誰もいないってことです。
大きな数すぎて、わからなくなってしまいました。

>>567さん
あぅあぅあぅ、なんだかすごい。
そのプログラムでシミュレーションできるのですか。
私にはわかりませんが、どうもありがとうございました。

>>555の問題は、どこかで誰かしらが同じグループの人と隣り合ってしまうんですね。
なんだか不思議です。同じ誕生日の話も意外でした。
感覚的には、同じ誕生日の人と同じクラスになるなんて、すごい偶然って思ってしまいます。
皆さま、ありがとうございました!
574132人目の素数さん:2010/04/26(月) 19:52:58
改行が多いと怒られてしまったので連投すみません。
しつこいですが>>555です。
もう一問教えていただきたい問題があります。
以下の問題なのですが、なんだか解けそうと思ったのもつかの間でした。
何通りあるか、数えてみようと思ったのですが、わからなくなってしまいました。

<問題>
赤・青・黄・緑の4色×各6コずつの24コのボールがあります。
これを全部袋に入れて、そのうちの6コを順番に取り出していきます。
(一度取り出したボールは袋には戻さない。)

(1) 赤のボールが連続する確率は?
(2) 同じ色のボールが連続する確率は?
575132人目の素数さん:2010/04/26(月) 21:01:53
5*4^3/4^6=5/4^3
4*5/4^3=5/4^2
576132人目の素数さん:2010/04/26(月) 21:19:43
sinc関数をラプラス変換したい時どうすれば良いですか?
577132人目の素数さん:2010/04/26(月) 21:23:17
nxnの為替レートのマトリックスAがある
nxnの外貨準備のマトリックスBがある
nxnの外貨需要のマトリックスCがある
毎回Cにしたがって取引が行われると、時刻tにはBはどうなっているか
Aは時刻t-1のBにしたがって上下する、その変動率をnxnのマトリックスRであらわす。
式を立てて連立微分法定式を固有値でときなさい。
外貨準備の境界値を適当に設定しなさい。

まあ30点
578132人目の素数さん:2010/04/26(月) 21:26:18
579132人目の素数さん:2010/04/26(月) 21:37:32
売買函数
レートが購入レートよりr高いとき売り
レートが購入レートよりr低いとき売り
それいがいは持ち越し
580132人目の素数さん:2010/04/26(月) 21:38:13
写像任意のxをx^2に移すf_1があるとき
f_1^(-1)([0,1])ってどうやって求めればいいんでしょう?
[-1,1]も[0,1]も当てはまりますよね?
まとめて書く方法があるんですか?
581132人目の素数さん:2010/04/26(月) 22:17:57
+/-y^.5
582132人目の素数さん:2010/04/26(月) 22:22:29
e^(π √163) は10^(-10) の精度で計算すれば整数である。

お願いします。
583132人目の素数さん:2010/04/26(月) 22:28:45
>>582
そのまんまぐぐれ。
584132人目の素数さん:2010/04/26(月) 22:51:01
ありがとうございます。>>583
585132人目の素数さん:2010/04/27(火) 06:36:33
t
586132人目の素数さん:2010/04/27(火) 10:15:21
この板で速レスしてくるような数ヲタニートさんって日ごろ何してるんですか?
587132人目の素数さん:2010/04/27(火) 11:13:31
1/2(log|x-a| - log|x+a|)
解き方がわからないので途中式もお願いします。
588132人目の素数さん:2010/04/27(火) 11:14:58
>>587
y=1/2(log|x-a| - log|x+a|)
でした すいません
589132人目の素数さん:2010/04/27(火) 11:15:43
これをどうしたいんだ?
590132人目の素数さん:2010/04/27(火) 12:16:35
X を2次元の有向線分全体のなす集合とする.X における関係∼x を

a ∼x b ⇔ |a| = |b| かつa, b は同じ向きを持つ(a, b ∈ X)

と定義する.このとき関係∼X は同値関係であることを示せ。
591132人目の素数さん:2010/04/27(火) 12:39:02
>>590
分からない問題はここに書いてね330
http://science6.2ch.net/test/read.cgi/math/1269099055/918
592132人目の素数さん:2010/04/27(火) 15:18:15
n種類からなる有根系統樹の可能な樹形の数は?
n種類からなる無根系統樹の可能な樹形の数は?

有根の場合
n=3で 樹形の数2
図形で表すと根一つ枝一つ葉が三つのパターンが二つ出来るので2です

お願いします
593132人目の素数さん:2010/04/27(火) 17:20:08
代数学の群論の問題です。

Vを複素数体C上のベクトル空間とし、GL(V)を、Vからそれ自身の上への同型写像全体のなす群とする。
p:G→GL(V)を、Gの線形表現とする。Vが0でなく、Vのどの部分ベクトル空間も、Gで保たれないとき、pは既約または単純であるという。定理1により、この第二の条件は、Vは二つの表現の直和でないということと同値である。
この、同値であることを証明せよというのが問題です。
ちなみに定理1は、
p:G→GL(V)を、GのVにおける線形表現とし、Wを、Gで安定なVの部分空間とする。このとき、VにおけるWの補空間W'であってGで安定なもの、が存在する。
どなたかお願いします。
594132人目の素数さん:2010/04/27(火) 17:22:43
定理1には条件がいるな
G は有限群か?
595132人目の素数さん:2010/04/27(火) 17:52:22
>>594
そうです。書きわすれてました汗
596132人目の素数さん:2010/04/27(火) 18:50:15
pじゃなくてρだったりしそうだなw
597132人目の素数さん:2010/04/27(火) 18:58:05
>>589
y=1/2(log|x-a| - log|x+a|) を微分せよ。 という問題でした
重要なところ抜かしててすいません
598132人目の素数さん:2010/04/27(火) 19:01:20
>>597
どの変数についてだ?
599132人目の素数さん:2010/04/27(火) 19:08:02
>>597
∂y/∂x=(-((x+a)|x-a|^2)+(x-a)|x+a|^2)/((x+a)|x-a|^2+(x-a)|x+a|^2)

答えを得たいのなら質問は推敲して書き込んだ方がイイ
600132人目の素数さん:2010/04/27(火) 20:02:27
>>574です。
>>575さん、ありがとうございました。
いただいた答えを元に考えてみましたが、???でした・・・。
もしよろしければ、考えかたを教えていただけるとうれしいです。

>>567さん
昨日は呪文のように見えましたが、ちょっとperlを勉強して、自分でも動かしてみました。
そのままでは自分の環境での動かしかたがわからず、
自分なりに改造して逐一出力しながらやってみました。(あっているかどうかわかりませんが)
手順としては以下の考え方であっていますか?ちょっとスレチですみません。

900×300の配列を作る(1から27万)
そのデータをそれぞれの所属するグループ名(1から900)に置き換える
データをシャッフルして並べ替え、前から9999番目までのデータを見る
n番目のデータとn+1番目のデータのグループ名が一致したら出力

1回ずつ実行して確認するコードしかできなかったので、何回も回して確認しました。
1回につきだいたい10個〜20個ぐらいは一致するものが出現していました。
601132人目の素数さん:2010/04/27(火) 21:57:04
>>590
同値関係の定義を知らなさそうだね
602132人目の素数さん:2010/04/27(火) 23:15:03
今 d/dx e^x=e^x の証明を http://aozoragakuen.sakura.ne.jp/kaisekikiso/node37.html ここを見ながら解いているんですが

1. e^(h)-1=X とおいたとき、 h=log(X+1) への変換の仕方が分かりません
とりあえず e^(h)=X+1 とするだろうなということは分かるんですが、これだと h=log[e](X+1) となってしまうのですがこれで正しいのでしょうか?

2. 証明の終りの方に 1/(log(e))=1 とあるのですがどのように持っていくのかわかりません
  こちらは見当がつかなくて困っています
603132人目の素数さん:2010/04/27(火) 23:25:46
1はいい

log e=1 は証明もなにもそのまま計算してるだけだろ
604132人目の素数さん:2010/04/27(火) 23:35:01
>>603
こちらの log e は log[e] ではなく log(e) で、log(e)=1ということでしょうか?
605132人目の素数さん:2010/04/28(水) 00:01:17
もう一つ分からない部分ができてしまいました
limの前に持っていった e^x はどこへ行ったのでしょうか?
606132人目の素数さん:2010/04/28(水) 00:08:19
自分なりに解釈できてきたはずなので確認お願いします

1/(loge)=1 は log[e](e)なので 1 となり、e^(x) × 1 = e^(x) となるので d/dx e^x = e^x でよろしいでしょうか?
607132人目の素数さん:2010/04/28(水) 00:22:19
>>606
合ってますよ
底がeのときは省略されることもあるって習いませんでした?
608132人目の素数さん:2010/04/28(水) 00:28:47
常用対数ばかり扱っていたのでてっきり10だと思い込んでいました…言われるまで忘れていました
ありがとうございました
609132人目の素数さん:2010/04/28(水) 04:33:47
>>600ですが、>>575さんに対する質問は取り下げます。
なんとか理解できました。ありがとうございました。
6105567:2010/04/28(水) 04:41:17
>>600
合っています
611132人目の素数さん:2010/04/28(水) 22:56:47
>>610さん
ありがとうございました。
612132人目の素数さん:2010/04/29(木) 07:40:12
鳩の巣箱の原理ってのは
公理/公準/定理のどれにあたるのでしょうか
613612:2010/04/29(木) 08:59:32
wikipediaの証明のページにあるように
数学的帰納法や背理法などと同じ分類のものみたいですね
(演繹法が載ってないのは何故)

背理法は直観主義論理では排中律の仮定が必要であるように
数学的帰納法や鳩の巣箱の原理も
それを公理(推論規則?)として認めるか認めないかで
なんちゃら論理という分類があるのかと思い
上記のような質問をしました

調べたところcoqではこのように書けるそうですが
該当pdfには証明(or証明ができるのか)は載っていませんでした
Lemma pigeon_hole :
forall m n, m < n ->
forall f, (forall i, i < n -> f i < m) ->
exists i, i < n /\
exists j, j < n /\ i <> j /\ f i = f j.

これがcoqで証明出来るのであれば
直観主義論理においては鳩の巣箱の原理は定理といえそうですが
多分、そうじゃないですよね?
どなたかよろしくお願いします
614132人目の素数さん:2010/04/29(木) 09:49:49
C君は、暇そうでいいにゃ〜
615132人目の素数さん:2010/04/29(木) 10:02:12
と暇人が申しております
616 ◆27Tn7FHaVY :2010/04/29(木) 13:43:01
ん?彼なの?
617132人目の素数さん:2010/04/29(木) 14:03:14
なんでコンピューター君の話題が出てくるんだ?
数学板に来てるのか?
618132人目の素数さん:2010/04/29(木) 15:56:33
>>613
数学的帰納法によって証明できます
619132人目の素数さん:2010/04/30(金) 00:17:24
t
620132人目の素数さん:2010/04/30(金) 04:10:30
e
621猫は雑魚 ◆ghclfYsc82 :2010/04/30(金) 09:41:19
ちょっと参考までに。



--------------------
73 名前:132人目の素数さん[] 投稿日:2007/12/23(日) 12:49:18
にちゃんねらーに個性はないが次の点は言える。

1.アンチ権力ではない。それほどの度胸は無い。
2.アンチ権力をからかって楽しむ。結果的に権力の思う壺。
3.弱いものと見ると寄ってたかっていじめぬく。学校でいじめられた
  腹いせ。匿名だからありがたい。
4.強いものには本質的に弱い。一見強気を挫くにみえるが、そんな
  恐ろしいことは到底できない。
5.政治に参加できるほど成長していない。選挙は棄権。
  なりゆきまかせ。
622132人目の素数さん:2010/04/30(金) 16:11:11
>>601
いかにも
愚かな私にそこんとこも含めて解説願えないかな?
623132人目の素数さん:2010/04/30(金) 17:13:05
どなたか以下の問題を教えてください。
Iを区間、fをI上の連続関数、<an>をIの点列、αをIの点とする。
lim[n→∞]an=αならば、lim[n→∞]f(an)=f(α)が成り立つ事を示せ。
624132人目の素数さん:2010/05/01(土) 00:11:29
t
625132人目の素数さん:2010/05/01(土) 02:59:01
>>623
> どなたか以下の問題を教えてください。

と頼む場合、マルチが逆効果となるのはご存知の通り。


> Iを区間、fをI上の連続関数、<an>をIの点列、αをIの点とする。
> lim[n→∞]an=αならば、lim[n→∞]f(an)=f(α)が成り立つ事を示せ。

この程度を示すこともできないとは

626132人目の素数さん:2010/05/01(土) 03:32:41
>>623
ε-δ式の収束定義における「任意」と「ある」の理解を試す良問
627132人目の素数さん:2010/05/01(土) 12:42:32
t
628132人目の素数さん:2010/05/01(土) 18:09:31
絶対値の問題で
|x-2|+|x-5|≦5

という式を 3つの場合分けをして解くのはわかり

x≧5の場合の共通範囲は5≦x≦6
2>xの場合の共通範囲は1≦x<2

まではわかるのですが2≦x<5の場合xが消えてしまうのでどうすればいいのかわかりません
解くと0≦2となります
どうすればいいのかと最終的な答えをお願いします
629132人目の素数さん:2010/05/01(土) 18:26:40
>>628
2≦x<5のとき不等式はどうなっている?
630132人目の素数さん:2010/05/01(土) 18:51:16
628ですが

lx-2l=x-2
lx-5l=-x+5
なので
x-2-x+5≦5
でしょうか
631132人目の素数さん:2010/05/01(土) 18:57:38
0≦2は任意のxについて成り立つ
632132人目の素数さん:2010/05/01(土) 19:00:19
ということはどのように書けばいいのでしょうか
2≦x<5の場合わけはどのように書けばいいのでしょうか
633132人目の素数さん:2010/05/01(土) 19:01:28
2≦x<5がそのまま残るだけだろ
634132人目の素数さん:2010/05/01(土) 19:04:38
不等式で、解く、というのは
その不等式が成り立つxの範囲を求めるということ
xが含まれてなければxに関係なく成り立ってるということ
場合分けしたのでxの範囲はあるけれど

あーわかりにくいわ
635132人目の素数さん:2010/05/01(土) 19:05:03
わかりました最終的の共通範囲は
1≦x≦6になりました
636132人目の素数さん:2010/05/01(土) 19:15:23
すいません。21個の数字のなかから6個を選ぶと、何通りできるでしょうか?
637132人目の素数さん:2010/05/01(土) 19:21:59
21個の数字はどの2つも相異なりますか?
選んだ数字は元に戻しますか?
順番に選び出しての順列ですか?一気に6個取り出しての組合せですか?
海は死にますか?山は死にますか?
638132人目の素数さん:2010/05/01(土) 19:24:09
普通異なるだろ
選ぶのに元に戻すってなんだよ?
選ぶんだから組合せだろ
知らん
639132人目の素数さん:2010/05/01(土) 19:25:52
重複順列とか重複組合せとか知らんのかこいつは。
640132人目の素数さん:2010/05/01(土) 19:26:02
>>637
海や山の生死の定義は?
641132人目の素数さん:2010/05/01(土) 19:27:08
さだまさしに聞いて
642132人目の素数さん:2010/05/01(土) 19:37:13
39枚のハズレと1枚のアタリで計40枚の「山札」があります。
ここにSカードを3枚加え合計43枚となっています。
ここから1枚ずつ引いて1カウントとし、
アタリを引いたらゲームは終わりです。
Sカードを引いた場合はさらに3枚引かなければなりません。
2回目のカウントSカードをで引いた場合は、
2回目のカウントのままさらに3枚引くことができます。
つまり「2回目のカウントで4枚のカードを引いた」ことになります。
Sカードを使って引いた3枚のうちの2枚を山札に戻してシャッフルします。
SカードでSカードを引いた場合はSカード優先でその他2枚を戻します
Sカードは1回のカウントで1度しか適応されず、
1回のカウントで2枚引いた場合1つの適応は次のカウントに回されます。
もしSカードでSカード2枚とハズレ1枚を引いてしまった場合ですが、
Sカードの適応は最終的に持っていることが条件のため、
1回のカウントで4枚以上引くことはありません
アタリは「絶対優先」であり、
SカードでSカード2枚とアタリの計3枚を引いたの場合はアタリを優先するため、
Sカード2枚を戻し終了となります。
643132人目の素数さん:2010/05/01(土) 19:39:25
>>642続きです..ex.
【1回目のカウント】
…40枚の山札から1枚引きます
…Sカードを引きました
…さらに3枚引きます
…Sカード1枚とハズレ2枚を引きました
…Sカード以外の2枚を戻します
【2回目のカウント】
…38枚の山札から1枚引きます
…Sカードを引きました
…1回目のカウントのSカードを適応します
…さらに3枚枚引きます
…ハズレ3枚でした
…そのうち2枚を戻します
【3回目のカウント】
…36枚の山札から1枚引きます
…ハズレでした
…2回目のカウントのSカードを適応します
…3枚引きます
…アタリが出ました
…終了

Q.
出来るだけ少ないカウントでアタリを引きたい場合、 Sカードを加えない40枚の山札か、
Sカード3枚を含めた43枚の山札、どちらの山札が有利でしょう?

という問題です。 43枚が有利だと思うのですが数値にして表せなくて困っています。
40枚は単純に1/40ですよね。 長文問題なのですが、解ける方いませんでしょうか?宜しくお願いします。
644132人目の素数さん:2010/05/01(土) 19:47:20
とりあえず s/適応/適用/g な
645132人目の素数さん:2010/05/01(土) 22:10:44
すいません…数学はさっぱりなんです
単純に40か43のどっちが有利か教えていただきたかったのですが
もしかして計算できない?ような問題なのでしょうか?
646132人目の素数さん:2010/05/01(土) 22:22:38
gradV=(∂v/∂x,∂v/∂y,∂v/∂z)

を球座標で書くとどうなりますか?
647132人目の素数さん:2010/05/01(土) 22:33:50
次のア、イ、ウに-.*.÷を一つずつ入れてしきを完成させなさい
8ア(-6)イ4ウ(-2)=5
全然わかりません…
648132人目の素数さん:2010/05/01(土) 22:40:20
3!通りすればいいんじゃね?
649132人目の素数さん:2010/05/01(土) 22:50:03
{Ф,{1},{2},{1,2}}を内包的に表せという問題がわからないんだが・・・
どなたかわかりますか?
650132人目の素数さん:2010/05/01(土) 22:55:47
>>649
{x|x⊂{1,2}} とか?
651647:2010/05/01(土) 22:57:08
>>648 とりあえず何通りかはやってみたのですがサッパリです。5になりません…
652132人目の素数さん:2010/05/01(土) 22:57:45
>>650
確かに!ありがとうございます。
653132人目の素数さん:2010/05/01(土) 23:05:02
>>647
8-(-6)÷4*(-2)
654647:2010/05/01(土) 23:12:33
>>653分数使うとは思いませんでした
ありがとうございました!
655132人目の素数さん:2010/05/02(日) 04:04:08
s
656132人目の素数さん:2010/05/02(日) 07:35:27
>>645
お前は何を言っているんだ
657132人目の素数さん:2010/05/02(日) 10:19:56
>>656
40か43どっちが有利かの答えを知りたかったんですけど
計算方法教えてあげるから自分でやれって趣旨なんですかね
658132人目の素数さん:2010/05/02(日) 10:39:45
>>657、どっちも3に比べりゃ無理いっぱい
659132人目の素数さん:2010/05/02(日) 11:01:26
>>658
お前は何を言っているんだ
660132人目の素数さん:2010/05/02(日) 12:38:26
拡大係数行列の基本変形を用いて連立方程式を解く問題なのですが
x,y,zの三元一次方程式で

1 -2 -3 -2
-2 3 18 15
2 -5 6 7

これを基本変形していくと

1 0  -27 -24
0 -1 12 11
0 0 0 0

となって一番下の行がすべて0になって解がだせないのですが
答えにはcを用いて
x = 27c -24
y = 12c -11
z = c
となっているのですがどのようにして求めているのでしょうか?
詳しく教えてもらえるとありがたいです
661132人目の素数さん:2010/05/02(日) 12:49:43
>>660
だからさ、行列の表記の仕方くらいテンプレ>>2読んでよ。
662132人目の素数さん:2010/05/02(日) 13:35:08
>>660
拡大係数行列のランクは2なので、x,y,zは一意的には求められない。

>x = 27c -24
>y = 12c -11
>z = c
>となっている

これで分らないなら、教科書を100遍読むしかなし。
663132人目の素数さん:2010/05/02(日) 14:16:50
宿題なので具体的に書くのははばかられるので一般化して書きますが,
たとえば{(x,y,z)∈R^3|f(x,y,z)=0}が
R^3の部分多様体であることを示すためにはまずどうしたらいいですかね。
fが具体的にわからないと解けないなら書きますが。
664132人目の素数さん:2010/05/02(日) 14:24:40
部分多様体の定義を満たしていることを示す。
端的にはこれしかないね。
665132人目の素数さん:2010/05/02(日) 15:09:06
>>657
回答なんですかね、ありがとうございますm(_ _)m
やっぱり43が有利ですよね?
666132人目の素数さん:2010/05/03(月) 04:04:04
t
667132人目の素数さん:2010/05/03(月) 05:09:00
x + y + z < 15
ただし、x,y,zは非負整数のとき、これを満たす解は何通りあるか?
よろしくお願い致します。
668132人目の素数さん:2010/05/03(月) 05:36:36
x + y + z = n (nは非負整数) の非負整数解 (x, y, z) の個数は
重複組合せ H[m, k] を用いて表すことが出来るね。
それの n = 0, ..., 14 の総和をとりたまえ。健闘を祈る。
669132人目の素数さん:2010/05/03(月) 06:00:00
a=14−x−y−z。
670132人目の素数さん:2010/05/03(月) 12:38:56
>>668
他に方法ないでしょうか?
あまりに泥臭い気がします

>>669
詳しくお願いします
671132人目の素数さん:2010/05/03(月) 12:57:55
>>670
>>668>>669をあわせると殆ど解答一歩手前の大きなヒントなんだけどね。
669のaはどんな数かをよく考えてみる。
そうすると、3変数の場合も4変数の場合も同じ扱いで解けるのだから・・・
672132人目の素数さん:2010/05/03(月) 14:53:08
21C6
673132人目の素数さん:2010/05/03(月) 15:00:21
なんだそれ
674132人目の素数さん:2010/05/03(月) 15:10:56
単純な21C6が答になる意外な問題、綺麗な問題、を問うている、
か?
675132人目の素数さん:2010/05/03(月) 15:18:57
(a^0+a^1+a^2+...+a^15)^3
676132人目の素数さん:2010/05/03(月) 17:17:34
>>671
どういうことでしょうか?
a+x+y+z=15?
677132人目の素数さん:2010/05/03(月) 17:21:58
ああそうか
a+x+y+z=14

(14+4-1)C(4-1)ですか
678132人目の素数さん:2010/05/03(月) 17:26:53
679132人目の素数さん:2010/05/03(月) 20:48:38
二つのnxnの行列
(a11…a1n)
A=(:  :)
(an1…ann)

(d11…d1n)
D=(:  :)
(dn1…dnn)
であるとき

det(AD)
(DA)
答えはあるんだが、導入方法が分かりません
680132人目の素数さん:2010/05/03(月) 21:47:32
数式と質問ははっきり相手に伝わるように
681neetubot:2010/05/04(火) 00:18:54
682132人目の素数さん:2010/05/04(火) 02:27:58
f=(1+x+x^2+...)^3=(1-x)^-3
d^nf/n!=(n+2)!/n!=(n+2)(n+1)
16*17
683132人目の素数さん:2010/05/04(火) 02:59:03
f=(1+x+x^2+...)^3=(1-x)^-3
d^nf/n!=(n+2)!/2*n!=(n+2)(n+1)/2
17*16/2=17*8
684132人目の素数さん:2010/05/04(火) 04:05:15
t
685132人目の素数さん:2010/05/04(火) 12:18:37
ベクトルの問題をやっているのですがいきづまりました
問題は以下です

 次の平行な 2 直線を含む平面の方程式を求めよ
 (x-1)/3 = (y+2)/4 = (z+3)/(-5)
 (x+1)/3 = y/4 = (z-1)/(-5)

答えは 13x - y + 7z = -6 なのですが、解き方がどうしてもわかりません
求める平面を ax + by + cz = d として
与式の方向ベクトル (3, 4, -5) と
与式どうしの位置ベクトルの差 (2, -2, -4) の成分をそれぞれ
求める平面の式に代入して
a + 6b - z = 0 が得られ、これは回答と一致するんですが
ここからどうすればいいかわかりません お願いします
686685:2010/05/04(火) 12:22:07
すこし訂正します

>a + 6b - z = 0 が得られ
a + 6b - c = 0 でした
回答では a = 13, b = -1, c = 7 で
これを代入すると 13 - 6 - 7 = 0 なのであってます
687132人目の素数さん:2010/05/04(火) 12:29:29
>>685
どうしてもエレガントな方法が浮かばないなら
各直線上の点を具体的にやたらめったら列挙して
平面の方程式に突っ込みまくるだけで求まるよ
688132人目の素数さん:2010/05/04(火) 12:43:13
なるほど!
それは知りませんでした ありがとうございます。。
689132人目の素数さん:2010/05/04(火) 12:57:22
t
690132人目の素数さん:2010/05/04(火) 14:15:35
すいません…教えてください
ttp://izumi-math.jp/S_Shimizu/integ_abc/print1.htm

距離の微分は速さ
ということをわかってはいるのですが
上記URLの一番上の時に
微分をしても速さが出てこないと思うのですが

これが何故、というところを理解できず
691132人目の素数さん:2010/05/04(火) 14:29:04
>>690
一番上のって横軸が時間で縦軸が速さ、
それらの面積で距離を表してるグラフのこと?

これはある時間での速さ(正確には速度)を表す関数(のグラフ)なんだから
この関数を微分して出てくるのは速さじゃなくて加速度だ。
縦軸に距離をとったグラフなら、そのグラフで表される距離についての
関数の時間での微分は速さになる。
692132人目の素数さん:2010/05/04(火) 14:29:54
時間tの関数としてtで微分。
一番上の例は定速の例で時刻tまでに進んだ距離は100t
微分して定数100がでる。
693132人目の素数さん:2010/05/04(火) 14:39:01
>>691-692
なるほど。とんだカン違いをしていました。
確かに、出るのは加速度ですね。等速なら加速度は0で納得です。

ありがとうございます!
694132人目の素数さん:2010/05/05(水) 10:12:27
2次関数の答えを見てどっちにまとめたらいいの?と思ったので質問します。
--
問題

関数y=x^2+2(aー1)x(−1≦x≦1)の最大値Mをaの式で表せ。

解答

[1]1−a<0 すなわち a>1のとき M=f(1)=2a−1
[2]1−a=0 すなわち a=1のとき M=f(−1)=f(1)=1
[3]1−a>0 すなわち a<1のとき M=f(^−1)=−2a+3

以上から

M=2aー1 (a≧1)
M=−2a+3(a<1)
[1]と[2]をまとめた
--
と解答にはあるのですが、疑問が4点。

疑問1:なぜ1と2をまとめた答え方をするのか?
疑問2:2と3をまとめた答え方は不正解なのか?
疑問3:まとめた答え方でないと不正解なのか?
疑問4:1,2,3とまとめると1と3が矛盾してしまう?
のでだめなのはなんとなくわかりますが・・・
全体的にまとめに関してゆくわかってないです。

疑問が多いのですが、ご指南、ご指摘のほどよろしくお願いします。
695132人目の素数さん:2010/05/05(水) 10:17:23
次の命題は成り立つか?

Xを位相空間、Yをハウスドルフ空間、f:x→yを連続写像とする。
AがXのコンパクト集合ならば、f(A)はYの閉集合である。
696132人目の素数さん:2010/05/05(水) 10:19:47
訂正
次の命題は成り立つか?

Xを位相空間、Yをハウスドルフ空間、f:X→Yを連続写像とする。
AがXのコンパクト集合ならば、f(A)はYの閉集合である。
697132人目の素数さん:2010/05/05(水) 10:45:17
コンパクトの連続像はコンパクト
ハウスドルフ空間のコンパクト集合は閉集合
698132人目の素数さん:2010/05/05(水) 11:03:16
>>694
>疑問1:なぜ1と2をまとめた答え方をするのか?
出来るだけ簡潔な答えを書きたいから
>疑問2:2と3をまとめた答え方は不正解なのか?
別に不正解ではない。
しかし、1,2,3を纏めると不正解(どうやってまとめる?)
だからまとめるなら「1と2」か、「2と3」
>疑問3:まとめた答え方でないと不正解なのか?
一応そんな事はないけど学校だと△になるかも。
>疑問4:1,2,3とまとめると1と3が矛盾してしまう?
グラフ描いてみそ
699132人目の素数さん:2010/05/05(水) 11:08:57
>>694
疑問1:式3つより式2つの方が簡潔だから。
疑問2:2と3の方をまとめても正解です。
疑問3:まとめなくても正解です。
疑問4:[1]を満たす全てのaに対して[3]の書き方で表すことができるならば[1]は[3]にまとめることができます。
     しかし、今回の場合はそのように書けないのでそもそも[1]は[3]にまとめることができません。
700132人目の素数さん:2010/05/06(木) 18:41:27
CP^2の中の曲面M={[x,y,z]∈CP^2:z(y^2)=x^3+x^3+(z^2)x+z^3}は何か説明せよ。
という問題で何をしていいかわかりません。
まずどこから手をつけたらいいでしょうか。
701132人目の素数さん:2010/05/06(木) 19:47:02
書いたら見直しするという小学生でもできることから。
702132人目の素数さん:2010/05/06(木) 23:59:26
t
703132人目の素数さん:2010/05/07(金) 08:30:39
行列 A=[[0,1],[-1,-1]]の固有値は±ωでまたA^3=Eを満たすので
120度の回転行列Rとなんらかの関係があると思うのですが具体的には
AとRはどういう関係にあるのでしょうか?
704132人目の素数さん:2010/05/07(金) 08:52:20

たまに"t"だけ書き込んでる屑はなんだ?
705132人目の素数さん:2010/05/07(金) 10:37:13
>>704
age荒らし
706132人目の素数さん:2010/05/07(金) 11:02:21
>>703
> 固有値はωとω'
707132人目の素数さん:2010/05/07(金) 11:12:35
>>703
例えば S=[[-2/ √3,-1/√3],[0,1]] として SAS^(-1) を計算してみて
708132人目の素数さん:2010/05/07(金) 13:12:57
>>703
行列 A=[[0,1],[-1,-1]]
S=[[-2/ √3,-1/√3],[0,1]] として SAS^(-1)だと
ちょっと計算が合わないのは列と行が逆になっているのかなぁ?下のルールの表記のつもりですが、、
>>2
> ●行列  M[i,j], I[i,j]=δ_[i,j]  M=[[M[1,1],M[2,1],...],[M[1,2],M[2,2],...],...], I=[[1,0,0,...],[0,1,0,...],...]

S=[[0,1],[ -√3/2,-1/2]] として SAS^(-1)を計算するとR(120度)=[[,-1/2, √3/2],[ -√3/2,-1/2]]
709132人目の素数さん:2010/05/07(金) 13:34:34
>>707
>>708
> 列と行が逆になっている
みたいね。
710132人目の素数さん:2010/05/07(金) 16:13:13
質問なんだが
p、q、rを命題として

(p∧¬q)∨(¬p∧q)∨r ≡ (p∨q∨r)∧(¬p∨¬q∨r)

この論理同値はどうして成り立つの?
711132人目の素数さん:2010/05/07(金) 17:00:08
右辺
≡(p∨q∨r)∧(¬p∨¬q∨r)
≡{(p∨q)∨r}∧{(¬p∨¬q)∨r}
≡{(p∨q)∧(¬p∨¬q)}∨r
≡{p∧(¬p∨¬q)}∨{q∧(¬p∨¬q)}∨r
≡{(p∧¬p)∨(p∧¬q)}∨{(q∧¬p)∨(q∧¬q)}∨r
≡{φ∨(p∧¬q)}∨{(q∧¬p)∨φ}∨r
≡(p∧¬q)∨(q∧¬p)∨r
≡左辺
712132人目の素数さん:2010/05/07(金) 23:51:15
>>711
はぁ・・・感心するわ
どうもありがとう
713132人目の素数さん:2010/05/08(土) 00:05:05
感心している場合じゃないっしょ。
分配則をドンドコ適用しているだけなんだから。
714132人目の素数さん:2010/05/08(土) 01:59:18
(p∧¬q)∨(¬p∧q)∨r ≡ (p∨q∨r)∧(¬p∨¬q∨r)
(p*(1-q)+((1-p)*q)+r ≡ (p+q+r)*((1-p)+(1-q)+r)
p-pq+q-pq+r 
=p+q-pq+r
≡ (p+q+r)*(1-p+1-q+r)
=0+0+p-pq+q-qp+pr+qr+r+r-rp+r-rq
=p+q-pq+r
715132人目の素数さん:2010/05/08(土) 07:27:52
問、複素数z=x+iy(x,yは実数、iは複素数)の平方根は?
ただし表示には実数の平方根のみを用いる。



問題はこれだけです・・・なぜ決まった実数を言わないのに表示できるのかなぞです・・・
716132人目の素数さん:2010/05/08(土) 11:34:48
書けるようにはなったかね。
717132人目の素数さん:2010/05/08(土) 11:39:29
>>715
安直にやるなら
x+yi=(p+qi)^2
とおいて、右辺を展開して係数比較して、とゴリゴリ計算すればできる。
あるいは極形式に変換して、r(cosθ+isinθ)と書ける複素数の平方根は
±√r(cos(θ/2)+isin(θ/2))
であることを使って、半角の公式にぶち込む手もあるか。

なんにせよ答えは汚い。
±(√((√(x^2+y^2)+x)/2) + √((√(x^2+y^2)-x)/2)i)
かな。
718132人目の素数さん:2010/05/08(土) 12:04:41
s
719132人目の素数さん:2010/05/08(土) 13:07:39
>>718
荒らすなボケ
720132人目の素数さん:2010/05/08(土) 14:02:54
ksks
721132人目の素数さん:2010/05/08(土) 14:10:51
>>715
> iは複素数
虚数単位じゃないのか?
722132人目の素数さん:2010/05/08(土) 16:07:37
>>721
指摘通り「虚数単位」でした

ちょっと違いを調べてきます・
723132人目の素数さん:2010/05/08(土) 16:14:16
いいよ、調べなくて。
724132人目の素数さん:2010/05/08(土) 16:30:52
いや調べたほうがいいだろ
725132人目の素数さん:2010/05/08(土) 16:33:38
うん、調べても構わないけど、そのレベルだとここで回答を得ても
理解できないんじゃないかと思う。
726132人目の素数さん:2010/05/08(土) 16:36:25
どうせ調べるならクロネッカーの定理くらいまでは調べておけ。
727132人目の素数さん:2010/05/08(土) 16:47:34
回答されたものが理解できなかったら
調べなくてもいいってわけじゃないんだからさ
728132人目の素数さん:2010/05/09(日) 00:07:03
g^2=0
729132人目の素数さん:2010/05/09(日) 01:28:35
はじめまして。

ところで、半径rの円の内部に含まれる格子点の数についてですが、
ガウス記号などを使って、不等式ではなく、等式の形で表す事は
可能でしょうか?
730729:2010/05/09(日) 01:30:51
何故そんなことを質問するかというと、昔、高校の時の数学の塾の最初
の方の講義で、そのような問題が扱われた記憶があるのです。
それでネットで調べてみたのですが、不等式はあれども、等式は
見つからないので、本当はどうだったのか知りたいのです。
塾は、京都の「Mat」という塾でした。
731132人目の素数さん:2010/05/09(日) 03:06:09
[π r^2]
732132人目の素数さん:2010/05/09(日) 03:18:30
733729:2010/05/09(日) 03:53:05
>>732
解説をお願いしたいのですが、正確な値は、Σ記号で和を取った形では
表せているのだと思うんですが、Σを使わずになおかつ等号でずばりと
表記することは出来てないんですか?
734729:2010/05/09(日) 03:59:03
>>731
確認はしていませんが、それは違うんじゃないでしょうか?
735132人目の素数さん:2010/05/09(日) 21:04:07
離散距離空間において可算と可分は同値になるでしょうか?
736132人目の素数さん:2010/05/09(日) 21:09:53
(X,d):距離空間とする。
このとき、
X:コンパクト・局所弧状連結
ならば、
∀ε>0 ∃δ>0 s.t. d(x,y)<δ (x,y∈X) ⇒ ∃f:[0,1]→X|連続 s.t. f(0)=x,f(1)=y,diam(f([0,1]))<ε
となることを示せ。

うまく示せません。よろしくお願いします。
737132人目の素数さん:2010/05/09(日) 21:31:58
738132人目の素数さん:2010/05/09(日) 23:36:23
>>736
εとxを先に固定してから、
> ∃δ>0 s.t. d(x,y)<δ (y∈X) ⇒ ∃f:[0,1]→X|連続 s.t. f(0)=x,f(1)=y,diam(f([0,1]))<ε
を満たすδ(x)を構成し、Xを各点周りのδ(x)近傍で覆う。
コンパクト性を使ってその中から有限被覆を選んで、
距離δ以下の二点が常に同一の被覆に入るようにδを適当に小さくとればできあがり。
739132人目の素数さん:2010/05/09(日) 23:50:52
可算公理満たすなら可算っていえますか?第一、二どっちでもいいです
740736:2010/05/10(月) 09:08:23
>>738
回答ありがとうございます。
質問なのですが、2行目の
「> ∃δ>0 s.t. d(x,y)<δ (y∈X) ・・・」
は、
「δ(x)> ∃δ>0 s.t. d(x,y)<δ (y∈X) ・・・」
ということでしょうか?
741132人目の素数さん:2010/05/10(月) 14:50:30
-2
742132人目の素数さん:2010/05/10(月) 15:10:53
>>736
それ何の本の問題?
743132人目の素数さん:2010/05/10(月) 16:20:37
>>740
> 「δ(x)> ∃δ>0 s.t. d(x,y)<δ (y∈X) ・・・」
> ということでしょうか?


何がいいたいの?
744132人目の素数さん:2010/05/10(月) 19:00:29
∫[|z|=1] |z-1||dz| の値を求めよ.

∫[|z|=r] Re(z)dz の値を線積分の定義を用いて計算せよ.ただしRe(z)はzの実部.


745132人目の素数さん:2010/05/10(月) 19:33:21
>>744
z=exp(iθ), z=r exp(iθ)と置換
746132人目の素数さん:2010/05/10(月) 21:40:19
>>739
Rは可算か、Rはそれら可算公理を満たすか、以上2点を考えてみよ。
747132人目の素数さん:2010/05/11(火) 00:00:56
#Aut(Z/(p^a)Z),a:自然数,p:素数
を求めるにはどうしたらいいですかね。
748132人目の素数さん:2010/05/11(火) 00:09:05
その前に、Aut(Z/(p^a)Z)の元を幾つか示してみな。
749132人目の素数さん:2010/05/11(火) 00:11:37
調子に乗るな雑魚
750132人目の素数さん:2010/05/11(火) 00:13:36
と、群論に挫折したアホが喚いています
751132人目の素数さん:2010/05/11(火) 00:18:32
>>735ですが解決しました
752132人目の素数さん:2010/05/11(火) 03:36:30
2を法とする加法、乗法が定義された位数2の体F_2={0,1}の上の
3次の既約多項式x^3+x+1により生成される拡大体は

{0,1,x,x+1,x^2,x^2+1,x^2+x,x^2+x+1}

でよろしいんでしょうか?
753132人目の素数さん:2010/05/11(火) 03:55:59
F(x)=log(1+x^2)のF(n)(0)を求めよって問題ですnはn回微分ってことです
ライプニッツの公式使わないと解けないのですか
帰納法でやろうとしたのですが、規則性がわからないという。
ライプニッツは使い方が良くわかりません
754132人目の素数さん:2010/05/11(火) 07:56:49
the word vector literally means carrier

の意味教えてください・・
線形代数の本の文章なのですが・・
「ベクトルという言葉は、文字通り運び屋を意味する」


と訳したのですが意味がまったくわからなくて・・

the dimension n can have any value 1 2 3・・・
the 1-vectors are the simplest case of vectors,
though perhaps a rather trivial one
「次元nは1、2,3・・などの値をとり、1-vectorはもっとも簡単な場合のベクトルである」
まではわかるのですが、rather以下がわかりません・・教えてください
a mechanical system of two particles requires 2×3=6 coordinates for its discription
to discribe it completely, we shall need their velocities as coordinates (because the equations of motion are of the sevond order)
2つの粒子からなる力学系は、記述するためには6成分必要で、完全に記述するためには、その6成分のほかに速度も必要になるだろう(なぜならば運動方程式は2階の微分方程式だからである)
とあるのですが()の中が自信ありません。運動方程式が2階の微分方程式ならば速度が必要になるのですか・・?
755132人目の素数さん:2010/05/11(火) 08:08:43
>>754の追加です

(X_1,X_2・・・・X_n)=(Y_1,Y_2・・・・Y_n)・・・★
というのは、i=1、2、・・・・・nに対して
X_i=Y_i
のときに限る
とあってこの後に
thus any equation betweem n-vectors is equivalent to n scalar equations: we express this fact by saying that we may equate components in any equation★
ゆえにn-vectorの間の方程式はn個のスカラーの方程式と同等である。この事実を、私たちはどんな★のようなベクトルの等式においても成分を等式化する、ということで表現するのである

と訳したのですが、:以下がよくわからないです・・教えてください・・
756132人目の素数さん:2010/05/11(火) 10:07:35
>>748
f(x)≡x,x+1,x+2,…,x+(p^a)-1 (mod p^a)
の形のものしか思いつきません…
757132人目の素数さん:2010/05/11(火) 10:36:51
>>754
> literally means carrier
文字通りの意味は運び屋である(が、数学用語としては違う意味で使う)というニュアンスか。

>運動方程式が2階の微分方程式ならば速度が必要になるのですか・・?
x''=-x のような微分方程式の解は x(0) と x'(0) の両方を与えないと一意に決まらない。また、この方程式を x'=v, v'=-x のように変数を倍に増やした連立一階微分方程式として解くこともある。
758132人目の素数さん:2010/05/11(火) 12:02:56
exp(jA)*exp(-jA)

の答えお願いします。
759132人目の素数さん:2010/05/11(火) 12:33:37
1
760132人目の素数さん:2010/05/11(火) 15:11:17
生活一般的で我々が使う「ベクタ」は(英語では何かの)運び屋を意味します。
1次元ベクタはベクタ空間のもっとも単純な場合です。
761132人目の素数さん:2010/05/11(火) 18:22:40
>>757
>>760
ありがとうございます
授業で返信できませんでした

vectorという単語が一般的に使う意味と数学で使う意味ではちがうということですね
rather以下の意味を教えていただけませんか・・?

ratherが絡むとどうもわからなくて・・・

>>755もお願いします・・・・
762132人目の素数さん:2010/05/11(火) 18:23:13
>>753
F(x) = log((1+xi)(1-xi)) = log(1+xi)+log(1-xi) = 2Re(log(1+xi)) であるから、
g(x) = log(1+xi)の高階微分を求めればよい。これは簡単で、
g^(n)(x) = (-1)^(n-1) i^n (n-1)! / (1+xi)^n. F^(n)(0) = 2Re(g^(n)(0))だから、
nが奇数のときはゼロ、偶数のときは2 (-1)^(n/2 -1) (n-1)!.
763132人目の素数さん:2010/05/11(火) 19:41:06
一般的に「ベクタ」の単語は(何かを)キャリーアしていることを意味します。
次元として使うNは、普通は1,2,3(多分自然数の意味)で・・・・
1次元ベクタ集合(1次元のベクタ空間)は、ベクタを集めてまとめた集合(1次元の点列とかのベクタ空間)のもっとも単純な場合です。

そのratherは「より正確に言えば」のフレーズでしょうしperhapsとかの修辞は「天下り的に言えば」とかでしょうか。
だけれでもよくあるたとえで正しく言えば、
2つの粒子(微少量ベクタ)によって成立するあるメカニズムは、2x3=6等式を満足する座標系を使うと
この微少量ベクタ集合の説明を完全に(そのベクトル空間の公理と数式で)記述できます。
そこで私達はその各ベクタの「いわゆる速度」を、その座標の系(つまりベクトル空間)の設定と同時に設定する必要がある。
(運動の方程式系は一般的にsevond orderなため)

外国の教科書は日本人と違って著者のクセも少なくて読みやすいってことですか。
764132人目の素数さん:2010/05/11(火) 19:42:35
>>736
それの元ネタ知ってる?
765132人目の素数さん:2010/05/11(火) 19:58:07
>>763は質問者をおちょくって何か楽しいことがあるんだろか…
766132人目の素数さん:2010/05/11(火) 20:02:10
>>763は質問者をおちょくって何か楽しいことがあるんだろか…
767132人目の素数さん:2010/05/11(火) 20:02:14
>>763
>>765
〜〜〜though perhaps a rather trivial one

の部分を
1−ベクトルはもっとも単純なベクトルの場合ですが
おそらくかなり平凡なベクトルです
と訳してはだめですか?
ratherをかなり
trivialを平凡な、ありふれた
oneはベクトル
と考えました・・・

>>755もお願いします・・・
768132人目の素数さん:2010/05/11(火) 20:04:50
数列{a[n]}に対してb[n]=(a1+a2+…+a[n])/nとおくとき{b[n]}が等差数列ならば{a[n]}も等差数列であることを示せ。
この問題はb[n]の公差をdとしてa[n+1]-a[n]=2dになると思うんですけど、2dになりません。2dになるまでの過程を教えてほしいです。
769132人目の素数さん:2010/05/11(火) 20:09:01
何で2dになると思ったの?
770132人目の素数さん:2010/05/11(火) 20:11:14
>>769問題のヒントに書いてあったので…
771132人目の素数さん:2010/05/11(火) 20:12:25
>>769
ここにそうかかれたからだろ
http://science6.2ch.net/test/read.cgi/math/1272681059/615
772132人目の素数さん:2010/05/11(火) 20:17:59
>>768

409 :132人目の素数さん:2010/05/09(日) 14:24:26
数列{a[n]}に対してb[n]=(a1+a2+…+a[n])/nとおくとき{a[n]}が等差数列ならば{b[n]}も等差数列であることを示せ。
この問題がわかりません。教えてください。

605 :132人目の素数さん:2010/05/11(火) 18:12:31
数列{a[n]}に対してb[n]=(a1+a2+…+a[n])/nとおくとき{b[n]}が等差数列ならば{a[n]}も等差数列であることを示せ。
これの証明の仕方がわかりません。教えてください。
773132人目の素数さん:2010/05/11(火) 20:19:12
そのヒント、というか答えそのものを目にしてなぜわからないのだ
774132人目の素数さん:2010/05/11(火) 20:53:04
すみません>>755に追加で質問です

numerical quantities such as temperatures, lengths, speeds, weights are called scalars,because they can be read off on a scale
温度や速さ、質量という数に関する量はスカラーと呼ばれる、

becauseの後がよくわからないです・・・
read offなどが調べてもわからなくて・・・

お願いします
775132人目の素数さん:2010/05/11(火) 20:53:41
大学一年が質問させていたきます

1/(n+1)<log(n+1)-logn<1/が成り立つことをを証明しなさい

関数y=1/xを[n.n+1]で積分すると log(n+1)-logn が出るのですが、それをどのように利用すればいいのかわかりません
よろしくおねがいします
 
776132人目の素数さん:2010/05/11(火) 20:55:25
すいません 修正です

大学一年が質問させていたきます

1/(n+1)<log(n+1)-logn<1/nが成り立つことをを証明しなさい

関数y=1/xを[n.n+1]で積分すると log(n+1)-logn が出るのですが、それをどのように利用すればいいのかわかりません

よろしくおねがいします
 
777132人目の素数さん:2010/05/11(火) 20:58:28
>>776
y=1/x のグラフを描いてみる。
778132人目の素数さん:2010/05/11(火) 21:21:54
>>774
計量器(scale)で読み取ることができるから、スカラー(scalar)と呼ばれる。
779132人目の素数さん:2010/05/11(火) 21:23:19
>>776
階段関数でサンドイッチしてるだけだろ
780132人目の素数さん:2010/05/11(火) 21:25:09
ベクトルなる単語は、一般的な字義ではキャリーアを意味します。

>the 1-vectors are the simplest case of vectors, though perhaps a rather trivial one
で一つか。次の文と繋がっているように見えるけど。

1次元キャリーア集合はキャリーアの集合のもっとも単純な場合であり、しかしながら多分、
一番よく使うありふれたそれ(集合・空間・公理・体系つまりキャリーア空間)です。

それゆえ、えぬ次元のキャリーア集合空間の2つの等式は、えぬ・スケイラ・方程式の関係(いわゆる有限えぬ次元多項式で
X[n] = Y[n]の等式関係)とまったく同じことです(be equivalent)。
このequivalentについて私たちは上の事実(つまり定義)をよく「各等式は各キャリーアの構成成分が同一であるとみなしてよい(といえる)」のように言いまわして表現します。

もしコピペでなく手打ちならピリオドや大文字・複数形とかちゃんと写さないとまったく違う意味になるよ。
781132人目の素数さん:2010/05/11(火) 21:32:24
で、一体どの教科書を使ってゼミしてるんだ?
782132人目の素数さん:2010/05/11(火) 21:38:22
>>765
機械翻訳(しかも、相当能力が低い)の結果を並べただけのように見える。
783132人目の素数さん:2010/05/11(火) 21:42:42
>>777 >>776
グラフを書いて、与式の意味をだいたい理解できましたが

それをうまく文章やら数式やらでまとめれません
784132人目の素数さん:2010/05/11(火) 21:46:35
>>783
n<x<n+1において 1/(n+1)<1/x<1/n 。
nからn+1まで積分すれば
1/(n+1)≦∫_[n,n+1](1/x)dx≦1/n
785132人目の素数さん:2010/05/11(火) 22:00:42
>>778
read offは読み取るという意味でしょうか?

温度や速さは計量器で読み取られるからスカラーである
でしょうか?
>>780
The order of the components in a vector is essential. For example, (3 7 -1)≠(7 3 -1), and generally,two vectors are equal:
(X_1・・・X_n)=(Y_1・・・Y_n)・・・★
if and only if the corresponding components are equal:
X_i=Y_i for i=1,2,3・・・
thus any equation betweem n-vectors is equivalent to n scalar equations: we express this fact by saying that we may equate components in any equation★

これで全部です・・
>>equivalentについて私たちは上の事実(つまり定義)をよく「各等式は各キャリーアの構成成分が同一であるとみなしてよい(といえる)」のように言いまわして表現します。


if and only if the corresponding components are equal:
X_i=Y_i for i=1,2,3・・・
↑この部分で「キャリーアの構成成分が同一である」と言っていて、最後の文章で再び
「各等式は各キャリーアの構成成分が同一であるとみなしてよい」
と繰り返すのに違和感があるのですが・・
we express this fact by saying that we may equate components in any equation★

この部分詳しくお願いします

ちなみに
Elements of Linear Algebra
という教科書です

786132人目の素数さん:2010/05/11(火) 22:09:44
any equations
any equation
は違うのか?
とかそいうところが分かってないと教えてもらっても理解出来ないと思うよ。
787132人目の素数さん:2010/05/11(火) 22:17:38
>>785
> ↑この部分で「キャリーアの構成成分が同一である」と言っていて、最後の文章で再び
> 「各等式は各キャリーアの構成成分が同一であるとみなしてよい」
> と繰り返すのに違和感があるのですが・・

繰り返しではない。
前半は2つのベクトルの相等の定義、
後半では、その定義に基づきn次元ベクトルの間に成立する方程式は対応する成分同士を等しいとおいた
n個のスカラー方程式に同値だと言っている。

788132人目の素数さん:2010/05/11(火) 22:26:25
>>785
英語の試験かなんかじゃないなら、相当どうでもいいことに拘ってるだけに見える。
それと、>>763,>>780はスルーしたほうがいい。
789132人目の素数さん:2010/05/11(火) 22:29:30
英語wikiのどこかの記事と大して変わらないよね。
この程度の文で理解できないなら高校1年程度の英語で、英作文問題や長文すらも読めないのと同じだから英語なんか読んで無理しないほうがいいよ。
790132人目の素数さん:2010/05/11(火) 22:34:40
>>788
荒らすなチンカス
791132人目の素数さん:2010/05/11(火) 22:51:00
>>790
>>763,>>780が荒し
792132人目の素数さん:2010/05/11(火) 22:58:40
>>787
ありがとうございます

参考にして頑張ります
793132人目の素数さん:2010/05/11(火) 23:31:56
aを定数とする -1≦x≦1における2次関数y=x^2-2ax-5について
最大値M最大値Nをaの式で表せ、またそのときのxの値を求めよ

という問題で
軸が定義域内の場合の2種類の場合わけについて質問なんですが
解答では
(@)
-1≦a<0
これは最大値が必ずx=1の場合で限定されるのですが

(A)
0≦a<1
の場合a=0だったときには最大値はxが-1,1両方最大値になるのですが解答では
x=-1の場合としか書いていません。
これはどういうことなんでしょうか
794132人目の素数さん:2010/05/11(火) 23:49:08
x,yがx>0,y>0,x+y=1を満たすとき
1/xy とりうる値の最小値を求めよ

という問題なんですが、どう取り掛かっていいかわかりません。
どなたか教えていただけないでしょうか
795132人目の素数さん:2010/05/11(火) 23:51:38
あるベクターの(数学では成分とも言うところの)構成要素間のオーダーは、そのベクターの本質的なことを現す。
たとえば成分では[3,7,-1] != [7,3,-1] であり、一般的に、2つのベクターは次のときイーコールである(定義とも)。
(X_1・・・X_n)=(Y_1・・・Y_n)・・・★

(つまり2つのベクターの成分の秩序[3,7,-1]=[3,7,-1]が同じとき)

さらにこのときに限り、調和(一致)している2つのベクターで全ての成分は次のときイーコールという(定義とも)
X_i=Y_i for i=1,2,3・・・

(つまり一次独立の⇔の話しと同じで、集合についての論理学による定義方法と同じとも言えなくはないけど)

それゆえ、えぬ次元ベクター集合の2つの空間間の同等関係は、えぬ次スケイラー式・えぬ次方程式の同等性による評価とまったく同じです。
(いわゆる2つのベクターの相関関係で、通常はえぬ次代数多項式poly[z]=0が2つで、polyX[1]=polyY[1], polyX[2]=polyY[2],...polyX[n]=polyY[n]の話し)

私たちはこの評価方法を(一次独立定義と多項式同等性評価の類似点を同一視して)
「ベクター集合間の同等関係においては、(equivalentであったなら)その2つのベクターは成分が同一であるとみなしてよい(といえる)」の言い回しで表現します。

自分が知ってるカタカナや数学用語を使わないと英語の翻訳文が理解できないというなら、英文を自分で辞書で翻訳したりまたは英語で直に読んでも何を書いてあるのか理解できないと思いますよ。
多少補足しておきましたけど、英語wikiを翻訳して読んだほうが理解も早いかなって感じもしなくはないです。
796132人目の素数さん:2010/05/11(火) 23:55:55
>>795
酷すぎる。
こんなのは訳でもなんでもない、勝手なお話。
797132人目の素数さん:2010/05/12(水) 00:01:22
>>796
どのあたりが?
798132人目の素数さん:2010/05/12(水) 00:01:43
>>794
1/xyが最小になるということはxyが最大になること。また条件よりy=1-xとかける
そうすると
xy=x(1-x)となりx=y=1/2の時xyが最大で1/4
よって求めるxyの最少値は4

だと思います。違ったらすいません
799132人目の素数さん:2010/05/12(水) 00:07:14
>>798
なるほど一文字消去する発想がでなかった
ありがとうございました。
800132人目の素数さん:2010/05/12(水) 00:16:25
あ、打ち間違えた
1/xyでした。>>798のよって〜のところ
すいません
801132人目の素数さん:2010/05/12(水) 00:36:34
>>791
荒らすなチンカス
802132人目の素数さん:2010/05/12(水) 00:42:30
>>785
The order of the components in a vector is essential. For example, (3 7 -1)≠(7 3 -1), and generally,two vectors are equal:
(X_1・・・X_n)=(Y_1・・・Y_n)・・・★
if and only if the corresponding components are equal:
X_i=Y_i for i=1,2,3・・・
thus any equation betweem n-vectors is equivalent to n scalar equations: we express this fact by saying that we may equate components in any equation★

ベクトルにおいては要素の並び順が本質である。例えば、(3 7 -1)≠(7 3 -1)であり、
一般に2つのベクトルが等しい:(X_1・・・X_n)=(Y_1・・・Y_n)・・・★ とは
対応する要素が等しい:X_i=Y_i for i=1,2,3・・・ときをいう。
かくして、n-ベクトルの間の方程式はn個のスカラー方程式に同値である:このことを方程式★において要素を同一視する、ということにする。
803132人目の素数さん:2010/05/12(水) 00:43:03
>>797
全部
804132人目の素数さん:2010/05/12(水) 00:48:42
後出しジャンケン乙
805132人目の素数さん:2010/05/12(水) 00:56:34
確率の問題です。
平面上で点Oを中心とする半径Rの円内に、N個(N>2)の点を無作為にとる。N個の点の中で、点Oから最も近い点までの距離をsとする。
sの確率密度関数f(s)を、任意のr(0<r<=R)に対してsが0<=s<rを満たす確率がF(r)=∫[0→r]f(s)dsで与えられるものとして定義する。
このとき関数f(s)を求めよ。

確率密度関数だから、0からRまでf(s)をsについて積分したら1というのはわかるんですが、
f(s)の具体的な求め方が全く思い浮かびません。
1つだけsのところに落ちて、それ以外は全部sの外側に落ちる。という方針も考えたんですが、なんかしっくりきません。

どなたか本当によろしくお願いします。
806132人目の素数さん:2010/05/12(水) 01:09:24
>>796,803
この離散とか一次独立の話しとかは、フレーズは知っていても2流以下(しかも専攻外だと数学科でも)だとぜんぜん理解してないの大学生が多いでしょ。
このスレでも連続とか無限とを説明できる数学科学生はまったくいないようだし、多分その概念をちゃんと理解できてないってことだと思うよ。
ここの回答者でもそういう奴が多いでしょ。
807132人目の素数さん:2010/05/12(水) 01:19:29
なるほどこれが馬鹿か。
808132人目の素数さん:2010/05/12(水) 01:20:46
>>789,>>795,>>806
コンピューター君こそ数学無理しないほうがいいよwww
809132人目の素数さん:2010/05/12(水) 01:32:34
大体の文意が取れているのに正確な訳に拘るあまり
自縄自縛に陥っている元質問者は確かに哀れだが
>>795のような訳にも何にもなってない出鱈目はないわ。
質問者がかわいそうだ。
810132人目の素数さん:2010/05/12(水) 01:32:48
>>805
f(s)=2s/R^2
多分間違ってる・・・
811132人目の素数さん:2010/05/12(水) 01:34:27
自演乙
812132人目の素数さん:2010/05/12(水) 01:35:48
> we express this fact by saying that we may equate components in any equation★
> このことを方程式★において要素を同一視する、ということにする。

これを「方程式★の各成分が等しい」という表現で言い表す
くらいが妥当かねえ。
813132人目の素数さん:2010/05/12(水) 01:41:35
理屈っぽい奴はほんと正直者なんだな
特に「数学命です!」なんて時間の無駄で金にもならない数学なんかを本気で頑張っちゃてる奴とか…哀れでしかない
最近の数学板はツンデレっぽい数ヲタしかいないようだけど、そのせいでこの板は個性的な数ヲタがいなくなってつまらなくなった
814132人目の素数さん:2010/05/12(水) 01:48:54
>>813
の類の書き込み、何度目だろ
815132人目の素数さん:2010/05/12(水) 01:52:46
数学者ってのは、受験数学のように答えがある問題をしょっちゅう解いていて定理・公式なら人一倍ストックしてますよって人のことでしょ?(一握りの天才を除いて)
816 ◆27Tn7FHaVY :2010/05/12(水) 01:54:55
シランガナ
817132人目の素数さん:2010/05/12(水) 02:00:38
>>812
その英文を簡潔に翻訳するか、初学者向けの本の趣旨を加味して多少補足するように翻訳するかの違いでしょう。
その1行だけでは書いてあることしか翻訳できませんけど、助動詞mayやthat,byなどは適当にせず、他の専門用語や名詞なんかよりも正確に訳したほうがいいとおもいます。
818132人目の素数さん:2010/05/12(水) 02:06:35
>>810
それは1点を無作為に選んだ場合(N=1)だな。

>>805
>1つだけsのところに落ちて、それ以外は全部sの外側に落ちる。
この考え方で問題ない。>>810が示してくれたN=1の場合を利用すればよい。
あるいは1-F(r)=∫[r→R]f(s)dsが「N個の点すべてが半径rの円内に含まない確率」であることから
F(r)を計算し、それをrで微分すればf(r)がわかる。
819132人目の素数さん:2010/05/12(水) 03:58:26
>>817
プw
820132人目の素数さん:2010/05/12(水) 07:19:39
関数で気になることがあったので質問します

放物線y=x^2-2x-3を原点に関して対象移動したのちx軸方向に平行移動したもので
点(1,0)を通る放物線
という問題なんですが

自分のやり方では
まず平方完成して得られた頂点とx^2の係数を変えたあとに
y=(x-p)^2+4
という式に置き換えてやったのですが解けませんでした

しかしy=x^2-2x-3をそのまま対称移動して

y=-x^2-2x+3とした後に
y=-(x-p)^2-2(x-p)+3と置き換えて代入をするとすんなり解けます
自分のやり方ではどこがだめだったのでしょうか
821132人目の素数さん:2010/05/12(水) 07:40:41
>>820
y=(x-p)^2+4はx^2の係数を変え忘れている
y=-(x-p)^2+4
822132人目の素数さん:2010/05/12(水) 09:32:27
文法厨きめえよw荒らしはENGLISH板に帰れww

あとくだらん質問してる奴も、その程度辞書引いて自分で読めないならお前に洋書ははえーよ。諦めろ。
823132人目の素数さん:2010/05/12(水) 14:06:04
>>822
まったくそのとおりだな

こういう能力の無さはゆとり教育のせいだろうかねえ
824132人目の素数さん:2010/05/12(水) 14:16:51
あるベクターの構成要素間のオーダーは、そのベクターの本質的なことを現す。
たとえば成分では[3,7,-1] != [7,3,-1] であり、一般的に、2つのベクターは次のときイーコールである
(X_1・・・X_n)=(Y_1・・・Y_n)・・・★
さらにこのときに限り、調和(一致)している2つのベクターで全ての成分は次のときイーコールという
X_i=Y_i for i=1,2,3・・・
それゆえ、えぬ次元ベクター集合の2つの空間間の同等関係は、えぬ次スケイラー式・えぬ次方程式の同等性による評価とまったく同じです。
私たちはこの評価方法を
「ベクター集合間の同等関係においては、その2つのベクターは成分が同一であるとみなしてよい」の言い回しで表現します。
825132人目の素数さん:2010/05/12(水) 14:40:10
>>822
たぶん洋書素読ゼミの発表だろ
中卒のおまえにはとっては大学の教育なんてのは一生縁がない世界だろうな(ワ
826132人目の素数さん:2010/05/12(水) 14:52:47
>>802
thus any equation betweem n-vectors is equivalent to n scalar equations:

かくして、n-ベクトルの間の方程式はn個のスカラー方程式に同値である:

訳がなんか違うかなって感じです。
「n-ベクトルの間の方程式」って具体的になんですか?
827132人目の素数さん:2010/05/12(水) 15:10:29
>>825
てことはゆとり大学生か

分数計算もできないようなレベルの大学生ってのは
自分の周りにはいなかったけどなぁ
828132人目の素数さん:2010/05/12(水) 15:22:22
>>824,>>826
おいゴミクズども、とっとと出て行けってw荒らしてんじゃねーよ
829132人目の素数さん:2010/05/12(水) 15:22:22
今日も学校行かなくていいんですか?
830132人目の素数さん:2010/05/12(水) 17:48:44
e,gを2次元平面における正規直交系であるとする。2次元平面上の任意のベクトルxが
x=(e・x)e+(g・x)g
と表せられることを示せ。

よろしくお願いします
831132人目の素数さん:2010/05/12(水) 17:59:05
>e,gを2次元平面における正規直交系であるとする。



なんか言葉足らずな気がするが…
正規ってとこからegを単位ベクトルとでも読みとるの?
832132人目の素数さん:2010/05/12(水) 18:15:08
>>807

今日の敵は何匹倒せたんですか?
833132人目の素数さん:2010/05/12(水) 18:20:33
>>831
そうです。分かりにくくてすいません
834132人目の素数さん:2010/05/12(水) 18:35:52
>>807
そうです。分かりにくくてすいません
835132人目の素数さん:2010/05/12(水) 21:02:36
連分数を使って780x-229y=1の整数解を求めてください
836132人目の素数さん:2010/05/12(水) 21:42:20
820のものですが

y=-(x-p)^2+4と符号を変えた後に(1,0)を代入して計算しても
0=-(1-p)^2+4
0=-(1-2p+p^2)+4
0=-1+2p-p^2+4
0=-p^2+2p+3
p^2-2p-3=0
(p-3)(p+1)=0
P=3,-1でよろしいでしょうか後に記述した式ではP=0,4となるのですが・・・
837132人目の素数さん:2010/05/12(水) 21:54:27
>>836
同じPを使っているけど・・・実は
838132人目の素数さん:2010/05/12(水) 21:58:27
>>836
y=-(x-p)^2+4 で p=3 としたもの

y=-(x-p)^2-2(x-p)+3 で p=4 としたもの
は同じでは?
839132人目の素数さん:2010/05/12(水) 22:25:58
>>833
平面上の直交するベクトルe,gはベースになるので任意のベクトル x は
x=ae+bg とあらわすことができて、
このとき (x・e)=((ae+bg)・e)=(ae・e)+(bg・e)=a(e・e)+b(g・e)=a
同様に(x・g)=b
よってx=(x・e)e+(x・g)g
840132人目の素数さん:2010/05/12(水) 22:34:25
ベースってなんですか?
841132人目の素数さん:2010/05/12(水) 22:37:25
基底
842132人目の素数さん:2010/05/12(水) 22:42:44
キミは捨てられちゃったんだよエリザベス
843132人目の素数さん:2010/05/12(水) 22:48:31
>>839
「正規直交系である」と「ベースになる」では問題が解けなくなるほどの大きな違いってあるんですか?
844132人目の素数さん:2010/05/12(水) 22:53:29
>>838

代入してみたら確かに一致しました最後までやらなかった自分がいけなかったです
ありがとうございました!
845831:2010/05/12(水) 22:54:25
>>839
分かりやすい解説ありがとうございましたm(_ _)m
846830:2010/05/12(水) 22:55:13
830でした
847132人目の素数さん:2010/05/12(水) 22:55:15
>>844
じゃ、次は句読点の使い方を覚えてから来てくれ。
848132人目の素数さん:2010/05/12(水) 23:19:27
>>843
ググレ
849132人目の素数さん:2010/05/14(金) 00:07:59
今年大学1年の者です。他のスレでも質問しましたがここでも書き込みさせていただきます。
線形代数の問題ですが、

行列Aを A=  a0+a1   -a1   とする。
             -a2   a0+a2

 ただしa0,a1,a2は正の定数とする。

(a)固有値λkと、対応する固有ベクトルrkを求めよ。但し(k=1,2)

(b)2×2行列UをU=(r1,r2)とするとき、U^-1AU=Eとなることを示せ。
  但し、Eは固有値λjを対角成分とする対角行列
 
 E= λ1  0
      0  λ2

 である。

(c)xが2成分の縦ベクトル、λをある定数とし、Ax=λxを満たすとき、線形変換y=Vx
(Vは2×2正則行列)を行うと、yは、上の問の対角行列Eを用いて、Ey=λyと書けた。
 正則行列Vを求めよ。

(a),(b)は解けたと思うのですが(c)の意味が理解できません・・
どなたか教えてくれると幸いです。

※行列の()が抜けていますが、うまい表記の仕方がわからなかったので省いてあります。
850132人目の素数さん:2010/05/14(金) 00:10:12
851132人目の素数さん:2010/05/14(金) 00:11:42
>>849
ほかのスレで聞いたのならここでは聞くなよ。
852132人目の素数さん:2010/05/14(金) 00:11:50
>>849
>>1
> 複数のスレッドで質問する行為はご遠慮下さい。
853132人目の素数さん:2010/05/14(金) 00:24:51
失礼しました。今後気をつけます

表記を直しましたがこんなかんじでよろしいでしょうか?



行列Aを A=[[a0+a1,-a2],[-a1,a0+a2]]   とする。
             

 ただしa0,a1,a2は正の定数とする。

(a)固有値λkと、対応する固有ベクトルrkを求めよ。但し(k=1,2)

(b)2×2行列UをU=(r1,r2)とするとき、U^-1AU=Eとなることを示せ。
  但し、Eは固有値λjを対角成分とする対角行列
 
 E= [[λ1,0][0,λ2]]
     

 である。

(c)xが2成分の縦ベクトル、λをある定数とし、Ax=λxを満たすとき、線形変換y=Vx
(Vは2×2正則行列)を行うと、yは、上の問の対角行列Eを用いて、Ey=λyと書けた。
 正則行列Vを求めよ。

(a),(b)は解けたと思うのですが(c)の意味が理解できません・・
どなたか教えてくれると幸いです。

854132人目の素数さん:2010/05/14(金) 02:49:02
>>849,853
>>他のスレでも質問しましたがここでも書き込みさせていただきます。

ほかのスレで聞いたのならここでは聞くなよ。
複数のスレッドで質問する行為はご遠慮下さい。
855132人目の素数さん:2010/05/14(金) 06:56:53
「禁煙場所ですが、これから喫煙させていただけます」みたいな。

ルールに反することでも宣言すれば許されると勘違いしてる屑じゃね?
856132人目の素数さん:2010/05/14(金) 07:07:51
つか、どうやらkummerスレに誤爆したってことみたいだけど、
誤爆なら誤爆で、ふつうに向こうを取り下げて改めて質問しなおす
とかいう行動ができないのか、という点がわからないんだよな。
取り下げるでもなく
> 他のスレでも質問しましたがここでも書き込みさせていただきます。
のような、立場と言葉がちぐはぐな発言になるのは本当によくわからん。
857132人目の素数さん:2010/05/14(金) 08:19:18
そんな気が利く人間ならこんなところで質問しない
858132人目の素数さん:2010/05/14(金) 08:34:11
>失礼しました。今後気をつけます
>表記を直しましたがこんなかんじでよろしいでしょうか?

禁煙所で喫煙してるの注意されて、「失礼しました。今後気をつけます」って言ってそのまま吸い続けるみたいなww
ナメてんのかwwwww
859132人目の素数さん:2010/05/14(金) 10:16:16
853です

みなさんのおっしゃる通りですね。
レポート提出期限が間近なのと問題を解けない焦りで自分のことしか考えずに
行動していました。

問題については取り下げさせていただきます
ありがとうございました。

860132人目の素数さん:2010/05/14(金) 10:21:11
向こうを取下げればいいのに、こっちを取下げるのか。ふーん。
まあ、いいけど、ageてんのは目立ちたいからなのか?
861132人目の素数さん:2010/05/14(金) 10:46:26
>>859

(a)λ1=a0 r1=(1,1) 、 λ2=a0+a1+a2,r2=(a1,-a2)

(b)行列Uの逆行列をもとめて普通の2×2行列の四則計算。よって省略

(c)y=Vx をAy=λyへ代入→AVx=λVx
 Vは正則行列(逆行列をもつ)なので、左側からV^-1を掛けて
 V^-1AVx=λVx
 
 (b)の結果をV^-1AVへ問(b)の結果を適用させると、V^-1AV=E
 
 あとは考えるべし

 あと今後は発言に気をつけろ。以上
862132人目の素数さん:2010/05/14(金) 10:54:06
なにが「今後は発言に気をつけろ」だ。へっ。

取下げるって書いただろ、答えんなよバーカ
863132人目の素数さん:2010/05/14(金) 11:35:53
取り下げたんなら見るなよ・・・
864132人目の素数さん:2010/05/14(金) 12:45:08
いったい誰と戦って(ry
865132人目の素数さん:2010/05/14(金) 13:23:43
水深がh[m]のところでは波は√(10h)[m/s]の速さで動きます。
A点からB点まで波が動くとき、かかる時間を求めてください。

AB=90m
点Aの水深は120m
点Bの水深は30m
点Aの水深から点Bの水深は一次関数的に上昇します
(本当は図があるんですが、わかりづらくてすみません)


という問題なのですが、解けません・・・orz
力を貸してください!
v-tグラフを書こうとしても無理みたいです
866132人目の素数さん:2010/05/14(金) 13:37:27
>>865 普通に微分方程式解きゃいいだろ

dx/dt=√(10h)
h=120-x

t=0のときx=0

なにがムズイの?
867132人目の素数さん:2010/05/14(金) 14:07:16
 あと今後は発言に気をつけろ。以上

これを言いたいが為にいちいちその問題の回答書いたのか。
ご苦労なこったw
868132人目の素数さん:2010/05/14(金) 14:11:26
はいはい
わかったからもう見なくていいよ
869132人目の素数さん:2010/05/14(金) 14:22:33
質問した人間しか見ちゃいけないなら、回答者が居なくなってしまうから、
「見るな」ってのは言い過ぎ。
870132人目の素数さん:2010/05/14(金) 15:09:38
>>868みたいな奴こそ見なくていいよw
871132人目の素数さん:2010/05/14(金) 15:22:08
子供のケンカだなw
872132人目の素数さん:2010/05/14(金) 22:07:42
ここまですべて俺の責任
873132人目の素数さん:2010/05/14(金) 23:24:56
いやいや俺の責任だ
874132人目の素数さん:2010/05/15(土) 00:04:25
>>865

 >>866 から
 (√g)dt = dx/√h = dx/√(120-x),
 (√g)T = ∫[0,90] dx/√(120-x) = [ -2√(120-x) ](x=0,90) = 4√30,
 T = 4√(30/g),
ここに、g [m/s^2] は重力加速度。
875132人目の素数さん:2010/05/15(土) 00:06:14
>>874
その微分方程式を解くことはできますがどうしてdx/dt=√(10h)になるのかわかりませn
876132人目の素数さん:2010/05/15(土) 00:12:53
>>875
>>874は間違ってるからスルーしろ。gなど必要ない。>>866を解け。
877132人目の素数さん:2010/05/15(土) 00:14:49
sin3°の値を
整数と√と四則演算だけで表せ
878132人目の素数さん:2010/05/15(土) 00:17:35
ここは質問スレで、出題スレではない。
879132人目の素数さん:2010/05/15(土) 00:20:55
>>877  どうしても知りたいのなら、教えてやるのもやぶさかではないがな。
880132人目の素数さん:2010/05/15(土) 00:36:22
せめてsin1°って言えよ
881132人目の素数さん:2010/05/15(土) 00:38:09
sin1°はムリ
882132人目の素数さん:2010/05/15(土) 00:41:21
うかつにも解き方を教えてくれることを依頼してませんでした
知りたいので教えてください
883132人目の素数さん:2010/05/15(土) 00:42:39
ググレってみれば誰かがやっていたような・・・
884132人目の素数さん:2010/05/15(土) 00:43:20
sin1゜は三乗根が出るからダメだな。
885132人目の素数さん:2010/05/15(土) 00:49:47
886132人目の素数さん:2010/05/15(土) 00:52:45
二重根号なしでいけますか?
887132人目の素数さん:2010/05/15(土) 01:04:55
>>886 sin3゜のことなら二重根号は不可避。
888132人目の素数さん:2010/05/15(土) 02:45:58
>>877
たとえば
sin(3゚) = sin(18゚-15゚)
    = sin(18゚)cos(15゚) - cos(18゚)sin(15゚),
ここに
sin(18゚) = (√5 - 1) /4,
cos(18゚) = √(10+2√5) /4,
sin(15゚) = √{1-cos(30゚)} /√2 = √(4-2√3) /√8 = (√3 - 1) /√8,
cos(15゚) = √{1+cos(30゚)} /√2 = √(4+2√3) /√8 = (√3 + 1) /√8,
889132人目の素数さん:2010/05/15(土) 03:12:48
>>887
半角などが必要なら二重根号になる可能性があるが
加法定理や倍角などで済むなら二重根号は要らないと思う

あとは>>64などを使えば
(4/15)πー(1/4)πなどで。
890132人目の素数さん:2010/05/15(土) 03:29:41
>>875

水深(水面の高さ)を h(x,t)、流体の速度を u、流体の密度をρ [kg/m^3]、重力加速度をg [m/s^2] とする。
 ∂h/∂t = -div(hu)     (連続の式)
 ∂(ρu)/∂t = - grad(P) = - grad(ρgh) ≒ -ρg・grad(h), (運動方程式)
これより、uについての波動方程式
 ∂^2 u/(∂t)^2 = g・∂^2 (hu)/(∂x)^2 ≒ gh・∂^2 u/(∂x)^2,
を得る。
∴ c = √(gh),
891132人目の素数さん:2010/05/15(土) 07:23:35
ということはx^(1/n)の累乗根は2乗根と3乗根があれば全ての累乗根が表現できるってことですか?
sin(1°)も3乗根があればいいってことですよね。
892132人目の素数さん:2010/05/15(土) 08:17:37
>>891 違います。N゜の三角比に限ります(Nは整数)。
893132人目の素数さん:2010/05/15(土) 09:20:40
>>890
よくわかりました。ありがとう
894132人目の素数さん:2010/05/15(土) 10:03:47
>>889
論より証拠。手を動かしてやってみれば?
二重根号要るってわかるよ。
895132人目の素数さん:2010/05/15(土) 12:50:25
表現に一意性はあるのか?
896132人目の素数さん:2010/05/15(土) 14:22:21
ないだろ。
でも、二重根号のない表記はないだろうさ。
897132人目の素数さん:2010/05/15(土) 15:23:28
>>892
有理数だとどうですか?
sin(a/b °)の三角比でも2乗根、3乗根だけで全ての累乗根が表現できるってことですか?
898132人目の素数さん:2010/05/15(土) 16:24:58
x = 360°/11 のとき
11 sin(x)-220 sin^3(x)+1232 sin^5(x)-2816 sin^7(x)+2816 sin^9(x)-1024 sin^11(x) = 0

俺の手には負えないので逃げる
899132人目の素数さん:2010/05/15(土) 17:25:57
>>897 超無理♡
900132人目の素数さん:2010/05/15(土) 19:21:32
>>877 とりあえず正解置いとくわ。
これで二重根号ナシでもできるんじゃね?と主張する馬鹿はやってみろよと言いたい。

http://en.wikipedia.org/wiki/Exact_trigonometric_constants#3.C2.B0:_60-sided_polygon
901132人目の素数さん:2010/05/15(土) 20:23:52
すまん・・・誰かあほな俺にこの問題解説付きで教えてください(汗


次の微分方程式を解き、yをxの関数であらわせ
dy/dx=2*y/x x=1においてy=3
902132人目の素数さん:2010/05/15(土) 20:48:40
>>901

597 名前:132人目の素数さん :2010/05/15(土) 20:19:11
すまん・・・誰かあほな俺にこの問題解説付きで教えてください(汗


次の微分方程式を解き、yをxの関数であらわせ
dy/dx=2*y/x x=1においてy=3
903132人目の素数さん:2010/05/15(土) 20:55:59
>>901
両辺を積分して、
y=nx+1
x=1 , y=3より
n=2
よって
y=2x+1 . 以上
904132人目の素数さん:2010/05/15(土) 20:56:42
変数分離形なんぞ微分方程式として認めん
905132人目の素数さん:2010/05/15(土) 21:31:36
妹にこの問題教えてと言われたがチンプンカンプンなんだ
答えは出るんだがわかりやすい説明ができん
誰かヘルプ

次の式を、ax+bの形をもつことに注意して因数分解せよ
12x^3+8x^3-13x+3

4x^3+x+1
なんだが・・・
906132人目の素数さん:2010/05/15(土) 21:37:00
問題の意味がチンプン(ry
907132人目の素数さん:2010/05/15(土) 21:37:11
>>905 誤記じゃねーの?

×12x^3+8x^3-13x+3
○12x^3+8x^2-13x+3
908132人目の素数さん:2010/05/15(土) 21:43:32
>>906
すまない、誤記だ
>>907ので頼む
>>907
サンクス
909132人目の素数さん:2010/05/15(土) 21:46:29
>>906
すまない誤記だ
>>907ので頼む
>>907
サンクス
910132人目の素数さん:2010/05/15(土) 21:47:15
>>905
与式=0と置いた時の解を因数定理で探しゃいいだろ。
解の候補は「±(定数項の約数)/(最高次の係数の約数)」で。

参考:高次方程式の解の候補
ttp://questionbox.jp.msn.com/qa5752295.html
911132人目の素数さん:2010/05/15(土) 21:47:17
なんで2回も・・・すまん
912132人目の素数さん:2010/05/15(土) 21:53:01
>>911
サンクス
その解の候補の出し方が分からなかったっぽい
913132人目の素数さん:2010/05/15(土) 21:55:07
>>908
>>910使えば、12x^3+8x^2-13x+3はx=1/2、4x^3+x+1はx=-1/2で0になるから
それぞれ(2x-1)、(2x+1)を因数に持つのはわかるだろ?あとは割り算で次数を下げる。
やれるとこまで繰り返し。2次式になったら解の公式も使える。
914907,910,913:2010/05/15(土) 21:56:44
>>912
おまいは誰にアンカ付けてんだ
915132人目の素数さん:2010/05/15(土) 21:57:04
>>910
因数定理は解を探す方法を述べた定理ではない。
916132人目の素数さん:2010/05/15(土) 22:03:15
>>914
oh・・・
なんかいろいろ申し訳ない・・・
とりあえず、みんなサンクス!
917132人目の素数さん:2010/05/15(土) 22:03:48
>>915
じゃ何を述べた定理だったんですか?
918132人目の素数さん:2010/05/15(土) 22:07:14
>>915
それがどうかしたか?そもそもリンク先読んでないだろ。

『「±(定数項の約数)/(最高次の係数の約数)の中に因数定理を満たす解の候補があることが多い」という話は、
「出題される整係数方程式は、有理数解を持っていることが多い」ということを言っているのです。
後半は、入試問題についての経験則です。』

てーことでテクニックとしての話であって、確実な解法じゃないのよ。
919132人目の素数さん:2010/05/15(土) 22:29:24
>>917-918
因数定理というのは剰余の定理の特別の場合で
f(x)がx-aを因数に持つならばaはf(x)=0の解であり逆もまた成り立つ
という定理。解を探すための定理ではない。

> 『「±(定数項の約数)/(最高次の係数の約数)の中に因数定理を満たす解の候補があることが多い」
というのは
> 因数定理を満たす解の候補
という意味不明な箇所があることを除けば
ガウスの定理(整数係数の範囲で既約ならば有理数係数でも既約)の簡単な系で、
整数係数の範囲で因数分解できるならば一次因子x-aのaは
その形でなければならない。ゆえに
> 入試問題についての経験則です。
というのも正しくない。
920132人目の素数さん:2010/05/15(土) 22:34:04
なにか論破したつもりのバカハケーン
921132人目の素数さん:2010/05/15(土) 22:36:46
>>919
> 因数定理というのは剰余の定理の特別の場合で
> f(x)がx-aを因数に持つならばaはf(x)=0の解であり逆もまた成り立つ
> という定理。解を探すための定理ではない。

誰もこれを否定してないのに、なんでそんなに必死なのか理解できないんだぜ。
922132人目の素数さん:2010/05/15(土) 22:40:26
>>921
> 解を因数定理で探しゃいいだろ
がおかしなことを言ってるといっているだけだが。
923132人目の素数さん:2010/05/15(土) 22:42:31
>>921
> 誰もこれを否定してないのに
>>910>>918で引用されてる文章が否定していますよ。
924132人目の素数さん:2010/05/15(土) 22:43:48
>>922
テクニックとしての話だから無理はないと思うけど?
イチャモンつける理由がわからんんわ。
925132人目の素数さん:2010/05/15(土) 22:45:59
くだらねえな。こっち逝けよ。
高校生のための数学の質問スレPART263
http://science6.2ch.net/test/read.cgi/math/1272681059/
926132人目の素数さん:2010/05/15(土) 22:47:15
質問主の妹さんがこのスレ見てたら驚くだろな
927132人目の素数さん:2010/05/15(土) 22:48:08
何お前ら因数定理なんかで熱くなってんだよw
928132人目の素数さん:2010/05/15(土) 22:48:53
飛び火させんなカス
929132人目の素数さん:2010/05/15(土) 22:50:43
915が必死になってるだけのこと
930132人目の素数さん:2010/05/15(土) 22:52:25
いつまでもつか
みものだわ
931132人目の素数さん:2010/05/15(土) 22:54:06
919が誰と闘ってるのかもわからんが…
932132人目の素数さん:2010/05/15(土) 22:54:53
>>928
高校数学のレベルだと思うぞ
933132人目の素数さん:2010/05/15(土) 22:56:16
因数分解程度でここまで熱くなれるおまいらって…
934132人目の素数さん:2010/05/15(土) 22:56:23
因数分解は結構当てずっぽになることもあるから解き方を教えるって難しいかもね
935132人目の素数さん:2010/05/15(土) 22:57:21
暇だし面白そうだから
適当な時間に適当なレス落として
甚振ってやるか
936132人目の素数さん:2010/05/15(土) 22:57:35
>>920
荒らすなボケ
937132人目の素数さん:2010/05/15(土) 23:00:49
俺は今から休むが
お前らは朝まで続けておけよ
938132人目の素数さん:2010/05/15(土) 23:05:58
>>934
同意。
因数分解で、数学もけっこうカンが必要なんだなと思ったよ。
939132人目の素数さん:2010/05/15(土) 23:54:49
ここまですべて俺の責任
940132人目の素数さん:2010/05/16(日) 00:11:51
↑思ったのだけど お前かっこいいな
941132人目の素数さん:2010/05/16(日) 00:31:01
なんか大変なことになってるな
申し訳ない・・・
942132人目の素数さん:2010/05/16(日) 00:38:10
いや、俺のせいだ。
943132人目の素数さん:2010/05/16(日) 01:07:55
>>940

>>939みたいなレスは荒らしの常套手段なんだなこれが
944132人目の素数さん:2010/05/16(日) 01:19:27
YOUもな
945132人目の素数さん:2010/05/16(日) 02:22:03
遊モナ
946132人目の素数さん:2010/05/16(日) 07:48:24
テンプレ>>4更新版


【関連スレッド】
雑談はここに書け!【37】
http://science6.2ch.net/test/read.cgi/math/1261794262/
分からない問題はここに書いてね332
http://science6.2ch.net/test/read.cgi/math/1273762506/

【業務連絡】
■レスの数が970ぐらいになったら新しいスレッドを立て、そちらには
  関連リンク・注意書きを、古い方には新スレへの誘導を貼るようお願いします。
■単発質問スレと古いスレに書き込まれた質問は、このスレか関連スレに誘導して下さい。
【削除依頼スレッド】
http://qb5.2ch.net/test/read.cgi/saku/1033142451/l50 (レス削除)
http://qb5.2ch.net/test/read.cgi/saku/1106022021/l50 (スレッド削除)
http://qb5.2ch.net/test/read.cgi/saku2ch/1039177898/l50 (重要削除)

━━━━━━━━━━━━━━━━━━━━━━━━━━━━━━

       ◆ わからない問題はここに書いてね 266 ◆
 移転が完了致しましたわ♪ それでは皆様、遠慮なくお使い下さい。

━━━━━━━━━━━━━━━━━━━━━━━━━━━━━━
947132人目の素数さん:2010/05/16(日) 22:48:45
くだらねぇ問題はここへ書け
なくなったのか。。。
948132人目の素数さん:2010/05/16(日) 23:10:52
>>946
重要削除が消えたみたいなん修正


【関連スレッド】
雑談はここに書け!【37】
http://science6.2ch.net/test/read.cgi/math/1261794262/
分からない問題はここに書いてね332
http://science6.2ch.net/test/read.cgi/math/1273762506/

【業務連絡】
■レスの数が970ぐらいになったら新しいスレッドを立て、そちらには
  関連リンク・注意書きを、古い方には新スレへの誘導を貼るようお願いします。
■単発質問スレと古いスレに書き込まれた質問は、このスレか関連スレに誘導して下さい。
【削除依頼スレッド】
http://qb5.2ch.net/test/read.cgi/saku/1033142451/l50 (レス削除)
http://qb5.2ch.net/test/read.cgi/saku/1106022021/l50 (スレッド削除)

━━━━━━━━━━━━━━━━━━━━━━━━━━━━━━

       ◆ わからない問題はここに書いてね 266 ◆
 移転が完了致しましたわ♪ それでは皆様、遠慮なくお使い下さい。

━━━━━━━━━━━━━━━━━━━━━━━━━━━━━━
949132人目の素数さん:2010/05/17(月) 13:40:23
書いてあるとおり970だからまだまだ先だよ
950132人目の素数さん:2010/05/18(火) 00:29:36
X の一点コンパクト化について
X ∪ {∞}の位相Oは次のいずれかをみたす部分集合 E ⊂ X ∪ {∞} の全体である。
ただしO_XはXの位相。

1. E ⊂ X かつ E ∈ O_X
2. ∞ ∈ E かつ X − E がコンパクト

包含写像τ: X → X ∪ {∞}が連続であることの証明で
「開集合Eに対してτ^-1(E)=X-E∈O_Xであるから」とある本に書かれているが、
これは、
「開集合Eに対してτ^-1(E)=E∈O_Xであるから」の間違い?
951132人目の素数さん:2010/05/18(火) 08:12:51
ここに書いてよいのかどうか?、間違ってたら該当スレを教えて。

集合の順序数の話なんだけど・・・
0=φ, 1={0} ,2={0,1} ,3={0,1,2} ってふうにに定義していくのは、すんなりと理解
できるんだけど、問題は、これ→ω={0,1,2,3,・・・}。
「自然数の全体の集合をωと名づける」って定義してしまえば、それで終わり
なんだろうけど、当方の頭の悪さ故なんだが、なんだか、ちょっと
だまされているような感じが、どうも抜けきれない。ちょうど、虚数を習ったとき
と、似た感じがしてね。前記したように、「それが定義なんだ!」と言ってしまえば
それまで、それで必要かつ充分なんだろうけど。
虚数の場合は、例のガウス座標で、なるほどぉ、って納得したんだけれど。
このωなるものが、も一つピンとこなくてねぇ。
なんか、図形的にというか、直感的にというか、ピンと理解できる
なにか(つまり虚数におけるガウス座標のように直感的に理解できるモノ)、
はあるんだろうか?
ω={0,1,2,3,・・・}の、・・・・、が、どうも感覚的に気になってね。
ωのように、「一つ前の順序数が存在しない」順序数は、極限順序数と言う
らしんだけれど。数学は、それ自身に矛盾がなけれは、直感的に分かろうが
分かるまいが、それ自身は正しい、ってことは分かるんだけれど・・・
952132人目の素数さん:2010/05/18(火) 08:33:48
自然数と順序数は全然ちげいぞ
953132人目の素数さん:2010/05/20(木) 09:31:53
a_n → ∞ のとき b_n = (a_1+・・・+a_n)/n → ∞ を教えてください。
954132人目の素数さん:2010/05/20(木) 10:25:27
半径3以下の球は面積が体積以上(半径3でようやく、同一)
なんですが、なんででしょうか?
どんな立体であれ、面積が体積を上回るなんて想像し難いんですが。
955132人目の素数さん:2010/05/20(木) 10:33:43
>>954
面積と体積は単位が違うんで比較にならんのだが。
956132人目の素数さん:2010/05/20(木) 10:34:08
なんでって計算したらそうなっただけだろ
まぁ単位が違うから比べても意味ないけど
とりあえず大小の意味はわかるけどな
薄い直方体とか、たいがいの紙は体積より表面積の数字のほうが大きいだろ
957132人目の素数さん:2010/05/20(木) 10:48:14
厚さ10^6ÅのA4用紙の体積はおおむね1.76*10^25立方Å、面積は1.76*10^19平方Åだな。
958132人目の素数さん:2010/05/20(木) 10:57:39
>>957
違う
959132人目の素数さん:2010/05/20(木) 11:00:12
>>958
違うって言うだけで正答を提示できない奴ww
960132人目の素数さん:2010/05/20(木) 12:35:13
なんでμmじゃないんだよ
961132人目の素数さん:2010/05/20(木) 12:39:12
SI単位系じゃないと気が済まない人か?
962132人目の素数さん:2010/05/20(木) 12:56:40
A4のページの面積は2^(-4)m^2=6.25×10^18Å^2なんで、1.76なんて数が出るのは変だと思うけど。それから>>957は紙を直方体とみなすと大きい面が2つあることを見落としてるっぽい。
963132人目の素数さん:2010/05/20(木) 19:28:04
単位が違う・・ですか・・・なんとなく分かりました。
ありがとうございました。
964132人目の素数さん:2010/05/20(木) 21:55:35
>>960-961
そもそもmmで充分だろ
なんでÅ使っておきながら10^6オーダーの数値なんだよ
965132人目の素数さん:2010/05/20(木) 22:06:07
面白い問題おしえて〜な 十六問目
http://science6.2ch.net/test/read.cgi/math/1254690000/183
966132人目の素数さん:2010/05/21(金) 01:03:46
>>954
>>955が指摘しているように、同列に比べられるようなものではない。
例えば一辺1mの立方体は
体積1立方m、表面積6平方m。mを基準にとれば、数値だけ見ると面積が体積の6倍。

ところが同じ立方体を今度はcmで考えてみると
体積1000000立方cm、表面積60000平方cm。今度は数値だけ比べると面積は体積の0.06倍になってしまった。

次元の違いを無視して無理に比べると、同じ対象を扱ってもこういうことが起きてしまう。

中学校に入ると、一年で文字式というのを習う。
(球の体積・表面積はもっと先で習うのだが、>>954はもう知っているようなのでいいとしよう)
>>954が感じた疑問は文字式の扱い方を知ると解消されると思われる。
 V = 4*π*r^3 /3
 S = 4*π*r^2
 V/S = r/3 なので
半径の数値が3より大きいか小さいかによって
体積の数値が大きいか表面積の数値が大きいかが分かれる。
文字式や一般化の感覚なしで、半径3が切り替わりの境界だと気付いたのは素晴らしいと思う。
967132人目の素数さん:2010/05/21(金) 01:07:23
このつづきは放課後の補講で・・・
968132人目の素数さん:2010/05/21(金) 13:35:57
お昼休みが終わった・・・
969132人目の素数さん:2010/05/21(金) 21:54:20
>>950
「開集合Eに対してτ^-1(E)=E-{∞}∈O_Xであるから」
970凸 ◆fDtv.Vj0j. :2010/05/22(土) 01:05:46
凸関数に関することです。
http://ja.wikipedia.org/wiki/%E5%87%B8%E9%96%A2%E6%95%B0

1変数微分可能な関数が凸関数であるための必要十分条件は、微分が単調非減少であることである。
とあるのですが,これってどう示すのでしょうか?

微分可能性は1階しか仮定しません。

それから,970なのですが次スレ立てた方がいいですか?
971132人目の素数さん:2010/05/22(土) 09:33:48
>>969
>E-{∞}∈O_X
∞がXの元じゃないのは当然だから、それは意味無い。
知ったかぶりはやめたほうがいいよ。
972132人目の素数さん:2010/05/22(土) 11:42:55
は?
973132人目の素数さん:2010/05/22(土) 11:58:04
974132人目の素数さん:2010/05/25(火) 23:23:40
fが連続函数ならば{x|f(x)<b}はRの開集合であるという問題の証明がわかりません
よろしくお願いします。
975132人目の素数さん:2010/05/25(火) 23:28:31
どう解いていいか分からないときは、定義が分かってない事が多い。
976132人目の素数さん:2010/05/25(火) 23:42:58
>>974
連続関数、開集合
どちらの定義も分っていないとこの問には答えられない。
977残24:2010/05/27(木) 10:12:34
fはどこからどこへの写像なの? R から R? b って何? 問題文をきっちり書こう。もし、問題文がちょうどこの通りなら出題者の
国語力もちょっと(ry
978132人目の素数さん:2010/05/27(木) 13:20:26
すみません場違いかもしれないんですが、どーーしても解けなくて気持ち悪いんです。

Q.以下の数字の間に記号を入れ、答えが10になるよう式を作れ。

    2 2 4 8

※但し以下の条件がある
・四則演算 [+,−,×,÷] とかっこ [ ( ) ]のみ使用可能である。
・数字の順番を変えることはできない。
・数字をつなげて、2桁の数字として計算することはできない

解ける方、答えをお願い致します!><;
979132人目の素数さん:2010/05/27(木) 14:27:38
2-2*(4-8)
980132人目の素数さん:2010/05/27(木) 14:30:00
2−2×(4−8)=10。
981132人目の素数さん:2010/05/27(木) 14:34:29
980 無様過ぎ
982132人目の素数さん:2010/05/27(木) 14:54:59
他の解を探してみたらなかった…
983132人目の素数さん:2010/05/27(木) 15:00:07
> 数字の「間に」記号を入れ
解無しってこと?
984132人目の素数さん:2010/05/27(木) 15:35:35
???
985132人目の素数さん:2010/05/27(木) 15:48:28
恥は愛だではない
986132人目の素数さん:2010/05/27(木) 18:24:10
det(a,b)はa、bの張る平行四辺形の面積に符号をつけたものであり、aからbに正の向きに測った角がπ以下のとき正、πより大きいとき負となるそうですが、どのように証明すればよいのでしょうか?
987132人目の素数さん:2010/05/27(木) 18:29:04
x^2+x^2y-x^2-y
=(x-1)(x^2+xy+y)

になる理由が全くわからない
988132人目の素数さん:2010/05/27(木) 18:29:05
>>986
成分表示
989132人目の素数さん:2010/05/27(木) 18:30:36
>>987
ならない。(x-1)(x^2+xy+y) を展開してみて。
990132人目の素数さん:2010/05/27(木) 19:20:16
>>988
a=(a,c)、b=(b,d)とおく
det(a,b)=ad−bc
a、bの張る平行四辺形の面積をS、aからbに正の向きに測った角をθとすると
0≦θ≦πのとき

π<θ<2πのとき

出来ればこれに埋めてもらえませんか?
991132人目の素数さん:2010/05/27(木) 19:33:30
>>990
aの方向にx軸を選ぶと簡単。
992132人目の素数さん:2010/05/27(木) 19:40:27
連立微分方程式の問題です。t=0のとき、r=r0(>0),θ=0と言う条件の下で、
dr/dt=r(1-r)-rtanθ
dθ/dt=1+(1-r)tanθ
を解けというものなのですが、いいやり方が浮かびません。
おそらく非線形だと思われるのですが、解析的に解くとどうなるのでしょうか?
993132人目の素数さん:2010/05/27(木) 23:02:35
次スレ立てます
994132人目の素数さん:2010/05/27(木) 23:07:37
>>993
うむ、よきにはからえ。
995132人目の素数さん:2010/05/27(木) 23:09:56
残6
996132人目の素数さん:2010/05/28(金) 03:15:35
1
997132人目の素数さん:2010/05/28(金) 03:41:24
五十三日五時間。
998132人目の素数さん:2010/05/28(金) 03:42:24
五十三日五時間一分。
999132人目の素数さん:2010/05/28(金) 03:43:24
五十三日五時間二分。
1000132人目の素数さん:2010/05/28(金) 03:44:24
五十三日五時間三分。
10011001
このスレッドは1000を超えました。
もう書けないので、新しいスレッドを立ててくださいです。。。